by ca rahul kumar

133
Current Economics MCQs (Jan 2019 to Feb 20) + Eco Survey 2020 WriteToBeIAS.com Prelims 2020 Online Classes by CA Rahul Kumar Telegram https://t.me/writetobeias1 Youtube https://tinyurl.com/yx22y2yl Page 1 350+ Current Economy MCQs for Prelims 2020 WITH Answers & explanations to WITH ORIGINAL SOURCE LINKS By CA Rahul Kumar Period : Jan 2019 to Feb. 2020 + Economic Survey 1) Join our telegram channel for seprate files of MCQs and Ans & explanations https://t.me/IASeconomy 2) Join Our Youtube Channel https://tinyurl.com/yx22y2yl 3) Live online sessions for detailed explanations will be conducted soon and information will be available on telegram channel 4) March, April & May 2020 Current MCQs will be uploaded on our telegram channel Sources 1) The Hindu and Indian Express 2) P.I.B. 3) Economic Survey (latest 2) 4) Budget 2020 (latest 2) 5) NITI Aayog Strategy document 6) Yojna monthly https://t.me/UPSC_PDF Website https://upscpdf.com https://t.me/UPSC_PDF Website https://upscpdf.com

Upload: others

Post on 15-Feb-2022

1 views

Category:

Documents


0 download

TRANSCRIPT

Current Economics MCQs (Jan

2019 to Feb 20) + Eco Survey 2020

WriteToBeIAS.com Prelims 2020 Online Classes

by CA Rahul Kumar

Telegram https://t.me/writetobeias1 Youtube https://tinyurl.com/yx22y2yl Page 1

350+ Current Economy MCQs for Prelims 2020

WITH

Answers & explanations to

WITH ORIGINAL SOURCE LINKS

By CA Rahul Kumar

Period : Jan 2019 to Feb. 2020 + Economic Survey

1) Join our telegram channel for seprate files of MCQs and Ans &

explanations https://t.me/IASeconomy

2) Join Our Youtube Channel https://tinyurl.com/yx22y2yl

3) Live online sessions for detailed explanations will be conducted

soon and information will be available on telegram channel

4) March, April & May 2020 Current MCQs will be uploaded on

our telegram channel

Sources –

1) The Hindu and Indian Express

2) P.I.B.

3) Economic Survey (latest 2)

4) Budget 2020 (latest 2)

5) NITI Aayog Strategy document

6) Yojna monthly

https://t.me/UPSC_PDF Website ➡ https://upscpdf.com https://t.me/UPSC_PDF

Website ➡ https://upscpdf.com

Current Economics MCQs (Jan

2019 to Feb 20) + Eco Survey 2020

WriteToBeIAS.com Prelims 2020 Online Classes

by CA Rahul Kumar

Telegram https://t.me/writetobeias1 Youtube https://tinyurl.com/yx22y2yl Page 2

INDEX

Sn.

No.

Name of the Topic (No of MCQs) Page No

1. National Income (21) 3

2. Money and Banking (31) 13

3. External Sector including International Institutions (37) 28

4. Money and Capital Markets +NBFC (19) 41

5. Public Finance (7) 48

6. Taxation 52

7. Inflation (4) 60

8. Unemployment and Poverty (25) 63

9. Population and Demography (1) 71

10. International Institution (27) 71

11. Industry (37) 80

12. Agriculture (39) 90

13. Infrastructure (14) 103

14. Social Sector (14) 107

15. Service Sector (4) 111

16. Sustainable Development (22) 113

17. Government Schemes (27) 120

18. Miscellaneous (12) 129

Note – Questions given in blue font are from Economic Survey 2019-20

https://t.me/UPSC_PDF Website ➡ https://upscpdf.com https://t.me/UPSC_PDF

Website ➡ https://upscpdf.com

Current Economics MCQs (Jan

2019 to Feb 20) + Eco Survey 2020

WriteToBeIAS.com Prelims 2020 Online Classes

by CA Rahul Kumar

Telegram https://t.me/writetobeias1 Youtube https://tinyurl.com/yx22y2yl Page 3

NATIONAL INCOME

Q1. Consider following statements:

1. The Fifth Economic Census (5th EC) is being conducted by Ministry of Statistics and Programme

Implementation (MoSPI) in 2019.

2. Economic Census is the complete count of all establishment located within the geographical

boundary of India.

Which of the statements given above is/are correct?

(a) 1 only

(b) 2 only

(c) Both 1 and 2

(d) Neither 1 nor 2

Ans: b

Exp:

Economic Census-2019

The Seventh Economic Census (7th EC) is being conducted by Ministry of Statistics and Programme

Implementation (MoSPI) in 2019. In the current Economic Census, MoSPI has partnered with CSC

e-Governance Services India Limited, a Special Purpose Vehicle under the Ministry of Electronics

and Information Technology as the implementing agency for 7th EC. In the 7th EC, an IT based

digital platform for data capture, validation, report generation and dissemination will be used. The

fieldwork for the 7th EC is likely to commence by the end of June 2019 or next month. The results

of the exercise will be made available after verification and validation of the field work.

About Economic Census

Economic Census will cover all establishments including household enterprises, engaged in

production or distribution of goods/services (other than for the sole purpose of own consumption) in

non-farm agricultural and non-agricultural sector will be counted. The coverage is same as the one in

6th Economic Census, conducted in 2013.

https://pib.gov.in/PressReleseDetail.aspx?PRID=1573472

Q2. The goal to make India a 5 trillion-dollar economy is to be achieved by?

(a) 2022

(b) 2024

(c) 2020

(d)2030

Ans: b

Exp: “The goal to make India a $5 trillion economy by 2024 is challenging, but can surely be

achieved,” said PM, according to an official statement. For this, the States should recognise their core

competence and aim to increase their economy by 2 to 2.5 times, working towards raising GDP targets

right from the district level, he said. He added that the common man’s purchasing power would

increase as a result. He urged the States to focus on their potential to grow exports.

https://www.thehindu.com/news/national/pm-wants-states-to-put-in-concerted-efforts-in-growth-

endeavour/article27949885.ece

Q3. Consider the following statements about the Indian Economy -

1. India’s GDP growth has steadily increased from 2017.

2. India is the fifth largest economy in terms of GDP.

Select the correct code

a. 1 only

b. 2 only

https://t.me/UPSC_PDF Website ➡ https://upscpdf.com https://t.me/UPSC_PDF

Website ➡ https://upscpdf.com

Current Economics MCQs (Jan

2019 to Feb 20) + Eco Survey 2020

WriteToBeIAS.com Prelims 2020 Online Classes

by CA Rahul Kumar

Telegram https://t.me/writetobeias1 Youtube https://tinyurl.com/yx22y2yl Page 4

c. Both a and b

d. Neither a nor b

Ans: b

Exp: Reference- Economic Survey 2019-2020 (Vol 2) crux by CA Rahul Kumar

The deceleration in India’s GDP growth since 2017 has tracked the decline in world output.

Q4.World Economic Outlook is published by

a. OECD

b. World Bank

c. IMF

d. UNCTAD

Ans: c

Exp: Reference- Economic Survey 2019-2020 (Vol 2) crux by CA Rahul Kumar

Q5.Consider the following statements

1. On the supply side, the deceleration in GDP growth has been contributed generally by all

sectors.

2. On the demand side, the deceleration in GDP growth was caused by a decline in the growth

of real fixed investment.

Select the correct code

a. 1 only

b. 2 only

c. Both a and b

d. Neither a nor b

Ans: b

Exp: Reference- Economic Survey 2019-2020 (Vol 2) crux by CA Rahul Kumar

On the supply side, the deceleration in GDP growth has been contributed generally by all sectors save

‘Agriculture and allied activities’ and ‘Public administration, defence, and other services’.

Q6.Consider the following statements

1. The length of India's business cycle is about 13 quarters.

2. The last recorded low growth was in Q2 of 2019-20

Select the correct code

a. 1 only

b. 2 only

c. Both a and b

d. Neither a nor b

Ans: c

Exp: Reference- Economic Survey 2019-2020 (Vol 2) crux by CA Rahul Kumar

(INFLATION CHAPTER)

Q7. Consider the following statements :

1. There has been an uptick in headline inflation.

2. The Wholesale Price Index (WPI) inflation has increased sharply.

Select the correct code

https://t.me/UPSC_PDF Website ➡ https://upscpdf.com https://t.me/UPSC_PDF

Website ➡ https://upscpdf.com

Current Economics MCQs (Jan

2019 to Feb 20) + Eco Survey 2020

WriteToBeIAS.com Prelims 2020 Online Classes

by CA Rahul Kumar

Telegram https://t.me/writetobeias1 Youtube https://tinyurl.com/yx22y2yl Page 5

a. 1 only

b. 2 only

c. Both a and b

d. Neither a nor b

Ans: d

Exp: Reference- Economic Survey 2019-2020 (Vol 2) crux by CA Rahul Kumar

WPI declined sharply from 3.2% in April 2019 to 2.6% in December 2019

(UNEMPLOYMENT CHAPTER)

Q8.Consider the following statements :

1. There has been a decrease in the share of formal employment.

2. In urban regions, there has been a shift of employment from salaried jobs to self- employed.

Select the correct code

a. 1 only

b. 2 only

c. Both a and b

d. Neither a nor b

Ans: d

Exp: Reference Economic Survey 2019-2020 (Vol 2) crux by CA Rahul Kumar

As per the latest available data on employment, there has been an increase in the share of formal

employment, as captured by ‘Regular wage/salaried’, from 17.9 per cent in 2011-12 to 22.8 per cent

in 2017-18.

In urban region, there has been a shift of employment from self- employed to salaried jobs

(MONEY BANKING CHAPTER)

Q9.Consider the following statements

1. The stance of the Monetary Policy Committee of the Reserve Bank of India was

accommodative.

2. The transmission of policy rates has been stagnant across different market segments.

Select the correct code

a. 1 only

b. 2 only

c. Both a and b

d. Neither a nor b

Ans: a

Exp: Reference- Economic Survey 2019-2020 (Vol 2) crux by CA Rahul Kumar

The stance of the Monetary Policy Committee of Reserve Bank of India continued to be

accommodative as it reduced the policy repo rate by 135 basis points since February 2019.

The transmission has varied across different market segments

Q10. Which of the following statements are true about GVA contribution

(i) Share of agriculture and allied sectors in the total GVA of the country has declined.

(ii) The contribution of industrial activities to GVA has increased.

(iii)The contribution of the services sector to GVA has moved ahead faster.

Select the correct code

a. (i), (ii, (iii)

b. (i), (iii) only

https://t.me/UPSC_PDF Website ➡ https://upscpdf.com https://t.me/UPSC_PDF

Website ➡ https://upscpdf.com

Current Economics MCQs (Jan

2019 to Feb 20) + Eco Survey 2020

WriteToBeIAS.com Prelims 2020 Online Classes

by CA Rahul Kumar

Telegram https://t.me/writetobeias1 Youtube https://tinyurl.com/yx22y2yl Page 6

c. (ii), (iii) only

d. All the above

Ans: b

Exp: Reference- Economic Survey 2019-2020 (Vol 2) crux by CA Rahul Kumar

Share of agriculture and allied sectors in the total GVA of the country has declined from 2009-14 to

2014-19 mainly on account of relatively higher growth performance of tertiary sectors.

The contribution of industrial activities to GVA has also declined from 2009-14 to 2014-19.

Services sector has moved ahead faster, distancing itself further from agriculture and industry.

(PUBLIC FINANCE CHAPTER)

Q11. Consider the following statements

1. Highest component in the composition of government expenditure is the expenditure on

defence services, salaries, pensions, interest payments and major subsidies.

2. The share of capital expenditure in total expenditure is envisaged to decline

Which of the following statements are true?

a. 1 only

b. 2 only

c. Both 1 and 2

d. Neither 1 nor 2

Ans: c

Exp: The composition of government expenditure in the last few years reveals that expenditure on

defence services, salaries, pensions, interest payments and major subsidies account for more than

sixty per cent of total expenditure.

Share of capital expenditure in total expenditure is envisaged to decline roughly by a percentage point

in 2019-20 BE over 2018-19 PA.

(PUBLIC FINANCE CHAPTER)

Q12. Which among the following statements is not true?

a. Revenue receipts have grown at a much higher pace during the current financial year.

b. There is low growth in Net Tax revenue.

c. Capital expenditure has grown at roughly three times compared to the last financial year.

d. There has been an increase in the fiscal deficit of the Central Government compared to last

year.

Ans: d

Exp: The Fiscal Deficit is 12.7 % for April-November 2019-2020 against 17.1 % for April-November

2018-2019

(PUBLIC FINANCE CHAPTER)

Q13. Consider the following statements

1. Out of the total liabilities of the Central Government. 90% of it is public debt.

2. The total liabilities of the Central Government, as a ratio of GDP, has been consistently

declining.

Which of the following statements are true?

a. 1 only

b. 2 only

c. Both 1 and 2

d. Neither 1 nor 2

https://t.me/UPSC_PDF Website ➡ https://upscpdf.com https://t.me/UPSC_PDF

Website ➡ https://upscpdf.com

Current Economics MCQs (Jan

2019 to Feb 20) + Eco Survey 2020

WriteToBeIAS.com Prelims 2020 Online Classes

by CA Rahul Kumar

Telegram https://t.me/writetobeias1 Youtube https://tinyurl.com/yx22y2yl Page 7

Ans; c

Exp: Total liabilities of the Central Government at end March 2019 stood at `84.7 lakh crore and 90

per cent of which was public debt.

Total liabilities of the Central Government, as a ratio of GDP, has been consistently declining,

particularly after the enactment of the FRBM Act, 2003.

(PUBLIC FINANCE CHAPTER)

Q14. Consider the following statements

1. The fiscal deficit of the General Government is expected to decline in 2019-20.

2. The combined liabilities of Centre and States have also decreased in 2019.

Which of the following statements are true?

a. 1 only

b. 2 only

c. Both 1 and 2

d. Neither 1 nor 2

Ans: d

Exp: The combined liabilities of Centre and States have increased to 69.8 per cent of GDP as on end-

March 2019 (RE) from 68.5 per cent of GDP as on end-March 2016.

(INDUSTRY CHAPTER)

Q32.Consider the following statements

1. As per industry-wise distribution of recognized startups, IT Services accounted for the highest

percent of startups.

2. Maharashtra, Karnataka and Delhi are the top three performers in terms of State-wise

distribution of recognized startups in India.

Select the correct code

a. 1 only

b. 2 only

c. Both 1 and 2

d. Neither 1 nor 2

Ans: c

Exp: Maharashtra, Karnataka and Delhi are the top three performers in terms of State-wise

distribution of recognized startups in India (Figure 7). As per industry-wise distribution of recognized

startups, IT Services accounted for 13.9 per cent followed by Healthcare and Life Sciences (8.3 per

cent) and education (7.0 per cent)

https://www.indiabudget.gov.in/economicsurvey/doc/echapter_vol2.pdf

(EXTERNAL CHAPTER)

Q33. Consider the following statements

1. During 2019-20, total FDI Equity inflows were higher than 2018-19.

2. 80% of FDI Equity inflows came from Singapore, Mauritius, Netherlands, USA and Japan.

Select the correct code

a. 1 only

b. 2 only

c. Both 1 and 2

d. Neither 1 nor 2

Ans: b

https://t.me/UPSC_PDF Website ➡ https://upscpdf.com https://t.me/UPSC_PDF

Website ➡ https://upscpdf.com

Current Economics MCQs (Jan

2019 to Feb 20) + Eco Survey 2020

WriteToBeIAS.com Prelims 2020 Online Classes

by CA Rahul Kumar

Telegram https://t.me/writetobeias1 Youtube https://tinyurl.com/yx22y2yl Page 8

Exp: During 2019- 20 (up to September, 2019), total FDI Equity inflows were US$26.10 billion as

compared to US$22.66 billion during 2018-19 (up to September, 2019). Out of FDI Equity inflows

of US$26.10 billion during 2019-20 (up to September, 2019), nearly 80 per cent have come mainly

from Singapore, Mauritius, Netherlands, USA and Japan.

https://www.indiabudget.gov.in/economicsurvey/doc/echapter_vol2.pdf

Q34. Consider the following statements

1. India stood at third position in the production of crude steel.

2. It is also the largest consumer of finished steel.

Select the correct code

a. 1 only

b. 2 only

c. Both 1 and 2

d. Neither 1 nor 2

Ans : d

Exp: India stood at second position in the production of crude steel (Figure 9). It is also the third

largest consumer of the finished steel after China and USA.

https://www.indiabudget.gov.in/economicsurvey/doc/echapter_vol2.pdf

Q35. Consider the following statements

1. Textile sector is the biggest employer after agriculture.

2. Exports of textile have increased in 2018-19 compared to 2017-18.

Select the correct code

a. 1 only

b. 2 only

c. Both 1 and 2

d. Neither 1 nor 2

Ans: c

Exp: Textiles contributed 18.0 per cent of manufacturing and 2.0 per cent of GDP in 2017-18. The

share of textiles and clothing in India’s total exports was 12 per cent in 2018- 19. The sector is the

biggest employer after agriculture and it employs 4.5 crore people directly and 6 crore people in allied

sectors.

Exports of textile and clothing products including handicrafts from India have increased to US$40.4

billion in 2018-19 from US$ 39.2 billion in 2017-18 registering a growth of 3 per cent.

https://www.indiabudget.gov.in/economicsurvey/doc/echapter_vol2.pdf

Q36. Consider the following statements

1. India is now the global leader in monthly data consumption.

2. India is the largest energy consumer in the world.

Select the correct code

a. 1 only

b. 2 only

c. Both 1 and 2

d. Neither 1 nor 2

Ans: a

Exp: Total broadband connections increased by about ten times, from 610 lakh in 2014 to 5,946 lakh

in June 2019. This has accelerated the growth in internet traffic, with data usage touching the highest

ever level of 462 lakh terabytes in the year 2018. India is now the global leader in monthly data

https://t.me/UPSC_PDF Website ➡ https://upscpdf.com https://t.me/UPSC_PDF

Website ➡ https://upscpdf.com

Current Economics MCQs (Jan

2019 to Feb 20) + Eco Survey 2020

WriteToBeIAS.com Prelims 2020 Online Classes

by CA Rahul Kumar

Telegram https://t.me/writetobeias1 Youtube https://tinyurl.com/yx22y2yl Page 9

consumption, with average consumption per subscriber per month increasing 157 times from 62 MB

in 2014 to 9.8 GB in June 2019.

India is the third largest energy consumer in the world after USA and China

https://www.indiabudget.gov.in/economicsurvey/doc/echapter_vol2.pdf

(MONEY BANKING CHAPTER)

Q15. Consider following statements:

1. Out of the six Members of Monetary Policy Committee, three Members will be from the RBI and

the other three Members of MPC will be appointed by the Central Government.

2. The Members of the Monetary Policy Committee appointed by the Central Government shall hold

office for a period of two years.

Which of the statements given above is/are correct?

(a) 1 only

(b) 2 only

(c) Both 1 and 2

(d) Neither 1 nor 2

Ans: a

Exp: Out of the six Members of Monetary Policy Committee, three Members will be from the RBI

and the other three Members of MPC will be appointed by the Central Government. The Members of

the Monetary Policy Committee appointed by the Central Government shall hold office for a period

of four years.

https://www.thehindu.com/opinion/editorial/growth-prop/article26207118.ece

Q16. Consider the following:

1. India’s GDP in nominal terms is higher than its GDP in purchasing power parity (PPP) terms.

2. The economy of India is a developing mixed economy.

Which of the statements given above is/are correct?

Select the correct answer using the code given below:

(a) 1 only

(b) 2 only

(c) Both 1 and 2

(d) Neither 1 nor 2

Ans: b

Exp: The economy of India is a developing mixed economy. It is the world’s seventh-largest

economy by nominal GDP and the third-largest by purchasing power parity (PPP). The country ranks

139th in per capita GDP (nominal) with $2,134 and 122nd in per capita GDP (PPP) with $7,783 as

of 2018. After the 1991 economic liberalisation, India achieved 6-7% average GDP growth annually.

https://www.thehindu.com/todays-paper/tp- business/industrial-growth-declines-to-20-month-low-

inflation-up/article26826066.ece

Q17. Consider the following statements:-

1. India currently spends 2.5 per cent of the GDP on health.

2. United States currently spends about 8.5% of its GDP on health.

Which statement/statements given above is/are correct?

Select the correct answer using the codes given below.

(a) 1 only

(b) 2 only

(c) Both 1 and 2

(d) None

https://t.me/UPSC_PDF Website ➡ https://upscpdf.com https://t.me/UPSC_PDF

Website ➡ https://upscpdf.com

Current Economics MCQs (Jan

2019 to Feb 20) + Eco Survey 2020

WriteToBeIAS.com Prelims 2020 Online Classes

by CA Rahul Kumar

Telegram https://t.me/writetobeias1 Youtube https://tinyurl.com/yx22y2yl Page 10

Ans: d

Exp: India currently spends 1.15 per cent of the GDP on health.

https://www.thehindu.com/todays-paper/tp-opinion/moving-away-from-1/article26074736.ece

US spends around 17% of its GDP on health

https://data.worldbank.org/indicator/SH.XPD.CHEX.GD.ZS?locations=US

https://www.cms.gov/Research-Statistics-Data-and-Systems/Statistics-Trends-and-

Reports/NationalHealthExpendData/NationalHealthAccountsHistorical

(PUBLIC FINANCE CHAPTER)

Q18. Which of the following is/are included in Public Debt of India:-

1. Small savings

2. Provident funds

3. External debt

4. Treasury bills

Select the correct answer using the codes given below.

(a) 1, 2 and 4 only

(b) 2 and 4 only

(c) 3 and 4 only

(d) 1, 2, 3 and 4

Ans: c

Exp: In India, public debt refers to a part of the total borrowings by the Union Government which

includes such items as market loans, special bearer bonds, treasury bills and special loans and

securities issued by the Reserve Bank. It also includes the outstanding external debt.

https://www.thehindu.com/todays-paper/tp-business/centres-debt-to-gdp-falls-states-

rises/article26065459.ece

Q19. Consider the following statements:-

1. The Chinese Economy is half of the US Economy in terms of nominal GDP 2018-19.

2. The Indian Economy is one third of the Chinese Economy in terms of nominal GDP 2018-19.

Which statement/statements given above is/are correct?

Select the correct answer using the codes given below.

(a) 1 only

(b) 2 only

(c) Both 1 and 2

(d) None

Ans: c

Exp: The U.S. economy is still the largest in the world with a nominal GDP forecast to exceed USD

21 trillion in 2019. China jumped up to second place with a nominal GDP of USD 9.2 trillion. India

is projected to overtake both the UK by 2020 to become the fifth largest economy in the world with

a nominal GDP of USD 2.9 trillion having overtaken the French economy in 2018.

https://www.thehindu.com/opinion/editorial/shape-of-the-slowdown/article26030889.ece

(MONEY BANKING)

Q20. Consider the following statements:-

1. The Monetary Policy Committee of India is responsible for fixing the benchmark interest rate in

India.

2. The committee is answerable to the RBI Governor if the inflation exceeds the range prescribed for

three consecutive months.

Which statement/statements given above is/are correct?

https://t.me/UPSC_PDF Website ➡ https://upscpdf.com https://t.me/UPSC_PDF

Website ➡ https://upscpdf.com

Current Economics MCQs (Jan

2019 to Feb 20) + Eco Survey 2020

WriteToBeIAS.com Prelims 2020 Online Classes

by CA Rahul Kumar

Telegram https://t.me/writetobeias1 Youtube https://tinyurl.com/yx22y2yl Page 11

Select the correct answer using the codes given below.

(a) 1 only

(b) 2 only

(c) Both 1 and 2

(d) None

Ans: a

Exp: The committee is answerable to the Government of India if the inflation exceeds the range

prescribed for three consecutive months.

https://www.thehindu.com/todays-paper/tp-business/rbi-on-the-horns-of-a-

dilemma/article26046029.ece

Q21. Consider the following Statements-

1. The average growth of the Indian economy between 2014-15 and 2017-18 was 7.3%.

2. Indian economy is projected to be the fastest-growing major economy in 2018-19 and 2019-20 as

per IMF.

Which of the statements given above is/are correct?

Select the correct answer using the code given below:

(a) 1only

(b) 2only

(c) Both1and2

(d) Neither1nor2

Ans: c

Exp: The average growth of the economy between 2014-15 and 2017-18 was 7.3%, the fastest among

major economies in the world, the Ministry said in its Year End Review 2018.“Indian economy is

projected to be the fastest-growing major economy in 2018-19 and 2019-20 [International Monetary

Fund October 2018 database]. This is borne [out] by GDP growth of 7.6% in the first half of 2018-

19,” it said.

https://www.thehindu.com/todays-paper/tp-business/economy-on-track-to-maintain-high-growth-

rate/article25894488.ece

22. Which of the following defines the virtuous cycle of growth ?

a. Increase fixed investment -> accelerates GDP growth -> higher growth in consumption

b. Increase fixed investment -> higher growth in consumption -> accelerates GDP growth

c. Accelerates GDP growth -> Increase fixed investment -> higher growth in consumption

d. Higher growth in consumption ->Increase fixed investment- > accelerates GDP growth

Ans : a

Q31. Ease of Doing Business Report is published by

a. World Bank

b. UNCTAD

c. IMF

d. WTO

Ans: a

Q37. Energy Transition Index is published by

a. EIU

b. UNICEF

c. World Bank

d. World Economic Forum

https://t.me/UPSC_PDF Website ➡ https://upscpdf.com https://t.me/UPSC_PDF

Website ➡ https://upscpdf.com

Current Economics MCQs (Jan

2019 to Feb 20) + Eco Survey 2020

WriteToBeIAS.com Prelims 2020 Online Classes

by CA Rahul Kumar

Telegram https://t.me/writetobeias1 Youtube https://tinyurl.com/yx22y2yl Page 12

Ans: d

https://t.me/UPSC_PDF Website ➡ https://upscpdf.com https://t.me/UPSC_PDF

Website ➡ https://upscpdf.com

Current Economics MCQs (Jan

2019 to Feb 20) + Eco Survey 2020

WriteToBeIAS.com Prelims 2020 Online Classes

by CA Rahul Kumar

Telegram https://t.me/writetobeias1 Youtube https://tinyurl.com/yx22y2yl Page 13

MONEY AND BANKING

Q1. CDR, SDR, S4A, JLF terms recently seen in news are related to

(a) Chandrayaan Mission

(b) Mission Shakti

(c) Restructure of defaulted loans

(d) None of the above

Ans: c

Exp: Restructuring schemes such as CDR, SDR, S4A and the JLF.

https://www.thehindubusinessline.com/opinion/debt-recovery-back-to-square-one-for-

rbi/article26923809.ece

Q2. Consider following statements:

1. RBI is profit making agency by managing borrowings.

2. Reserve Bank of India has transferred an interim surplus of rupees 28,000 crore to the Centre.

3. This is first time such transfer has taken place from RBI to centre.

Which of the statements given above is/are correct?

(a) 1 and 2 only

(b) 2 and 3 only

(c) 1 and 3 only

(d) 1, 2 and 3

Ans: a

Exp: RBI is profit making agency by managing borrowings. Reserve Bank of India has transferred

an interim surplus of rupees 28,000 crore to the Centre. This is second time such transfer has taken

place from RBI to centre.

https://www.thehindu.com/business/centre-must-not-look-upon-the-reserve-bank-as-a-cash-

cow/article26621480.ece

Q3. PCA framework gets triggered when a bank breaches which of the following?

1. Crossing 6% net NPA

2. Return on Assets (ROA) less than 0.25%

3. CRAR less than 9%, but equal or more than 6%

Which of the statements given above is/are correct?

(a) 1 only

(b) 2 and 3 only

(c) 1 and 3 only

(d) 1, 2 and 3

Ans: b (crossing 10% net NPA)

Exp: The trigger points along with structured and discretionary actions that could be taken by the

Reserve Bank are described below:

1. CRAR

(i) CRAR less than 9%, but equal or more than 6%

(ii) CRAR less than 6%, but equal or more than 3%

(iii) CRAR less than 3%

2. Net NPAs

(i) Net NPAs over 10% but less than 15%

(ii) Net NPAs 15% and above point, bank’s Board is called for discussion on corrective plan of action.

https://t.me/UPSC_PDF Website ➡ https://upscpdf.com https://t.me/UPSC_PDF

Website ➡ https://upscpdf.com

Current Economics MCQs (Jan

2019 to Feb 20) + Eco Survey 2020

WriteToBeIAS.com Prelims 2020 Online Classes

by CA Rahul Kumar

Telegram https://t.me/writetobeias1 Youtube https://tinyurl.com/yx22y2yl Page 14

3. ROA less than 0.25%

https://www.thehindu.com/business/rbi-takes-3-banks-off-prompt-corrective-action-

framework/article26379149.ece

Q4. The benchmark rates for Treasury Bills is announced by

(a) RBI

(b) SEBI

(c) Finance Ministry

(d) Financial Benchmarks India Private Ltd (FBIL)

Ans: d

Exp: Banks had four options from which to choose the external benchmark: the repo rate, the 91-day

treasury bill, the 182-day T-bill or any other benchmark interest rate produced by the Financial

Benchmarks India Private Ltd (FBIL).

https://www.thehindu.com/business/Economy/repo-rate/article26367801.ece

Q5. Consider the following statements regarding the Bank Boards Bureau (BBB):

1. It helps Banks in developing strategies and capital raising plans.

2. It appoints the heads of public sector banks and other financial Institutions

3. All the Members and Chairman of BBB are part time appointment.

Select the correct answer using the codes given below:

(a) 1 only

(b) 2 and 3 only

(c) 1 and 3 only

(d) 1, 2 and 3

Ans: d

Exp: With a view to improve the governance of Public Sector Banks (PSBs), the Government of

India has set up an autonomous Banks Board Bureau (BBB). The Bureau will recommend for

selection of heads - Public Sector Banks and Financial Institutions and help banks in developing

strategies and capital raising plans. It will also advise banks how to go ahead with mergers and

acquisitions. Banks Board Bureau has three ex-officio members and three expert members in addition

to Chairman. All the Members and Chairman will be part time. The appointments have been made

for a period of two years.

https://www.thehindubusinessline.com/money-and-banking/bbb-identifies-75-senior-officers-for-

leadership-roles-in-psbs/article26884969.ece

Q6. Which of the following clearly explains the term ‘haircuts’ in the banking system?

(a) It is the difference between loan amount and actual value of the asset used as the collateral.

(b) It is the difference between the actual dues from the borrower and the amount she/he settles with

the bank.

(c) It is the difference between the total interest of the loan and the subvention provided by the

Government.

(d) Both statements (a) and (b) are correct in this context.

Ans: d

Exp: A haircut is the difference between the loan amount and the actual value of the asset used as

collateral. The haircut reflects the lender’s perception of the risk of fall in the value of assets. In the

context of loan recoveries, it is the difference between the actual dues from a borrower and the amount

he settles with the bank. Haircuts are used as a last resort when there is absolutely no hope of a

recovery and the loan is written off for a one time settlement.

https://t.me/UPSC_PDF Website ➡ https://upscpdf.com https://t.me/UPSC_PDF

Website ➡ https://upscpdf.com

Current Economics MCQs (Jan

2019 to Feb 20) + Eco Survey 2020

WriteToBeIAS.com Prelims 2020 Online Classes

by CA Rahul Kumar

Telegram https://t.me/writetobeias1 Youtube https://tinyurl.com/yx22y2yl Page 15

https://www.thehindubusinessline.com/opinion/columns/slate/all-you-wanted-to-know-

about/article26913848.ece

Q7. Consider following statements

1. MUDRA loans are not available for purchase of CNG Tempo/Taxi as public transport carrier.

2. MUDRA loans are available for Khadi Activity.

Which of the statements given above is/are correct?

(a) 1 only

(b) 2 only

(c) Both 1 and 2

(d) Neither 1 nor 2

Ans: b

Exp: MUDRA loans would be available for purchase of CNG Tempo/Taxi, in case the applicant

intends to use the vehicle as public transport carrier. MUDRA loans are applicable for any activity

which results in income generation. As Khadi is one of the eligible activities under Textile sector and

in case MUDRA loans are taken for income generation, the same can be covered.

http://www.pib.nic.in/PressReleseDetail.aspx?PRID=1562143

Q8. With reference to the ‘liquidity coverage ratio (LCR)’, consider the following statements:

1. The LCR assures that financial institutions have the necessary assets on hand to meet any long-

term liquidity disruptions.

2. RBI has allowed banks to use their Government bonds held under the minimum SLR requirement

to count as liquid assets for computing LCR.

3. Government securities under the Marginal Standing Facility and Facility to Avail Liquidity for

Liquidity Coverage Ratio can also be considered for LCR.

Which of the statements given above is/are correct?

(a) 2 and 3 only

(b) 1 and 3 only

(c) 3 only

(d) 1, 2 and 3

Ans: a

Exp:

o The LCR assures that financial institutions have the necessary assets on hand to ride out any

short- term liquidity disruptions.

o The LCR requires that the stock of liquid assets should equal to 90 percent (from January

2018) of the total net cash outflows over 30 days.

o This will increase to 100 percent by January 2019.

o As banks have to maintain such liquid assets over and above the mandated SLR requirement,

the RBI has allowed banks to dip into a portion of their Government bonds held under the

mandated SLR requirement to count as liquid assets for computing LCR.

o Assets allowed as Level 1 High Quality Liquid Assets (HQLAs) for the purpose of computing

LCR of banks include Government securities in excess of the minimum SLR requirement.

o Within the mandatory SLR requirement, HQLA assets include Government securities allowed

by the RBI under the Marginal Standing Facility (MSF) —at 2% of the bank’s NDTL — and

under Facility to Avail Liquidity for Liquidity Coverage Ratio (FALLCR) —at 13% of the

bank’s NDTL.

https://www.thehindubusinessline.com/money-and-banking/rbi-tweaks-lcr-norms-to-boost-

liquidity/article26734605.ece

https://t.me/UPSC_PDF Website ➡ https://upscpdf.com https://t.me/UPSC_PDF

Website ➡ https://upscpdf.com

Current Economics MCQs (Jan

2019 to Feb 20) + Eco Survey 2020

WriteToBeIAS.com Prelims 2020 Online Classes

by CA Rahul Kumar

Telegram https://t.me/writetobeias1 Youtube https://tinyurl.com/yx22y2yl Page 16

Q9. India conferred the Honorary Rank of General of the Indian Army on General Purna Chandra

Thapa. He is from:

(a) Bhutan Army

(b) Nepal Army

(c) Indian Army

(d) Chinese Army

Ans: b

Exp: The President of India, Shri Ram Nath Kovind, conferred the Honorary Rank of General of the

Indian Army on Sukirtimaya Rashtradeep General Purna Chandra Thapa, Chief of the Army Staff,

Nepalese Army, for his commendable military prowess and immeasurable contribution to fostering

Nepal’s long and friendly association with India, at a special Investiture Ceremony held at Rashtrapati

Bhavan today.

http://pib.nic.in/PressReleseDetail.aspx?PRID=1559703

Q10. Consider the following statements: -

1. The Capital Conservation Buffer has been introduced with the objective to increase capital

requirements in good times and decrease the same in bad times.

2. Counter-cyclical Buffer is to ensure that banks maintain a cushion of capital that can be used to

absorb losses during periods of financial and economic stress.

Which of the statements given above is/are correct?

Select the correct answer using the code given below:

(a) 1 only

(b) 2 only

(c) Both 1 and 2

(d) Neither 1 nor 2

Ans: d

Exp: The Capital Conservation Buffer is to ensure that banks maintain a cushion of capital that can

be used to absorb losses during periods of financial and economic stress.

Counter-cyclical Buffer has been introduced with the objective to increase capital requirements in

good times and decrease the same in bad times.

https://www.thehindu.com/todays-paper/tp-business/rbi-notifies-deferment-of-capital-buffer-norms-

as-per-boards-call/article25964632.ece

Q11. Consider the following: -

1. Foreign Currency Assets

2. SDRs

3. Reserve Position in the IMF

4. Gold

Which of the above are included in Foreign exchange reserves of Reserve bank of India?

Select the correct answer using the code given below:

(a) 1, 2 and 4 only

(b) 2, 3and 4 only

(c) 1, 2 and 3 only

(d) 1, 2, 3 and 4

Ans: d

Exp: All of above are included in Foreign exchange reserves of Reserve bank of India.

https://www.thehindu.com/todays-paper/tp-business/rbi-reserves-ratio-among-the-

highest/article25876191.ece

https://t.me/UPSC_PDF Website ➡ https://upscpdf.com https://t.me/UPSC_PDF

Website ➡ https://upscpdf.com

Current Economics MCQs (Jan

2019 to Feb 20) + Eco Survey 2020

WriteToBeIAS.com Prelims 2020 Online Classes

by CA Rahul Kumar

Telegram https://t.me/writetobeias1 Youtube https://tinyurl.com/yx22y2yl Page 17

Q12. Consider the following: -

1. Gross NPA: 100 Crores

2. Provision Aside: 10 Crores

3. At the end of the year, the bank collects 85 Crores only.

From the above figures, what is the figure of the Net NPA of Bank?

Select the correct answer using the code given below:

(a) 15 Crore

(b) 10 Crore

(c) 5 Crore

(d) None of the above

Ans: c

Exp: Now actual NPA is 15 Crores - Provision is 10 Crores = Net NPA = 5 Crores

https://www.thehindu.com/todays-paper/tp-business/npa-ratios-improve-but-are-still-high-

das/article25876202.ece

Q13. Consider the following Statements: -

1. Tokenisation involves a process in which a unique token masks sensitive card details.

2. Customers can be charged a nominal fee for availing tokenisation service.

Which of the statements given above is/are correct?

Select the correct answer using the code given below:

(a) 1only

(b) 2only

(c) Both 1 and 2

(d) Neither 1 nor 2

Ans: a

Exp: Customers cannot be charged for availing tokenisation service.

https://www.thehindu.com/business/customers-cannot-be-charged-for-availing-tokenisation-

service-rbi/article25943839.ece

Q14.Which one of the following best describes the term “Merchant Discount Rate” sometimes seen

in news?

(a)The incentive given by a bank to a merchant for accepting payments through debit cards pertaining

to that bank.

(b)The amount paid back by banks to their customers when they use debit cards for financial

transactions for purchasing goods or services.

(c)The charge to a merchant by a bank for accepting payments from his customers through the bank’s

debit cards.

(d)The incentive given by the Government to merchants for promoting digital payments by their

customers through Point of Sale (PoS) machines and debit cards.

Ans: c

Exp: Merchant discount rate (MDR) is the fee levied by banks from merchants for providing them

with payment (settlement) infrastructure— or POS (point-of-sale) machines. Zero MDR refers to the

proposal made by the Centre in its last budget to do away with all charges (including MDR) on digital

payment transactions.

https://www.thehindu.com/business/no-more-discounts-on-credit-card-payment-at-petrol-

pumps/article29508656.ece

https://t.me/UPSC_PDF Website ➡ https://upscpdf.com https://t.me/UPSC_PDF

Website ➡ https://upscpdf.com

Current Economics MCQs (Jan

2019 to Feb 20) + Eco Survey 2020

WriteToBeIAS.com Prelims 2020 Online Classes

by CA Rahul Kumar

Telegram https://t.me/writetobeias1 Youtube https://tinyurl.com/yx22y2yl Page 18

Q15.The rate of exchange for foreign currencies is determined by

(a) Central Board of Indirect Taxes and Customs

(b) Central Board of Direct Taxes

(c) RBI

(d) None of the above

Ans: a

Exp: In exercise of the powers conferred by Section 14 of the Customs Act, 1962 (52 of 1962), and

in supersession of the notification of the Central Board of Indirect Taxes and Customs No. 66/ 2019-

CUSTOMS (N.T.), dated 19 th September, 2019 except as respects things done or omitted to be done

before such supersession, the Central Board of Indirect Taxes and Customs hereby determines that

the rate of exchange of conversion of each of the foreign currencies.

https://pib.gov.in/PressReleseDetail.aspx?PRID=1586891

Q16.Which one of the following best describes the term “Bill Discounting” sometimes seen in news?

(a) Bill Discounting is a discount/ fee which a bank takes from a seller to release funds before the

credit period ends.

(b) The amount paid back by banks to their customers when they use debit cards for financial

transactions for purchasing goods or services.

(c) The charge to a merchant by a bank for accepting payments from his customers through the bank’s

debit cards.

(d) The incentive given by the Government to merchants for promoting digital payments by their

customers through Point of Sale (PoS) machines and debit cards.

Ans: a

Exp: Bill Discounting is a discount/ fee which a bank takes from a seller to release funds before the

credit period ends. This bill is then presented to seller's customer and full amount is collected.

https://pib.gov.in/PressReleseDetail.aspx?PRID=1587688

Q17.Consider the following statements:

1. National Electronic Funds Transfer (NEFT) is a nation-wide payment system facilitating one-to-

one funds transfer.

2. NEFT system also facilitates one-way cross-border transfer of funds from India to Nepal.

Which of the following statements given above is/are correct?

(a) 1 only

(b) 2 only

(c) Both 1 and 2

(d) Neither 1 nor 2

Ans: c

Exp: National Electronic Funds Transfer (NEFT) is a nation-wide payment system facilitating one-

to-one funds transfer. NEFT system also facilitates one-way cross-border transfer of funds from India

to Nepal.

https://www.thehindu.com/business/Industry/rbi-asks-banks-to-waive-neft-

charges/article30323488.ece

Q18. Consider following statements:

National Payments Corporation of India(NPCI)

1. is an initiative of Bank Board Bureau and RBI under the provisions of the Payment and Settlement

Systems Act, 2007

2. is “Not for Profit” Company, which intends to provide infrastructure to the entire Banking system

in India for physical as well as electronic payment and settlement systems.

https://t.me/UPSC_PDF Website ➡ https://upscpdf.com https://t.me/UPSC_PDF

Website ➡ https://upscpdf.com

Current Economics MCQs (Jan

2019 to Feb 20) + Eco Survey 2020

WriteToBeIAS.com Prelims 2020 Online Classes

by CA Rahul Kumar

Telegram https://t.me/writetobeias1 Youtube https://tinyurl.com/yx22y2yl Page 19

3. has developed the National Electronic Toll Collection (NETC) program to meet the electronic

tolling requirements of the Indian market.

Which of the statements given above is/are correct?

(a) 1only

(b )1 and 2 only

(c) 1 and 3 only

(d) 2 and 3 only

Ans: d

Exp: It is an initiative of Reserve Bank of India (RBI) and Indian Banks’ Association (IBA) under

the provisions of the Payment and Settlement Systems Act, 2007. It is a “Not for Profit” Company

under the provisions of Section 25 of Companies Act 1956 (now Section 8 of Companies Act 2013),

with an intention to provide infrastructure to the entire Banking system in India for physical as well

as electronic payment and settlement systems. National Payments Corporation of India (NPCI) has

developed the National Electronic Toll Collection (NETC) program to meet the electronic tolling

requirements of the Indian market.

Source:https://www.thehindubusinessline.com/money-and-banking/transactions-on-aadhaar-

enabled-payments-system-cross-20-crore-in-july-npci/article29034872.ece

https://www.npci.org.in/about-us-background

Q19.Reserve Bank of India recently revised its guidelines on liquidity risk management framework

for:

(a) Banks

(b) Pension Funds

(c) Insurance Companies

(d) Non-banking finance companies (NBFCs)

Ans: d

Exp: Liquidity coverage ratio (LCR) norms for non-banking financial companies (NBFCs) will take

effect on 1 December 2020. According to the RBI’s final guidelines on liquidity risk management

framework for NBFCs and core investment companies, LCR, which refers to the share of high quality

liquid assets to be set aside to meet short-term obligations, will be introduced in stages. Guidelines

issued by the central bank on liquidity risk management would discourage NBFCs to go for short-

tenor borrowings. Some non-banks in recent times came under financial stress due to the mismatch

in the asset-liability profiles. In light of this, the recently issued guidelines by the Reserve Bank of

India (RBI) would also improve discipline in liquidity maintenance and transparency through better

reporting standards.

Source:https://www.livemint.com/industry/banking/rbi-extends-rollout-date-for-lcr-norms-to-dec-

next-year-11572891939725.html

https://www.financialexpress.com/industry/banking-finance/lcr-frameworks-will-improve-nbfcss-

liability-profile-ind-ra/1771874/

Q20.Consider the following statements:

1. Insolvency and Bankruptcy Board of India was established on 1st October, 2016.

2. It has regulatory oversight over the Insolvency Professionals, Insolvency Professional Agencies,

Insolvency Professional Entities and Information Utilities.

Which of the statements given above is/are correct?

(a)1 only

(b)2 only

(c)Both 1 and 2

(d)Neither 1 nor 2

https://t.me/UPSC_PDF Website ➡ https://upscpdf.com https://t.me/UPSC_PDF

Website ➡ https://upscpdf.com

Current Economics MCQs (Jan

2019 to Feb 20) + Eco Survey 2020

WriteToBeIAS.com Prelims 2020 Online Classes

by CA Rahul Kumar

Telegram https://t.me/writetobeias1 Youtube https://tinyurl.com/yx22y2yl Page 20

Ans: c

Exp: The Insolvency and Bankruptcy Board of India was established on 1st October, 2016 under the

Insolvency and Bankruptcy Code, 2016 (Code). It has regulatory oversight over the Insolvency

Professionals, Insolvency Professional Agencies, Insolvency Professional Entities and Information

Utilities.

Source- https://www.ibbi.gov.in/about

Q21.With reference to Merchant Discount Rate, consider following statements:

1. It is a fee charged from a merchant by a bank for accepting payments from customers through

credit and debit cards in their establishments.

2. All companies with a turnover of Rs 50 crore or more need to provide the facility of payment

through RuPay Debit card and UPI QR code to their customers, under which no MDR fee will be

charged from customers as well as merchants

Which of the statements given above is/are correct?

(a) Both 1 and 2

(b) 2 only

(c) 1only

(d) Neither 1 nor 2

Ans: a

Exp:

The Central Board of Direct Taxes (CBDT) has notified a Budget announcement which said that

business establishments with annual turnover of more than ₹50 crore, shall offer low-cost digital

modes of payment (such as BHIM UPI, UPI QR Code, Aadhaar Pay, Debit Cards, NEFT, RTGS etc.)

to their customers, and also no charge or Merchant Discount Rates (MDR) shall be imposed on

customers and merchants.

Source:https://www.livemint.com/news/india/no-mdr-charges-on-transactions-via-rupay-upi-from-

january-nirmala-sitharaman-11577522520944.html

Q22.Consider the following statements:

1. Unified Payments Interface has been developed by National Payments Corporation of India

(NPCI).

2. With a UPI-enabled bank account, one can make real-time bank-to-bank payments, and even

transact using Aadhaar number, mobile number or Virtual Payment Address (UPI-iD).

Which of the statements given above is/are correct?

(a) Both1 and 2

(b) 1 only

(c) 2 only

(d) Neither 1 nor 2

Ans: a

Exp: Unified Payments Interface has been developed by National Payments Corporation of India

(NPCI). While the payment platform has been around since 2015, it received a big push after

demonetisation drive in November, 2016. Unified Payments Interface (UPI) is essentially a single

platform that merges different banking services and features under one umbrella.

Source:https://www.financialexpress.com/money/bhim-2-0-transacting-with-ease/1760502/

Q23.Which among the following organisations recently launched a global consortium for digital

currency governance?

https://t.me/UPSC_PDF Website ➡ https://upscpdf.com https://t.me/UPSC_PDF

Website ➡ https://upscpdf.com

Current Economics MCQs (Jan

2019 to Feb 20) + Eco Survey 2020

WriteToBeIAS.com Prelims 2020 Online Classes

by CA Rahul Kumar

Telegram https://t.me/writetobeias1 Youtube https://tinyurl.com/yx22y2yl Page 21

(a) IMF

(b) World Bank

(c) Asian Development Bank

(d) World Economic Forum

Ans: d

Exp: The World Economic Forum (WEF) in its annual meeting in Davos, launched a global

consortium for digital currency governance. The Global Consortium for Digital Currency Governance

will aim to increase access to the financial system through innovative policy solutions that are

inclusive and interoperable.

Source:https://www.business-standard.com/article/pti-stories/wef-forms-1st-global-consortium-for-

digital-currency-governance-120012401111_1.html

Q24. With reference to prompt corrective action (PCA) framework, consider the following

statements:

1. The RBI introduced the PCA framework in 2002 as a structured early-intervention mechanism for

banks that become under-capitalised due to poor asset quality, or vulnerable due to loss of

profitability.

2. It is a framework under which banks with weak financial metrics are put under watch by the RBI.

3. The PCA framework deems banks as risky if they slip some trigger points-capital to risk weighted

assets ratio (CRAR), net NPA, Return on Assets (RoA) and Tier 1 Leverage ratio.

Which of the statements given above is/are correct?

(a) 1 and 2 only

(b) 1 and 3 only

(c) 3 only

(d) 1, 2 and 3

Ans: d

Exp: The Reserve Bank of India (RBI) has decided to impose restrictions on urban cooperative banks

(UCBs) for deterioration of financial position, in line with the prompt corrective action (PCA)

framework that is imposed on commercial banks. Under the revised SAF, UCBs will face restrictions

on worsening of 3 parameters: Net non-performing assets exceed 6% of net advances, losses for two

consecutive financial years or have accumulated losses on their balance sheets and Capital adequacy

ratio (CAR) falls below 9%.

Source:https://www.thehindu.com/business/Industry/stressed-urban-cooperative-banks-to-face-pca-

like-curbs/article30496445.ece

Q25. Which among the following in India released the National Strategy for Financial Inclusion

(NSFI) for the period 2019-2024?

(a) Reserve Bank of India

(b) NITI Aayog

(c) Ministry of Rural Development

(d) Ministry of Finance

Ans: a

Exp: Recently, Reserve Bank of India released National Strategy for Financial Inclusion (NSFI) for

the period 2019-2024. It aims to strengthen the ecosystem for various modes of digital financial

services in all Tier-II to Tier VI centres to create the necessary infrastructure to move toward saless-

cash society by March 2022.

Source:https://www.thehindu.com/business/rbi-chalks-out-financial-inclusion-strategy-for-

2024/article30537029.ece

https://t.me/UPSC_PDF Website ➡ https://upscpdf.com https://t.me/UPSC_PDF

Website ➡ https://upscpdf.com

Current Economics MCQs (Jan

2019 to Feb 20) + Eco Survey 2020

WriteToBeIAS.com Prelims 2020 Online Classes

by CA Rahul Kumar

Telegram https://t.me/writetobeias1 Youtube https://tinyurl.com/yx22y2yl Page 22

Q26. With reference to the Block chain technology, consider the following statements:

1. Block chains are open, distributed ledger that can chronologically record transactions between two

parties efficiently in near real time.

2. It generally has key characteristics of centralization, persistency, anonymity and auditability.

Which of the statements given above is/are correct?

(a) 1 only

(b) 2 only

(c) Both 1 and 2

(d) Neither 1 nor 2

Ans: a

Exp: It is well-known that block chain is the core technology of Bitcoin and other crypto currencies,

which has now emerged as a disruptive innovation with a wide range of applications. Block chain

technology generally has key characteristics of decentralization, persistency, anonymity and

auditability.

Source:https://www.thehindubusinessline.com/opinion/using-blockchain-tech-can-improve-

governance/article30601253.ece

Q27.Consider the following statements

1. Small finance banks (SFBs) are primarily set up to undertake basic banking activities of acceptance

of deposits and lending to unserved and underserved sections, including small business units, small

and marginal farmers, micro and small industries etc.

2. Payments Banks can apply for conversion into SFB after five years of operations.

Which of the statements given above is/are correct?

(a) 1 only

(b) 2 only

(c) Both 1 and 2

(d) None of the above

Ans: c

Exp: A small finance bank is primarily set up to undertake basic banking activities of acceptance of

deposits and lending to unserved and underserved sections, including small business units, small and

marginal farmers, micro and small industries and unorganised sector entities. The objectives of setting

up of SBFs is to further financial inclusion and supply of credit "through high technology-low cost

operations" Payments banks willing to convert themselves into small finance banks (SFBs) can apply

for such a licence only after five years of operations, the Reserve Bank of India (RBI) said in the

final guidelines on on-tap licensing for SFBs.

Source:https://www.rbi.org.in/Scripts/BS_PressReleaseDisplay.aspx?prid=48807

https://www.thehindu.com/business/rbi-lays-down-guidelines-for-payments-banks-sfb-

licence/article30195255.ece

Q28.Consider following statements:

1. For enforcement of auditing standards and ensuring the quality of audits, Government has

established National Financial Reporting Authority as an independent regulator.

2. Pursuant to Government’s advice to PSBs, they have been obtaining certified copy of passport of

the promoters/directors and other authorised signatories of companies availing loan facilities of more

than Rs. 100 crore .

Which of the statements given above is/are correct?

(a) 1 only

https://t.me/UPSC_PDF Website ➡ https://upscpdf.com https://t.me/UPSC_PDF

Website ➡ https://upscpdf.com

Current Economics MCQs (Jan

2019 to Feb 20) + Eco Survey 2020

WriteToBeIAS.com Prelims 2020 Online Classes

by CA Rahul Kumar

Telegram https://t.me/writetobeias1 Youtube https://tinyurl.com/yx22y2yl Page 23

(b) 2 only

(c) Both 1 and 2

(d) Neither 1 nor 2

Ans: a

Exp: Increase in detection and reporting of bank frauds due to Government’s proactive measures.

Key steps taken in this regard include:

- Government has issued instruction that all accounts exceeding Rs. 50 crore, if classified as

NPAs, be examined by banks from the angle of possible fraud. In addition, Public Sector

Banks (PSBs) have also been advised to seek a report on the borrower from the Central

Economic Intelligence Bureau, in case an account turns NPA.

- Proactive action has been taken against wilful defaulters, with FIRs being registered by PSBs

against 2,881 wilful defaulters.

- For enforcement of auditing standards and ensuring the quality of audits, Government has

established National Financial Reporting Authority as an independent regulator.

- Fugitive Economic Offenders Act, 2018 has been enacted in order to deter economic offenders

from evading the process of Indian law by remaining outside the jurisdiction of Indian courts.

- Pursuant to Government’s advice to PSBs, they have been obtaining certified copy of passport

of the promoters/directors and other authorised signatories of companies availing loan

facilities of more than Rs. 50 crore, and are ensuring rotational transfer of officials/employees

on sensitive posts.

https://pib.gov.in/PressReleseDetail.aspx?PRID=1573465

Q29. Consider the following:

1. Bank Nationalisation Day came into existence ever since 14 banks were nationalised by the

government on July 19, 1969.

2. First Bank with Indian capital-Punjab National Bank was Founder of the Bank is Bal Gangadhar

Tilak.

Which of the statements given above is/are correct?

(a) 1 only

(b) 2 only

(c) Both 1 and 2

(d) Neither 1 nor 2

Ans: a

Exp: Bank Nationalisation Day came into existence ever since 14 banks were nationalised by the

government on July 19, 1969. First Bank with Indian capital-Punjab National Bank, Founder of the

Bank was Lala Lajpat Rai.

https://www.thehindu.com/opinion/letters/bank-nationalisation/article28970408.ece

Q30.The reserves of RBI include

1. Contingency Fund

2. The currency and gold revaluation reserve (CGRA)

3. Investment revaluation account

Select the correct answer using the code given below.

(a) 1 and 2 only

(b) 3 only

(c) 1 and 3 only

(d) 1, 2 and 3

Ans: d

https://t.me/UPSC_PDF Website ➡ https://upscpdf.com https://t.me/UPSC_PDF

Website ➡ https://upscpdf.com

Current Economics MCQs (Jan

2019 to Feb 20) + Eco Survey 2020

WriteToBeIAS.com Prelims 2020 Online Classes

by CA Rahul Kumar

Telegram https://t.me/writetobeias1 Youtube https://tinyurl.com/yx22y2yl Page 24

Exp: The RBI had Rs 10.46 lakh crore in reserves, bulk of it under two heads—currency and gold

revaluation reserve (Rs 6.91 lakh crore) and contingency reserve (Rs 2.32 lakh crore).

The currency and gold revaluation reserve (CGRA) accounts for 19.11% of total assets and the

contingency reserve for another 6.41%. Back in 2004, a committee under Usha Thorat, then Deputy

Governor, examined the question of what should be the ideal size of RBIs reserves. It suggested that

the CGRA should be 12.26 % of total assets while the contingency reserve should be 5.5 %, totalling

17.76 % in all. But the RBI Board did not accept there commendation and preferred to continue with

the level set by an earlier committee in 1997. That committee, under V. Subrahmanyam, had set a

contingency reserves level of 12 % of total assets. There serves are built through transfers from the

annual surpluses in the profit and loss account of RBI. The balance surplus after transfer to reserves

is given to the Centre as dividend.

https://www.thehindu.com/opinion/editorial/gift-from-rbi-on-transfer-of-funds-to-

government/article29271664.ece

Q31. Consider the following:

1. Prudential Framework for Resolution of Stressed Assets is issued by RBI.

2. The provisions of these directions shall not apply to NBFC.

Which of the statements given above is/are correct?

(a) 1 only

(b) 2 only

(c) Both 1 and 2

(d) Neither 1 nor 2

Ans: a

Exp: These directions shall be called the Reserve Bank of India (Prudential Framework for

Resolution of Stressed Assets) Directions 2019.

The provisions of these directions shall apply to the following entities:

Scheduled Commercial Banks (excluding Regional Rural Banks); All India Term Financial

Institutions (NABARD, NHB, EXIM Bank, and SIDBI); Small Finance Banks; and, Systemically

Important Non-Deposit taking Non-Banking Financial Companies (NBFC-ND-SI) and Deposit

taking Non-Banking Financial Companies (NBFC-D).

https://www.thehindubusinessline.com/money-and-banking/iba-finalises-revised-inter-creditor-

agreement/article28119021.ece#

Q32. The most appropriate term for “convertible digital currencies” or “digital equivalent of cash” is

(a) Digital wallet

(b) e-cash

(c) Crypto currency

(d) All of the above

Ans: c

Exp: Crypto currencies are legally defined as “convertible digital currencies” (according to the US

Financial Crimes Enforcement Network directive issue dearly in 2013) or as a “digital equivalent of

cash” (according to the European legislation EC/2009/110 on electronic money).

https://www.thehindu.com/opinion/op-ed/is-banning-cryptocurrencies-the-

solution/article28787561.ece )

Q33. Consider the following:

1. Anyone with a Bitcoin address can send and receive Bitcoins from anyone else with a Bitcoin

address.

2. Bitcoin has a very small carbon footprint.

https://t.me/UPSC_PDF Website ➡ https://upscpdf.com https://t.me/UPSC_PDF

Website ➡ https://upscpdf.com

Current Economics MCQs (Jan

2019 to Feb 20) + Eco Survey 2020

WriteToBeIAS.com Prelims 2020 Online Classes

by CA Rahul Kumar

Telegram https://t.me/writetobeias1 Youtube https://tinyurl.com/yx22y2yl Page 25

Which of the statements given above is/are correct?

(a) 1 only

(b) 2 only

(c) Both 1 and 2

(d) Neither 1 nor 2

Ans: a

Exp: Bitcoins can be sent directly to anyone, anywhere in the world. The only requirement is that

you should have a Bitcoin address of the recipient. The use of crypto currency Bitcoin emits 22 mega

tonnes of carbon dioxide annually, comparable to the emissions of cities such as Las Vegas and

Vienna, says a study.

https://www.thehindu.com/sci-tech/technology/bitcoin-use-causing-huge-co2-emissions-

study/article27979548.ece

Q34. Consider the following statements:

1. Insolvency Research Foundation (IRF) has been established by SIPI in partnership with the Indian

Institute of Corporate Affairs (IICA).

2. It is an independent research centre to serve public good and help in robust policy making based

on deep research.

Which of the above statements is/are correct?

(a) 1 only

(b) 2 only

(c) Both 1 and 2

(d) Neither 1 nor 2

Ans: c

Exp: Insolvency Research Foundation (IRF) has been established by SIPI in partnership with the

Indian Institute of Corporate Affairs (IICA), Ministry of Corporate Affairs, as an independent

research centre to serve public good and help in robust policy making based on deep research. IRF

will aim to develop a community in pursuit of scholarship in the field of insolvency. It will seek to

strengthen the interaction between government and academics in public policy making. Its activities

will lead to emergence of a cadre of scholars and a robust network of academics, scholars and jurists

in area of insolvency.

https://pib.gov.in/PressReleseDetail.aspx?PRID=1581185

Q35. Consider the following statements:

1. The Insolvency and Bankruptcy Board of India was established on 1st October 2010.

2. It is a unique regulator which regulates a profession as well as processes.

Which of the above statements is/are correct?

(a) 1 only

(b) 2 only

(c) Both 1 and 2

(d) Neither 1 nor 2

Ans: b

Exp: The Insolvency and Bankruptcy Board of India was established on 1st October, 2016 under the

Insolvency and Bankruptcy Code, 2016 (Code). It is a key pillar of the ecosystem responsible for

implementation of the Code that consolidates and amends the laws relating to reorganization and

insolvency resolution of corporate persons, partnership firms and individuals in a time bound manner

for maximization of the value of assets of such persons, to promote entrepreneurship, availability of

credit and balance the interests of all the stakeholders. It is a unique regulator: regulates a profession

as well as processes. It has regulatory oversight over the Insolvency Professionals, Insolvency

https://t.me/UPSC_PDF Website ➡ https://upscpdf.com https://t.me/UPSC_PDF

Website ➡ https://upscpdf.com

Current Economics MCQs (Jan

2019 to Feb 20) + Eco Survey 2020

WriteToBeIAS.com Prelims 2020 Online Classes

by CA Rahul Kumar

Telegram https://t.me/writetobeias1 Youtube https://tinyurl.com/yx22y2yl Page 26

Professional Agencies, Insolvency Professional Entities and Information Utilities. It writes and

enforces rules for processes, namely, corporate insolvency resolution, corporate liquidation,

individual insolvency resolution and individual bankruptcy under the Code. It has recently been

tasked to promote the development of, and regulate, the working and practices of, insolvency

professionals, insolvency professional agencies and information utilities and other institutions, in

furtherance of the purposes of the Code. It has also been designated as the ‘Authority’ under the

Companies (Registered Valuers and Valuation Rules), 2017 for regulation and development of the

profession of valuers in the country.

https://pib.gov.in/PressReleseDetail.aspx?PRID=1581185

Q36. “SWEEKAR” and “SWAGAT” recently seen in news are related to

(a) Biodiesel from cooking oil

(b) Illegal migrants

(c) Digital payment ecosystem

(d) Child adoption

Ans: c

Exp: India’s first indigenously developed payment ecosystem for transport based on National

Common Mobility Card (NCMC) standards was launched in March 2019. This is the first of its kind

in India. NCMC Card, SWEEKAR (Swachalit Kiraya: Automatic Fare Collection System) and

SWAGAT (Swachalit Gate) issued on NCMC Standards.

http://yojana.gov.in/FULLPDFYOJANAAUGUST2019.pdf

Q37.Consider the following statements

1. During 2019-20, the 10-year benchmark G-Sec yield mostly softened.

2. Systemic liquidity in 2019-20 has been largely in surplus.

Select the correct code

a. 1 only

b. 2 only

c. Both 1 and 2

d. Neither 1 nor 2

Ans: c

Exp: G-sec yields traded with a softening bias at the beginning of Q1:2019-20, taking cues from

several positive developments during Q4:2018-19, viz., monetary policy easing and a change in the

policy stance from calibrated tightening to neutral; injection of durable liquidity; announcement of

the voluntary retention route (VRR); and successive benign inflation prints.

Given deficit liquidity conditions in April and May, lending by MFs declined while borrowing by

MFs increased in the collateralised segment during these two months. With systemic liquidity turning

surplus since June, lending by MFs increased while borrowing declined marginally in the

collateralised segment during June- September.

https://www.rbi.org.in/Scripts/PublicationsView.aspx?id=19331

Q38.Consider the following statements

1. Despite a decrease in policy rates, the credit growth in the economy has been declining since

the beginning of this year.

2. The growth of loans from NBFCs has been increasing since the beginning of this year.

Select the correct code

a. 1 only

b. 2 only

https://t.me/UPSC_PDF Website ➡ https://upscpdf.com https://t.me/UPSC_PDF

Website ➡ https://upscpdf.com

Current Economics MCQs (Jan

2019 to Feb 20) + Eco Survey 2020

WriteToBeIAS.com Prelims 2020 Online Classes

by CA Rahul Kumar

Telegram https://t.me/writetobeias1 Youtube https://tinyurl.com/yx22y2yl Page 27

c. Both 1 and 2

d. Neither 1 nor 2

Ans: a

Exp: The loans are not getting cheaper because banks are shying away from transferring the entire

repo rate cut to customers. There is already a reduction of 2.85 per cent in repo rate but banks have

not transferred the benefits to consumers; they have not reduced interest rates in proportion to repo

rate cuts.

Faced with sustained challenges in raising capital at competitive prices and a slowing economy, non-

banking financial (NBFC) sector is set to record their lowest loan growth in over a decade in the

ongoing fiscal year of 2020, rating agency Crisil said on Wednesday.

https://economictimes.indiatimes.com/industry/banking/finance/nbfc-loan-growth-set-to-hit-a-

decadal-low-of-6-8-

report/articleshow/72475297.cms?utm_source=contentofinterest&utm_medium=text&utm_campai

gn=cppst

https://www.businesstoday.in/sectors/banks/loans-not-getting-cheaper-after-reserve-bank-of-india-

repo-rate-cuts-mpc-meet/story/391440.html

Q39.Consider the following statements

1. The insurance penetration for Life insurance has increased from 2011.

2. The non-life insurance has increased consistently.

Select the correct code

a. 1 only

b. 2 only

c. Both 1 and 2

d. Neither 1 nor 2

Ans: b

Exp: According to the report, the life insurance penetration was at 4.6% in 2009 but visibly showed

a downward trend after that. According to Pankaj Razdan, CEO, Aditya Birla Sun Life Insurance Co.

Ltd, volatility in the stock market in 2008-2009 and regulatory changes in 2010 adversely impacted

the sale of Ulip (unit-linked insurance plan) products and premiums. “Penetration in India reflected a

flat trend of 2.7% from FY 2014-15 to 2016-17.

Non-life insurance industry meanwhile grew. The industry recorded a penetration of 0.97% in FY

2018-19 compared to 0.93% in FY 2017-18. The density of non-life insurers also grew marginally

from $18 in FY 2017-18 to $19 in FY 2018-19.

https://www.livemint.com/money/personal-finance/insurance-penetration-in-india-at-3-69-one-of-

the-lowest-across-the-world-1550491451271.html

https://cafemutual.com/news/insurance/18197-indias-insurance-penetration-is-just-37

Q30. Consider the following statements

1. The real GVA of the industrial sector has been steadily declining.

2. The low growth in the industrial sector is primarily due to the manufacturing sector.

Select the correct code

a. 1 only

b. 2 only

c. Both 1 and 2

d. Neither 1 nor 2

Ans: b

https://t.me/UPSC_PDF Website ➡ https://upscpdf.com https://t.me/UPSC_PDF

Website ➡ https://upscpdf.com

Current Economics MCQs (Jan

2019 to Feb 20) + Eco Survey 2020

WriteToBeIAS.com Prelims 2020 Online Classes

by CA Rahul Kumar

Telegram https://t.me/writetobeias1 Youtube https://tinyurl.com/yx22y2yl Page 28

Exp: Industrial sector performance in terms of its contribution in GVA improved in 2018-19 over

2017-18. However, as per the estimates of Gross Domestic Product by National Statistical Office

(NSO), the real GVA of industrial sector grew by 1.6 per cent in first half (H1) (April-September) of

2019-20, as compared to 8.2 per cent in H1 of 2018-19.

The low growth in industrial sector is primarily due to manufacturing sector which registered a

negative growth of 0.2 per cent in 2019-20 H1.

https://www.indiabudget.gov.in/economicsurvey/doc/echapter_vol2.pdf

Q40. In the recent amalgamation of the three banks i.e. Bank of Baroda, Vijaya and Dena Bank,

which of the banks is/are transferee bank/(s)?

(a) Bank of Baroda

(b) Vijaya and Dena Banks

(c) Bank of Baroda and Dena Bank

(d) Bank of Baroda, Vijaya and Dena Bank

Ans: a

Exp: The Union Cabinet approved the amalgamation of the three banks, with Bank of Baroda as the

transferee bank, and Vijaya and Dena as transferor banks. This would mean that the merged entity

would be known as the Bank of Baroda.

https://www.thehindu.com/todays-paper/share-swap-ratio-for-bank-merger-

cleared/article25894656.ece

EXTERNAL SECTOR

Q1. Consider following statements:

1. India recently revoked the Most Favoured Nation status bestowed on Pakistan in 1996 in retaliation

to the terror attack in Uri.

2. Revoking it means India can levy whatever import tariffs it wants.

3. Pakistan has granted MFN status to India.

Which of the statements given above is/are correct?

(a) 1 and 2 only

(b) 2 and 3 only

(c) 2 only

(d) 1, 2 and 3

Ans: c

Exp: India recently revoked the Most Favoured Nation status bestowed on Pakistan in 1996 in

retaliation to the terror attack in Pulwama. Revoking it means India can levy whatever import tariffs

it wants. Pakistan has never granted MFN status to India.

https://www.thehindu.com/news/national/what-is-most-favoured-nation-status/article26283898.ece

Q2. Significant Reduction Exceptions (SRE) sometimes seen in the news recently is associated with

which of the following?

(a) Waiver from US sanctions

(b) Kimberly Process

(c) Cabotage laws

(d) None of the above

Ans: a

https://t.me/UPSC_PDF Website ➡ https://upscpdf.com https://t.me/UPSC_PDF

Website ➡ https://upscpdf.com

Current Economics MCQs (Jan

2019 to Feb 20) + Eco Survey 2020

WriteToBeIAS.com Prelims 2020 Online Classes

by CA Rahul Kumar

Telegram https://t.me/writetobeias1 Youtube https://tinyurl.com/yx22y2yl Page 29

Exp:

US granted waivers from its oil sanction from Iran is also known as Significant Reduction

Exceptions (SRE).

It was granted to eight countries — India, China, Japan, South Korea, Taiwan, Turkey, Italy and

Greece for six months until May 2, 2019.

https://www.thehindubusinessline.com/economy/india-to-get-additional-supplies-to-compensate-

iranian-oil-loss-pradhan/article26918415.ece

Q3. ‘Significant reduction exemptions [SRE]’, recently seen in news

(a) Waiver on Iran Oil imports

(b) Reduction in Carbon footprint

(c) Reduction plastic use

(d) None of the above

Ans: a

Exp: the United States will not issue any additional Significant Reduction Exceptions [SREs] to

existing importers of Iranian oil.

https://www.thehindu.com/news/international/us-ends-waiver-for-india-on-iran-

oil/article26914949.ece

Q4. Consider following statements:

1. Solid plastic waste has been prohibited from import into the country.

2. Polyethylene Terephthalate is a form of Nylon.

Which of the statements given above is/are correct?

(a) 1 only

(b) 2 only

(c) Both 1 and 2

(d) Neither 1 nor 2

Ans: a

Exp: Solid plastic waste has been prohibited from import into the country.

Polyethylene Terephthalate is a form of polyester.

https://www.thehindu.com/sci-tech/energy-and-environment/environment-ministry-plugs-loophole-

that-allowed-plastic-waste-import/article26449791.ece

Q5. Consider following statements:

1. The objective of launching RCEP was mutually beneficial economic partnership agreement among

the ASEAN Member States and ASEAN’s FTA partners.

2. India is not a member of RCEP.

Which of the statements given above is/are correct?

(a) 1 only

(b) 2 only

(c) Both 1 and 2

(d) Neither 1 nor 2

Ans: a

Exp: The objective of launching RCEP was mutually beneficial economic partnership agreement

among the ASEAN Member States and ASEAN’s FTA partners.

India is a member of RCEP.

https://www.pressreader.com/india/the-hindu/20190303/282205127186562

Q6. Consider following statements:

https://t.me/UPSC_PDF Website ➡ https://upscpdf.com https://t.me/UPSC_PDF

Website ➡ https://upscpdf.com

Current Economics MCQs (Jan

2019 to Feb 20) + Eco Survey 2020

WriteToBeIAS.com Prelims 2020 Online Classes

by CA Rahul Kumar

Telegram https://t.me/writetobeias1 Youtube https://tinyurl.com/yx22y2yl Page 30

1. The Financial Action Task Force (FATF) is an inter-governmental body established in 1989 by the

Ministers of its Member jurisdictions.

2. The FATF’s decision making body, the FATF Plenary, meets two times per year.

3. Pakistan is on the ‘grey list’ of countries of risk.

Which of the statements given above is/are correct?

(a) 1 and 2 only

(b) 2 and 3 only

(c) 1 and 3 only

(d) 1, 2 and 3

Ans: c

Exp: The Financial Action Task Force (FATF) is an inter-governmental body established in 1989 by

the Ministers of its Member jurisdictions. The FATF’s decision making body, the FATF Plenary,

meets three times per year. Pakistan is on the ‘grey list’ of countries of risk.

https://www.thehindu.com/news/international/fatf-keeps-pakistan-on-greylist-condemns-pulwama-

attack/article26340558.ece

Q7. With reference to Most Favoured Nation principle, consider the following statements:

1. MFN status gives preferential treatment to certain countries so as to have a better trade relationship.

2. It is applicable only to goods and services are exempted.

Which of the statement given above is/are correct?

(a) 1 only

(b) 2 only

(c) Both 1 and 2

(d) Neither 1 nor 2

Ans: d

Exp: MFN is the principle that mandates equal treatment of all trading partners by a country. Grant

someone a special favour (such as a lower customs duty rate for one of their products) and you have

to do the same for all other WTO members.

Some exceptions are allowed under strict conditions

o Free trade agreements can discriminate between goods originating within and outside the group.

o Developing countries may be given special access to markets.

o Barriers may be raised against unfair trade of products from specific countries.

o Countries are allowed to discriminate, in services, in limited circumstances.

https://www.thehindu.com/news/national/what-is-most-favoured-nation-status/article26283898.ece

Q8. Consider the following statements:

1. 100% FDI under automatic route is permitted in inventory based model of e-commerce or multi-

brand retailing.

2. FDI is not permitted in marketplace model of e-commerce.

Which of the statements given above is/ are correct?

(a) 1 only

(b) 2 only

(c) Both 1 and 2

(d) Neither 1 nor 2

Ans: d

Exp:

o Foreign e-commerce entities would engage only in Business to Business(B2B) e-commerce and not

in Business to Consumer (B2C) e-commerce.

o 100% FDI under automatic route is permitted in marketplace model of e-commerce.

https://t.me/UPSC_PDF Website ➡ https://upscpdf.com https://t.me/UPSC_PDF

Website ➡ https://upscpdf.com

Current Economics MCQs (Jan

2019 to Feb 20) + Eco Survey 2020

WriteToBeIAS.com Prelims 2020 Online Classes

by CA Rahul Kumar

Telegram https://t.me/writetobeias1 Youtube https://tinyurl.com/yx22y2yl Page 31

o FDI is not permitted in inventory based model of e-commerce or multi-brand retailing.

https://www.thehindu.com/business/Industry/optimistic-about-indian-market-despite-changes-in- e-

commerce-fdi-policy-walmart/article26191762.ece

Q9. Consider the following statements w.r.t objectives of EXIM Bank of India: -

1. Act on business principles with due regard to public interest.

2. Providing financial assistance to exporters and importers.

Which statement/statements given above is/are correct?

Select the correct answer using the codes given below.

(a) 1 only

(b) 2 only

(c) Both 1 and 2

(d) None

Ans: c

Exp: both are correct.

http://pib.nic.in/PressReleseDetail.aspx?PRID=1560103

https://www.eximbankindia.in/objectives

Q10. Consider the following statements:-

1. External Commercial Borrowing (ECB) norms are decided by Cabinet Committee of Economic

Affairs.

2. The ceiling for External Commercial Borrowing (ECB) is $750 million or equivalent per financial

year.

Which statement/statements given above is/are correct?

Select the correct answer using the codes given below.

(a) 1 only

(b) 2 only

(c) Both 1 and 2

(d) None

Ans: b

Exp: Reserve Bank of India has decided to liberalise external commercial borrowing (ECB) norms,

allowing all companies that are eligible for receiving foreign direct investment, to raisefunds through

the ECB route. All eligible borrowers can now raise ECBs up to $750 million or equivalent per

financial year under the automatic route replacing the existing sector-wise limits.

https://www.thehindu.com/todays-paper/tp-business/rbi-eases-norms-for-external-commercial-

borrowing/article26008871.ece

Q11. Consider the following statements:

1. RuPay Card has been launched in Bhutan.

2. The electricity generated by the Mangdechhu hydro power project will meet the energy

requirements only of Bhutan.

Which of the above statements is/are correct?

(a) 1 only

(b) 2 only

(c) Both 1 and 2

(d) Neither 1 nor 2

Ans: a

Exp:Most of the electricity generated by the Mangdechhu hydro power project will meet the energy

requirements of Bhutan and the surplus electricity will be exported to India. Mr. Modi also launched

https://t.me/UPSC_PDF Website ➡ https://upscpdf.com https://t.me/UPSC_PDF

Website ➡ https://upscpdf.com

Current Economics MCQs (Jan

2019 to Feb 20) + Eco Survey 2020

WriteToBeIAS.com Prelims 2020 Online Classes

by CA Rahul Kumar

Telegram https://t.me/writetobeias1 Youtube https://tinyurl.com/yx22y2yl Page 32

the RuPay Card in Bhutan by making a purchase at Simtokha Dzong, built in 1629 by Shabdrung

Namgyal, which functions as a monastic and administrative centre and is one of the oldest dzongs in

Bhutan.

https://www.thehindu.com/news/national/pm-modi-inaugurates-mangdechhu-hydroelectric-power-

plant-in-bhutan/article29120270.ece

Q12. Which of the following is not a member of `The Shanghai Cooperation Organisation (SCO)’?

(a) India

(b) Pakistan

(c) Turkmenistan

(d) Tajikistan

Ans: c

Exp: Members of SCO-China, Kazakhstan, Kyrgyzstan, Russia, Tajikistan, and Uzbekistan, India,

Pakistan.

https://pib.gov.in/PressReleseDetail.aspx?PRID=1575039

Q13. Consider the following:

1. The Financial Action Task Force (FATF) is an inter-governmental body established in 1980 by the

Ministers of its Member jurisdictions.

2. Pakistan was placed on the grey list by the FATF.

Which of the statements given above is/are correct?

(a) 1 only

(b) 2 only

(c) Both 1 and 2

(d) Neither 1 nor 2

Ans: b

Exp: The Financial Action Task Force (FATF) is an inter-governmental body established in 1989 by

the Ministers of its Member jurisdictions. The objectives of the FATF are to set standards and

promote effective implementation of legal, regulatory and operational measures for combating money

laundering, terrorist financing and other related threats to the integrity of the international financial

system. The FATF is therefore a “policy-making body” which works to generate the necessary

political will to bring about national legislative and regulatory reforms in these areas. Pakistan was

placed on the grey list by the FATF.

https://www.thehindu.com/news/international/fatf-india-says-pakistan-must-take-verifiable-

irreversible-steps-against-terrorism/article28107371.ece

Q14. Which one of the following sectors does not allow 100% FDI under automatic route as per

the FDI Policy 2018?

(a) Agriculture & Animal Husbandry

(b) Coal & Lignite

(c) Thermal Power

(d) Chit Funds

Ans: d

Exp: FDI is not allowed in chit funds sector. It comes under the list of prohibited sectors, which also

includes lottery business including government/ private lottery, online lotteries etc, chit funds, trading

in transferable development rights (TDR) etc.

Source:http://www.makeinindia.com/policy/foreign-direct-investment

https://t.me/UPSC_PDF Website ➡ https://upscpdf.com https://t.me/UPSC_PDF

Website ➡ https://upscpdf.com

Current Economics MCQs (Jan

2019 to Feb 20) + Eco Survey 2020

WriteToBeIAS.com Prelims 2020 Online Classes

by CA Rahul Kumar

Telegram https://t.me/writetobeias1 Youtube https://tinyurl.com/yx22y2yl Page 33

Q15. India has ratified the Multilateral Convention to Implement Tax Treaty Related Measures to

Prevent Base Erosion and Profit Shifting (MLI). Consider the following statements-

1. The MLI is a result of concerted work by the G20 countries to tackle the issue of base erosion and

profit shifting, something that affects them all.

2. BEPS is a tax avoidance strategy used by multinational companies by exploiting gaps and

mismatches in tax rules to artificially shift profits to low or no-tax locations.

3. India was part of the Ad Hoc Group of more than 100 countries and jurisdictions from the G20,

OECD and other interested countries, which worked on the finalizing the text of the Multilateral

Convention.

Which of the statements given above is/are correct?

Select the correct answer using the code given below:

(a) 2 and 3 only

(b) 1 and 2 only

(c) 1 and 3 only

(d) 1, 2 and 3

Ans. d

Exp: MLI is a result of concerted work by the G20 countries to tackle the issue of base erosion and

profit shifting.

Source:https://www.thehindu.com/business/centre-ratifies-convention-to-curb-company-profit-

shifting/article28264714.ece

Q16. “REER” and “NEER” sometimes mentioned in the news are related to

(a) Indicators of external competitiveness.

(b) Extinct species

(c) Mini satellites

(d) Drones

Ans: a

Exp: The indices of Nominal Effective Exchange Rate (NEER) and Real Effective Exchange Rate

(REER) are used as indicators of external competitiveness. NEER is the weighted average of bilateral

nominal exchange rates of the home currency in terms of foreign currencies.

https://www.thehindubusinessline.com/portfolio/big-story/what-is-reer/article24779863.ece#!

Q17. “Libra” recently seen in news is

(a) A new global currency

(b) Star war movie

(c) Space mission

(d) None of the above

Ans: a

Exp: ‘Libra’ — described as “a new global currency” — was unveiled on Tuesday in a new initiative

in payments for the world’s biggest social network with the potential to bring crypto-money out of

the shadows and into the mainstream.

https://www.thehindu.com/news/international/libra-is-facebooks-

cryptocurrency/article28067219.ece

Q18. Consider following statements:

1. Japan has w.e.f. 5 June 2019 withdrawn India’s GSP benefits.

2. These are unilateral, non-reciprocal and non-discriminatory benefits extended by some developed

countries to developing countries.

Which of the statements given above is/are correct?

https://t.me/UPSC_PDF Website ➡ https://upscpdf.com https://t.me/UPSC_PDF

Website ➡ https://upscpdf.com

Current Economics MCQs (Jan

2019 to Feb 20) + Eco Survey 2020

WriteToBeIAS.com Prelims 2020 Online Classes

by CA Rahul Kumar

Telegram https://t.me/writetobeias1 Youtube https://tinyurl.com/yx22y2yl Page 34

(a) 1 only

(b) 2 only

(c) Both 1 and 2

(d) Neither 1 nor 2

Ans: b

Exp: The United States of America (USA) has w.e.f. 5 June 2019 withdrawn India’s GSP benefits.

These are unilateral, non-reciprocal and non-discriminatory benefits extended by some developed

countries to developing countries. India as part of our bilateral trade discussions, had offered

resolution on significant US requests in an effort to find a mutually acceptable way forward. It is

unfortunate that this did not find acceptance by the US. India, like the US and other nations shall

always uphold its national interest in these matters. We have significant development imperatives and

concerns and our people also aspire for better standards of living. This will remain the guiding factor

in the Government’s approach.

https://pib.gov.in/PressReleseDetail.aspx?PRID=1573061

Q19. Consider the following:

1. The objective of launching RCEP negotiations was to achieve a modern, comprehensive, high-

quality, and mutually beneficial economic partnership agreement among the ASEAN Member States

and ASEAN’s FTA partners.

2. India is not partner to RCEP.

Which of the statements given above is/are correct?

(a) 1 only

(b) 2 only

(c) Both 1 and 2

(d) Neither 1 nor 2

Ans: a

Exp: In August 2012, the 16 Economic Ministers endorsed the Guiding Principles and Objectives for

Negotiating the Regional Comprehensive Economic Partnership. The RCEP negotiations were

launched by Leaders from 10 ASEAN Member States (Brunei Darussalam, Cambodia, Indonesia,

Lao PDR, Malaysia, Myanmar, the Philippines, Singapore, Thailand and Viet Nam) and six ASEAN

FTA partners (Australia, People’s Republic of China, India, Japan, Republic of Korea, and New

Zealand) during the 21st ASEAN Summit and Related Summits in Phnom Penh, Cambodia in

November 2012.

The objective of launching RCEP negotiations is to achieve a modern, comprehensive, high-quality,

and mutually beneficial economic partnership agreement among the ASEAN Member States and

ASEAN’s FTA partners. The RCEP negotiations commenced in early 2013.

https://www.thehindu.com/news/national/quad-one-way-to-fix-regional-issues-australian-

envoy/article27819823.ece

Q20.Which of the following agency develops Logistics Ease Across Different States (LEADS) index?

(a) Ministry of commerce and industry

(b) NITI Ayog

(c) Ministry of statistics and Programme Implementation

(d) PMO

Ans: a

Exp: The index is developed by the Commerce and Industry Ministry along with Deloitte. LEADS

index makes a perception-based assessment of international trade logistics across Indian states and

UTs–focusing on users and stakeholders. Gujarat has retained the highest place in these condedition

of the Logistics Ease Across Different States (LEADS) index.

https://t.me/UPSC_PDF Website ➡ https://upscpdf.com https://t.me/UPSC_PDF

Website ➡ https://upscpdf.com

Current Economics MCQs (Jan

2019 to Feb 20) + Eco Survey 2020

WriteToBeIAS.com Prelims 2020 Online Classes

by CA Rahul Kumar

Telegram https://t.me/writetobeias1 Youtube https://tinyurl.com/yx22y2yl Page 35

https://www.google.com/search?q=LEADS+Index&rlz=1C1CHBF_enIN821IN821&oq=LEADS+I

ndex+&aqs=chrome..69i57j0l7.2997j1j7&sourceid=chrome&ie=UTF-8

Q21. APEDA is mandated with the responsibility of export promotion and development for which of

the following scheduled products:

1. Pickles, Papads and Chutneys

2. Herbal and Medicinal Plants

3. Guar Gum

Which of the above statements is/are correct?

(a) 1 and 2 only

(b) 1 and 2 only

(c) 1 only

(d) 1, 2 and 3

Ans: d

Exp: APEDA is mandated with the responsibility of export promotion and development of the

following scheduled products:

• Fruits, Vegetables and their Products.

• Meat and Meat Products.

• Poultry and Poultry Products.

• Dairy Products.

• Confectionery,Biscuits and Bakery Products.

• Honey, Jaggery and Sugar Products.

• Cocoa and its products, chocolates of all kinds.

• AlcoholicandNon-AlcoholicBeverages.

• Cereal and Cereal Products.

• Groundnuts, Peanuts and Walnuts.

• Pickles, Papads and Chutneys.

• Guar Gum.

• Floriculture and Floriculture Products.

• Herbal and Medicinal Plants.

In addition to this, APEDA has been entrusted with the responsibility to monitor import of sugar.

https://pib.gov.in/PressReleseDetail.aspx?PRID=1586891

Q22. With reference to Regional Comprehensive Economic Partnership (RCEP) consider the

following statements:

1. The objective of launching RCEP negotiation was to achieve a mutually beneficial economic

partnership agreement among the ASEAN member states only.

2. India is not a member to RCEP.

Which of the above statements is/are correct?

(a) 1 only

(b) 2 only

(c) Both 1 and 2

(d) Neither 1 nor 2

Ans: b

Exp: The Association of Southeast Asian Nations (ASEAN) has Free Trade Agreements (FTAs) with

six partners namely People’s Republic of China (ACFTA), Republic of Korea (AKFTA), Japan

(AJCEP), India (AIFTA) and Australia and New Zealand (AANZFTA).

https://pib.gov.in/PressReleseDetail.aspx?PRID=1587688

https://t.me/UPSC_PDF Website ➡ https://upscpdf.com https://t.me/UPSC_PDF

Website ➡ https://upscpdf.com

Current Economics MCQs (Jan

2019 to Feb 20) + Eco Survey 2020

WriteToBeIAS.com Prelims 2020 Online Classes

by CA Rahul Kumar

Telegram https://t.me/writetobeias1 Youtube https://tinyurl.com/yx22y2yl Page 36

Q23. Consider the following statements related to Financial Action Task Force (FATF):

1. The Financial Action Task Force (FATF) is an inter-governmental body.

2. India is a Member of the FATF.

Which of the above statements is/are correct?

(a) 1 only

(b) 2 only

(c) Both 1 and 2

(d) Neither 1 nor 2

Ans: c

Exp: The Financial Action Task Force (FATF) is an inter-governmental body. India is a Member of

the FATF.

https://www.thehindu.com/news/national/india-japan-call-upon-pakistan-to-act-on-

terror/article30125707.ece

Q24. “INSTEX”, recently seen in news is related to

(a) mission to facilitate non-USD transactions

(b) space mission to Saturn

(c) New instrument for transaction in share market

(d) mission to study Artic

Ans: a

Exp: The Instrument in Support of Trade Exchanges (INSTEX) is a European special-purpose

vehicle (SPV) established in January 2019. Its mission is to facilitate non-USD transactions and non-

SWIFT to avoid breaking U.S. sanctions.

https://www.thehindu.com/news/international/6-european-nations-join-iran-barter-

system/article30131764.ece

Q25. Among the following, which one is not member of RCEP?

(a) India

(b) China

(c) Australia

(c) Japan

Ans: a

Exp: India’s decision to leave the Regional Comprehensive Economic Partnership (RCEP) was not

taken at the last minute, Commerce and Industry Minister Piyush Goyal told Parliament, in a

statement, on Tuesday while defending the government’s decision to quit the 16-nation Free Trade

Agreement (FTA) among the ASEAN and other countries.

https://www.thehindu.com/news/national/rcep-decision-not-last-minute-piyush-

goyal/article30269935.ece

Q26.With reference to Shanghai Cooperation Organisation (SCO), consider the following:

1. Shanghai Cooperation Organisation Charter was signed during the St. Petersburg SCO Heads of

State meeting in June 2002.

2. Pakistan will host the 19th council of heads of government of the Shanghai Cooperation

Organisation next year.

3. The SCO is a China-led security bloc in which India and Pakistan were admitted as full members

in 2016.

Which of the statements given above is/are correct?

(a) 1 only

https://t.me/UPSC_PDF Website ➡ https://upscpdf.com https://t.me/UPSC_PDF

Website ➡ https://upscpdf.com

Current Economics MCQs (Jan

2019 to Feb 20) + Eco Survey 2020

WriteToBeIAS.com Prelims 2020 Online Classes

by CA Rahul Kumar

Telegram https://t.me/writetobeias1 Youtube https://tinyurl.com/yx22y2yl Page 37

(b) 1 and 2 only

(c) 1 and 3 only

(d) 1, 2 and 3

Ans: a

Exp: India will host the 19th council of heads of government of the Shanghai Cooperation

Organisation next year. The SCO is a China-led security bloc in which India and Pakistan were

admitted as full members in 2017.

Source:https://www.business-standard.com/article/pti-stories/india-to-host-19th-council-of-heads-

of-government-of-sco-next-year-vladimir-norov-119111201069_1.html

Q27.Recently, European Union has expressed interest in exploring a bilateral investment protection

agreement (BIPA) with:

(a) China

(b) India

(c) Bangladesh

(d) Saudi Arabia

Ans: b

Exp: European Union (EU) has expressed interest in exploring a bilateral investment protection

agreement (BIPA) with India that would be delinked from the proposed free trade agreement (FTA)

where ongoing negotiations are in a state of limbo. The EU wants to follow the model it adopted with

Singapore with which it has recently concluded trade and investment deals separately. The proposal

has been made to India, which is yet to respond.

Source:https://www.thehindubusinessline.com/economy/eu-wants-a-separate-investment-

protection-pact-with-india/article30390422.ece

Q28.A new division named ‘New and Emerging Strategic Technologies (NEST)’ will be set up under

the:

(a)NITI Aayog

(b)Ministry of Skill Development and Entrepreneurship

(c)Ministry of External Affairs

(d)Ministry of Home Affairs

Ans: c

Exp: With India grappling with issues like the security implications of the introduction of 5Gand

artificial intelligence, the ministry of external affairs has announced the setting up of a new division

on New and Emerging Strategic Technologies (NEST). The division will act as the nodal point in

India’s foreign ministry for all matters connected to new and emerging technologies including

exchange of views with foreign governments and coordination with domestic ministries and

departments. It will also help assess foreign policy and international legal implications of emerging

technology and technology-based resources. The desk will also be involved in negotiations to

safeguard Indian interests at multilateral for alike the United Nations or the G20 where rules

governing the use and access to such technologies could be decided.

Source:https://www.livemint.com/news/india/govt-sets-up-division-on-new-and-emerging-

strategic-technologies-11578052462379.html

Q29.With reference to RuPay, consider following statements:

1. It is the first-of-its-kind domestic Debit and Credit Card payment network of India.

2. RuPay is a product of National Securities Depository Limited (NSDL).

3. All RuPay ATM-cum-debit cardholders are presently eligible for accidental death and permanent

disability insurance cover.

https://t.me/UPSC_PDF Website ➡ https://upscpdf.com https://t.me/UPSC_PDF

Website ➡ https://upscpdf.com

Current Economics MCQs (Jan

2019 to Feb 20) + Eco Survey 2020

WriteToBeIAS.com Prelims 2020 Online Classes

by CA Rahul Kumar

Telegram https://t.me/writetobeias1 Youtube https://tinyurl.com/yx22y2yl Page 38

Which of the statements given above is/are correct?

Select the correct answer using the code given below:

(a) 1 and 2 only

(b) 1 and 3 only

(c) 1 only

(d) 3 only

Ans: b

Exp: RuPay is a product of National Payments Corporation of India (NPCI), the umbrella

organisation that powers retail payments in the country.

Source:https://economictimes.indiatimes.com/news/economy/finance/npci-slashes-mdr-for-debit-

cards/articleshow/71117251.cms?from=mdr

https://www.financialexpress.com/money/7-amazing-benefits-of-rupay-debit-credit-cards-

payments-this-system-is-tailor-made-for-india-how-it-can-help-you/887281/

Q30.Consider the following statements

1. CAD to GDP ratio has been significantly improving from 2009-14 to 2014-19

2. The CAD to GDP ratio was lower in the first half of 2019-20.

Select the correct code

a. 1 only

b. 2 only

c. Both 1 and 2

d. Neither 1 nor 2

Ans: c

Exp: The CAD, which was 2.1 per cent of GDP in 2018-19, has improved to 1.5 per cent in H1 of

2019-20 on the back of significant reduction in trade deficit

https://www.indiabudget.gov.in/economicsurvey/doc/vol2chapter/echap01_vol2.pdf

Q31.Consider the following statements

1. An increase in merchandise exports to GDP ratio has a net positive impact on BOP position.

2. India’s top 5 trading partners jointly account for more than 50% of India’s total merchandise

trade.

Select the correct code

a. 1 only

b. 2 only

c. Both 1 and 2

d. Neither 1 nor 2

Ans: a

Exp: An increase in merchandise exports to GDP ratio has a net positive impact on BOP position.

Over the years the merchandise exports to GDP ratio has been declining, entailing a negative impact

on the BoP position.

https://www.indiabudget.gov.in/economicsurvey/doc/echapter_vol2.pdf

Q32.Consider the following statements

1. In 2019-20 (April-November), petroleum products continued to be the largest imported

commodity, in value terms.

2. In the export basket of 2019-20 (April-November), crude petroleum had the largest share.

Select the correct code

https://t.me/UPSC_PDF Website ➡ https://upscpdf.com https://t.me/UPSC_PDF

Website ➡ https://upscpdf.com

Current Economics MCQs (Jan

2019 to Feb 20) + Eco Survey 2020

WriteToBeIAS.com Prelims 2020 Online Classes

by CA Rahul Kumar

Telegram https://t.me/writetobeias1 Youtube https://tinyurl.com/yx22y2yl Page 39

a. 1 only

b. 2 only

c. Both 1 and 2

d. Neither 1 nor 2

Ans: d

Exp: In 2019-20 (April-November), petroleum products continued to be the largest exported

commodity, in value terms. In the import basket of 2019-20 (April-November), crude petroleum had

the largest share followed by gold and petroleum products.

https://www.indiabudget.gov.in/economicsurvey/doc/echapter_vol2.pdf

Q33.Consider the following statements

1. India’s net services surplus has been steadily increasing in relation to GDP.

2. The surplus on net services has been significantly financing the merchandise trade deficit.

Select the correct code

a. 1 only

b. 2 only

c. Both 1 and 2

d. Neither 1 nor 2

Ans: b

Exp: The surplus on net services has been significantly financing the merchandise trade deficit. The

financing reached its peak to about two-thirds of merchandise deficit in 2016-17 before declining to

less than half in the last couple of years (Figure 19). Given a steady decline in net services to GDP

ratio, the extent of financing will steadily fall unless merchandise trade deficit improves in relation

to GDP.

https://www.indiabudget.gov.in/economicsurvey/doc/echapter_vol2.pdf

Q34.Arrange the following items in the increasing order of its share in the service exports basket

(i) Business services

(ii) Software services

(iii) Transportation

(iv) Travel

Select the correct code

a. (ii), (iv), (iii), (i)

b. (iii), (iv), (i), (ii)

c. (iii), (i), (iv), (ii)

d. (ii), (iv), (i), (iii)

Ans: b

Exp: The composition of service exports has remained largely unchanged over the years. Software

services constitute the bulk of it at around 40-45 per cent, followed by business services at about 18-

20 per cent, travel at 11-14 per cent and transportation at 9-11 per cent

https://www.indiabudget.gov.in/economicsurvey/doc/echapter_vol2.pdf

Q35. Logistics Performance Index is released by

a. World Bank

b. UNCTAD

c. IMF

d. EU

Ans: a

https://t.me/UPSC_PDF Website ➡ https://upscpdf.com https://t.me/UPSC_PDF

Website ➡ https://upscpdf.com

Current Economics MCQs (Jan

2019 to Feb 20) + Eco Survey 2020

WriteToBeIAS.com Prelims 2020 Online Classes

by CA Rahul Kumar

Telegram https://t.me/writetobeias1 Youtube https://tinyurl.com/yx22y2yl Page 40

Q36. Consider the following statements

1. Net remittances from Indians employed overseas have been constantly increasing year after

year.

2. The net FDI into the country almost doubled from 2009-14 to 2014-19.

Select the correct code

a. 1 only

b. 2 only

c. Both 1 and 2

d. Neither 1 nor 2

Ans: c

Exp: An increase in net remittances improves the BoP position. Net remittances from Indians

employed overseas has been constantly increasing year after year and has continued doing so with

the amount received in the first half of 2019-20 being more than 50 per cent of the total receivables

in 2018-19

The impressive improvement in BoP position from March, 2014 to March, 2019 is mainly attributed

to almost doubling of net FDI into the country from 2009-14 to 2014-19.

https://www.indiabudget.gov.in/economicsurvey/doc/echapter_vol2.pdf

Q37. What is Net International Investment Position?

a. It measures the gap between two countries' stock of foreign assets at a specific point in time.

b. It measures the gap between a nation’s stock of foreign assets and foreigner’s stock of that

nation’s assets at a specific point in time.

c. It measures the gap between two countries' IMF tranche at a specific point in time.

d. It measures the gap between the export ratio of two countries.

Ans:a

Exp: NIIP measures the gap between a nation’s stock of foreign assets and foreigner’s stock of that

nation’s assets at a specific point in time.

https://www.indiabudget.gov.in/economicsurvey/doc/echapter_vol2.pdf

Q38. Consider the following statements with respect to Kimberley Process Certification

System (KPCS)

1. It came in to effect through a United Nations General Assembly Resolution.

2. It is a non binding agreement to remove conflict diamonds from the global supply chain.

3. India is one of the founding members.

Which of the statements given above are correct?

(a) 1 and 3 only

(b) 2 and 3 only

(c) 1, 2 and 3

(d) None

Ans: a

Exp:

Kimberley process is a joint initiative of Government, International Diamond Industry and Civil

Society to stem the flow of Conflict Diamonds.

Conflict Diamonds are rough diamonds used by rebel movements or their allies to finance conflict

aimed at undermining legitimate governments.

It came into effect in 2003 through a UNGA Resolution with representation from 81countries

including EU.

India is one of the founding members.

https://t.me/UPSC_PDF Website ➡ https://upscpdf.com https://t.me/UPSC_PDF

Website ➡ https://upscpdf.com

Current Economics MCQs (Jan

2019 to Feb 20) + Eco Survey 2020

WriteToBeIAS.com Prelims 2020 Online Classes

by CA Rahul Kumar

Telegram https://t.me/writetobeias1 Youtube https://tinyurl.com/yx22y2yl Page 41

It is a binding agreement to remove conflict diamonds from the global supply chain.

https://www.thehindubusinessline.com/news/centre-fears-broader-definition-of-conflict-diamond-

may-hurt-trade-by-india-others/article26924055.ece

MONEY AND CAPITAL MARKETS (+NBFC)

Q1. Consider following statements:

1. A mortgage is a debt instrument.

2. Mortgages are also known as claims on property

Which of the statements given above is/are correct?

Select the correct answer using the code given below:

(a) 1 only

(b) 2 only

(c) Both 1 and 2

(d) Neither 1 nor 2

Ans: c

Exp: both are correct.

https://www.thehindu.com/todays-paper/tp-business/rbi-rate-cut-will-prevent-mortgage-rate-rise-

moodys/article26813208.ece

Q2. Consider the following statements w.r.t. Competition Commission of India (CCI): -

1. The Commission is required to give opinion on competition issues referred by a statutory authority.

2. CCI consists of a Chairperson and 6 Members appointed by the Central Government.

Which statement/statements given above is/are correct?

Select the correct answer using the codes given below.

(a) 1 only

(b) 2 only

(c) Both 1 and 2

(d) None

Ans: c

Exp: both are correct.

http://pib.nic.in/PressReleseDetail.aspx?PRID=1560364

Q3. Consider following statements-

1. A company incorporated in India and registered under the Companies Act, 1956 / Companies Act,

2013 can issue and operate Prepaid Payment Instruments (PPIs) after receiving authorisation from

RBI.

2. Interest is payable on Prepaid Payment Instruments (PPIs) balances.

Which of the statements given above is/are correct?

(a) 1 only

(b) 2 only

(c) Both 1 and 2

(d) Neither 1 nor 2

Ans: a

Exp: A company incorporated in India and registered under the Companies Act, 1956 / Companies

Act, 2013 can issue and operate Prepaid Payment Instruments (PPIs) after receiving authorisation

from RBI. No interest is payable on Prepaid Payment Instruments (PPIs) balances.

https://t.me/UPSC_PDF Website ➡ https://upscpdf.com https://t.me/UPSC_PDF

Website ➡ https://upscpdf.com

Current Economics MCQs (Jan

2019 to Feb 20) + Eco Survey 2020

WriteToBeIAS.com Prelims 2020 Online Classes

by CA Rahul Kumar

Telegram https://t.me/writetobeias1 Youtube https://tinyurl.com/yx22y2yl Page 42

https://www.thehindu.com/business/Industry/ppi/article26367810.ece

Q4. Consider following statements-

1. The electoral bonds are available at specified branches of the State Bank of India (SBI) for 10 days

each in the months designated by the government from time to time.

2. The bonds may be bought for any value, in multiples of 1,000 or 10,000 rupees only.

Which of the statements given above is/are correct?

(a) 1 only

(b) 2 only

(c) Both 1 and 2

(d) Neither 1 nor 2

Ans: a

Exp: The electoral bonds are available at specified branches of the State Bank of India (SBI) for 10

days each in the months designated by the government from time to time. The bonds may be bought

for any value, in multiples of 1,000, 10,000, 1 lakh, 10 lakh or 1 crore rupees.

https://www.thehindu.com/news/national/the-hindu-explains-what-is-an-electoral-bond-and-how-

do-we-get-one/article22367124.ece

Q5.Which of the following is an example of Gandhian trusteeship philosophy?

(a) Planting of trees

(b) Corporate social responsibility

(c) Feeding poor

(d) Paying taxes

Ans: b

Exp: CSR is a true manifestation of Bapu’s Trusteeship Philosophy: President of India.

https://pib.gov.in/PressReleseDetail.aspx?PRID=1587688

Q6.Consider the following statements regarding:

Assertion (A): Exchange Traded Funds (ETFs) are mutual funds listed and traded on stock exchanges

like shares.

Reason (R): ETF simply copies an index and endeavours to accurately reflect its performance.

Mark correct answer as:

(a) if both A and R are true and R is the correct explanation of A

(b) if both A and R are true but R is not the correct explanation of A

(c) if A is true but R is false

(d) if A is false but R is true

Ans: b

Exp: Exchange Traded Funds (ETFs) are mutual funds listed and traded on stock exchanges like

shares. Index ETFs are created by institutional investors swapping shares in an index basket, for units

in the fund. Usually, ETFs are passive funds where the fund manager doesn’t select stocks on your

behalf. Instead, the ETFs imply copies an index and endeavours to accurately reflect its performance.

In an ETF, one can buy and sell units at prevailing market price on a real time basis during market

hours.

Source:https://www.thehindubusinessline.com/opinion/columns/all-you-wanted-to-know-about-

exchange-traded-funds/article9957174.ece

Q7.Consider the following statements:

https://t.me/UPSC_PDF Website ➡ https://upscpdf.com https://t.me/UPSC_PDF

Website ➡ https://upscpdf.com

Current Economics MCQs (Jan

2019 to Feb 20) + Eco Survey 2020

WriteToBeIAS.com Prelims 2020 Online Classes

by CA Rahul Kumar

Telegram https://t.me/writetobeias1 Youtube https://tinyurl.com/yx22y2yl Page 43

1. Alternative Investment Fund (AIF) means any fund established or incorporated in India which is a

privately pooled investment vehicle which collects funds from sophisticated investors, whether Indian

or foreign, for investing it in accordance with a defined investment policy.

2. AIF also includes funds covered under the SEBI (Mutual Funds) Regulations, 1996, SEBI

(Collective Investment Schemes) Regulations, 1999 or any other regulations of the Board to regulate

fund management activities.

3. An AIF under the SEBI (Alternative Investment Funds) Regulations, 2012 can be established or

incorporated in the form of a trust or a company or a limited liability partnership or a body corporate.

Which of the statements given above is/are correct?

Select the correct answer using the code given below:

(a) 1 and 3 only

(b) 1 and 2 only

(c) 2 and 3 only

(d) 1, 2 and 3

Ans: a

Exp: AIF does not include funds covered under the SEBI (Mutual Funds) Regulations, 1996, SEBI

(Collective Investment Schemes) Regulations, 1999 or any other regulations of the Board to regulate

fund management activities.

Source:https://economictimes.indiatimes.com/markets/stocks/news/sebi-nudges-aifs-to-make-more-

disclosures-for-better-transparency/articleshow/69884129.cms?from=mdr

https://www.sebi.gov.in/sebi_data/faqfiles/jan-2017/1485861425527.pdf

Q8.Credit rating agencies in India are regulated by:

(a) RBI

(b) SEBI

(c) Department of Economic Affairs

(d) Ministry of Corporate Affairs

Ans: b

Exp: All the credit rating agencies in India are regulated by SEBI (Credit Rating Agencies)

Regulations, 1999 of the Securities and Exchange Board of India Act, 1992. Currently there are six

credit rating agencies in India which are registered under SEBI namely-CRISIL, ICRA, CARE,

SMERA, FitchIndia, ONICRA.

Source:https://economictimes.indiatimes.com/markets/stocks/news/rating-agency-ceos-cant-be-

part-of-rating-panel-sebi/articleshow/71917636.cms?from=mdr

https://economictimes.indiatimes.com/news/economy/policy/view-why-it-is-high-time-to-review-

credit-rating-agencies-regulation/articleshow/70682175.cms?from=mdr

Q9.Consider the following statements:

1. National Investment and Infrastructure Fund Limited (NIIFL) is an investor-owned fund manager

2. The funds are registered as Alternative Investment Fund (AIF) with the Securities and Exchange

Board of India (SEBI).

Which of the above statements is/are correct?

(a) 1 only

(b) 2 only

(c) Both 1 and 2

(d) Neither 1 nor 2

Ans: c

https://t.me/UPSC_PDF Website ➡ https://upscpdf.com https://t.me/UPSC_PDF

Website ➡ https://upscpdf.com

Current Economics MCQs (Jan

2019 to Feb 20) + Eco Survey 2020

WriteToBeIAS.com Prelims 2020 Online Classes

by CA Rahul Kumar

Telegram https://t.me/writetobeias1 Youtube https://tinyurl.com/yx22y2yl Page 44

Exp: National Investment and Infrastructure Fund Limited (NIIFL) is an investor-owned fund

manager, anchored by the Government of India (GoI) in collaboration with leading global and

domestic institutional investors.

We manage over USD 3 billion of capital commitments across three funds, each with its distinct

investment strategy.

Our funds have investment mandates to invest in infrastructure assets and related businesses that are

likely to benefit from the long-term growth trajectory of the Indian economy. Our investment

objective is to generate attractive long-term risk-adjusted returns for our investors on a sustainable

basis. We are professionally driven with strong governance processes.

https://www.thehindu.com/business/cppib-to-invest-600-mn-via-niif/article30196199.ece

Q10. Consider the following statements:

1. The Chairman of the Financial Stability and Development Council (FSDC) is the RBI Governor.

2. FSDC was formed to bring greater coordination among financial market regulators.

Which of the statements given above is/are correct?

(a) 1 only

(b) 2 only

(c) Both 1 and 2

(d) Neither 1 nor 2

Ans: b

Exp: The Chairman of the Financial Stability and Development Council (FSDC) is the Finance

Minister of India. FSDC was formed to bring greater coordination among financial market regulators.

https://pib.gov.in/PressReleseDetail.aspx?PRID=1575006

Q11. Consider following statements regarding significance of ‘Regulatory Sandbox’:

1. The Securities and Exchange Board of India and The Insurance Regulatory and Development

Authority of India have announced working guidelines based on The Reserve Bank of India’s

final draft on regulatory sandbox

2. The three sandboxes are aimed at creating an environment of eased regulations for testing new

product-based and technology-based innovations.

3. However, unlike the RBI sandbox, which won’t accept applications from participants with

solutions based on crypto currency, credit rating, credit registry and chain marketing services,

IRDAI has set no such restrictions

Which of the statements given above is/are correct?

(a) 1 only

(b) 2 and 3 only

(c) 1, 2 and 3

(d) 1 and 3

Ans: b

Exp: The Reserve Bank of India, the Securities and Exchange Board of India and The Insurance

Regulatory and Development Authority of India (IRDAI) have announced working guidelines for

their respective sandbox programmes.

https://www.thehindubusinessline.com/markets/sebi-irdai-set-up-regulatory-sandbox-for-fintech-

innovations/article27195596.ece

https://t.me/UPSC_PDF Website ➡ https://upscpdf.com https://t.me/UPSC_PDF

Website ➡ https://upscpdf.com

Current Economics MCQs (Jan

2019 to Feb 20) + Eco Survey 2020

WriteToBeIAS.com Prelims 2020 Online Classes

by CA Rahul Kumar

Telegram https://t.me/writetobeias1 Youtube https://tinyurl.com/yx22y2yl Page 45

https://economictimes.indiatimes.com/industry/banking/finance/insure/irdai-opens-application-

window-for-regulatory-sandbox-participation/articleshow/71117185.cms?from=mdr

Q12. With reference to Core Investment Companies (CICs), consider following statements

1. CICs are non-banking financial companies (NBFCs) holding not less than 80% of their net assets

in the form of investment in equity shares, preference shares, bonds, debentures, debt or loans in

group companies.

2. Unlike NBFCs which are required to constitute board level committees, no such standards are

mandated for CICs.

Which of the statements given above is/are correct?

Select the correct answer using the code given below:

(a) 1 only

(b) 2 only

(c) Both 1 and 2

(d) Neither 1 nor 2

Ans: b

Exp: CICs are non-banking financial companies (NBFCs) holding not less than 90% of their net

assets in the form of investment in equity shares, preference shares, bonds, debentures, debt or loans

in group companies

Source:https://www.livemint.com/industry/banking/rbi-panel-suggests-stronger-corporate-

governance-in-core-investment-companies-11573038504883.html

Q13. Consider following statements:

1. Bharat Bond ETF would be the first corporate Bond ETF in the country.

2. It will invest in a portfolio of bonds of CPSE, CPSU, CPFI or any other Government or Private

organizations that matures on or before the maturity date of the ETF.

Which of the statements given above is/are correct?

(a) 1 only

(b) 2 only

(c) Both 1 and 2

(d) Neither 1 nor 2

Ans: a

Exp: Bharat Bond ETF would be the first corporate Bond ETF in the country.

Features of Bharat Bond ETF:

ETF will be a basket of bonds issued by CPSE/CPSU/CPFI/any other Government organization

Bonds (Initially, all AAA rated bonds)

• Tradable on exchange

• Small unit size Rs 1,000

• Transparent NAV (Periodic live NAV during the day)

• Transparent Portfolio (Daily disclosure on website)

• Low cost (0.0005%)

Bharat Bond ETF Structure:

• Each ETF will have a fixed maturity date

• The ETF will track the underlying Index on risk replication basis, i.e. matching Credit Quality

and Average Maturity of the Index

• Will invest in a portfolio of bonds of CPSE, CPSU, CPFI or any other Government organizations

that matures on or before the maturity date of the ETF

• As of now, it will have 2 maturity series - 3 and 10 years. Each series will have a separate index

of the same maturity series.

https://t.me/UPSC_PDF Website ➡ https://upscpdf.com https://t.me/UPSC_PDF

Website ➡ https://upscpdf.com

Current Economics MCQs (Jan

2019 to Feb 20) + Eco Survey 2020

WriteToBeIAS.com Prelims 2020 Online Classes

by CA Rahul Kumar

Telegram https://t.me/writetobeias1 Youtube https://tinyurl.com/yx22y2yl Page 46

Index Methodology:

• Index will be constructed by an independent index provider – National Stock Exchange

• Different indices tracking specific maturity years - 3 and 10 years.

https://pib.gov.in/PressReleseDetail.aspx?PRID=1594815

Q14. With reference to Sovereign Gold Bonds (SGB) consider the following statements:

1. Minimum permissible investment is for 1 gram of gold.

2. The Bond is issued by SBI on behalf of Government of India.

Which of the statements given above is/are correct?

(a) 1 only

(b) 2 only

(c) Both 1 and 2

(d) Neither 1 nor 2

Ans: a

Exp: SGBs are government securities denominated in grams of gold. They are substitutes for holding

physical gold. Investors have to pay the issue price in cash and the bonds will be redeemed in cash

on maturity. The Bond is issued by Reserve Bank on behalf of Government of India. Minimum

permissible investment is for 1 gram of gold.

https://pib.gov.in/PressReleseDetail.aspx?PRID=1575483

Q 15. Consider the following statements:

1. A debenture is a loan issued by a firm.

2. It usually involves a fixed repayment schedule, in terms of both time and interest.

3. The person who owns the debenture will never have any claim on the company and has no voting

rights.

Select the correct answer using the codes given below:

(a) 1 only

(b) 2 and 3 only

(c) 1 and 2 only

(d) 1, 2 and 3

Ans: c

Exp: A debenture is a loan issued by a firm. It usually involves a fixed repayment schedule, in terms

of both time and interest. If the firm keeps to the terms of the debenture, the person who owns the

debenture has no claim on the company and no voting rights. However, if the firm were to default on

their payment of a debenture. The holders of the debentures would have rights over the company.

They would have access to assets before even ordinary shareholders.

A convertible debenture occurs when at the end of a certain time frame a debenture can be converted

into shares.

https://pib.gov.in/PressReleseDetail.aspx?PRID=1582337

Q16. Consider the following:

1. Revenue bonds where the funds raised are earmarked for one project are termed Municipal bonds.

2. Municipal bonds have been in existence in India since 1951.

Which of the statements given above is/are correct?

(a) 1 only

(b) 2 only

(c) Both 1 and 2

(d) Neither 1 nor 2

Ans: d

https://t.me/UPSC_PDF Website ➡ https://upscpdf.com https://t.me/UPSC_PDF

Website ➡ https://upscpdf.com

Current Economics MCQs (Jan

2019 to Feb 20) + Eco Survey 2020

WriteToBeIAS.com Prelims 2020 Online Classes

by CA Rahul Kumar

Telegram https://t.me/writetobeias1 Youtube https://tinyurl.com/yx22y2yl Page 47

Exp: Municipal bonds where the funds raised are earmarked for one project are termed revenue

bonds. Municipal bonds have been in existence in India since 1997.

http://yojana.gov.in/FULLPDFYOJANAAUGUST2019.pdf

Q17. Consider the following statements:

1. Sovereign Gold Bond (SGB) are government securities denominated in grams of gold.

2. The Bond is issued by State Bank of India on behalf of Government of India.

Which of the above statements is/are correct?

(a) 1only

(b) 2only

(c) Both1and2

(d) Neither1nor2

Ans: a

Exp: SGBs are government securities denominated in grams of gold. They are substitutes for holding

physical gold. Investors have to pay the issue price in cash and the bonds will be redeemed in cash

on maturity. The Bond is issued by Reserve Bank on behalf of Government of India.

https://pib.gov.in/PressReleseDetail.aspx?PRID=1581208

Q18. Consider the following:

1. CPSEETF was launched by the government in March 2014 to help divest its stake in select public

sector undertakings through the ETF route.

2. The CPSEETF is managed by Reliance Mutual Fund.

Which of the statements given above is/are correct?

(a) 1only

(b) 2only

(c) Both1and 2

(d) Neither1nor2

Ans: c

Exp: CPSEETF, as the name suggests, is an exchange-traded fund (ETF) comprising public sector

enterprises (PSEs). The ETF was launched by the government in March 2014 to help divest its stake

in select public sector undertakings through the ETF route. The ETF is based on the Nifty CPSE index

that comprises 11 PSEs such as ONGC, NTPC, Coal India, Indian Oil Corporation, REC, Power

Finance Corporation, Bharat Electronics, Oil India, NBCC (India), NLC India and SJVN. The

parameters based on which companies have been made part of the index include a criteria that they

have paid at least 10% dividend in the last two consecutive years. The CPSEETF is managed by

Reliance Mutual Fund.

https://www.thehindu.com/business/Economy/cpse-etf-stakes-that-pay-rich-

dividends/article28978061.ece

Q19. Consider the following statements:

1. Serious Fraud Investigation Office (SFIO) has been established under the Ministry of Commerce.

2. The Central Govt. May assign the investigation into the affairs of a company to the Serious Fraud

Investigation Office in public interest.

Which of the above statements is/are correct?

(a) 1 only

(b) 2 only

(c) Both 1 and 2

(d) Neither 1 nor 2

Ans: b

https://t.me/UPSC_PDF Website ➡ https://upscpdf.com https://t.me/UPSC_PDF

Website ➡ https://upscpdf.com

Current Economics MCQs (Jan

2019 to Feb 20) + Eco Survey 2020

WriteToBeIAS.com Prelims 2020 Online Classes

by CA Rahul Kumar

Telegram https://t.me/writetobeias1 Youtube https://tinyurl.com/yx22y2yl Page 48

Exp: Serious Fraud Investigation Office (SFIO) has been established under the Ministry of

Corporate Affairs.

(i) As per Section 212(1) of the Companies Act, 2013, the Central Govt. May assign the

investigation into the affairs of a company to the Serious Fraud Investigation Office–

1-on receipt of report of the Registrar or Inspector under section 208;

2- on intimation of a special resolution passed by a company requesting an investigation into

its affairs;

3- in public interest;

4- on the request of any Department of Central Government or State Government

https://www.thehindu.com/news/cities/Delhi/hc-seeks-reply-from-ex-auditor-of-daewoo-motors-on-

sfio-petition/article29128968.ece

Q20. “Bharat-22” recently seen in news is ?

(a) Biodiesel

(b) Exchange Traded Fund

(c) Solar plane

(d) Gold Bond

Ans: b

Exp: The government has so far raised around ₹35,900 crore via through the Bharat-22 ETF-

Exchange Traded Fund.

https://www.thehindu.com/business/Industry/bharat-22-etf-fourth-tranche-on-october-

3/article29533619.ece

PUBLIC FINANCE

Q1. With reference to the Fifteenth Finance Commission, consider the following

1. The Fifteenth Finance Commission was constituted on 27 November 2017.

2. Its ‘Terms of Reference’ include recommending monitorable performance criteria for important

national flagship programmes and examining the possibility of setting up a permanent non lapsable

funding for India’s defence needs.

Which of the statements given above is/are correct?

Select the correct answer using the code given below:

(a) 2 only

(b) Both 1 and 2

(c) 1 only

(d) Neither 1 nor 2

Ans. b

Source: https://fincomindia.nic.in/

Q2.Consider the following statements:

1. National Pension Scheme for Traders, Shopkeepers and Self-Employed Persons, which is a

voluntary and contributory pension scheme launched in 2019.

2. The traders in the age group of 18-50 years with an annual turnover, not exceeding Rs.1.5 crore

and who are not members of EPFO/ESIC/NPS/PM-SYM or an income tax payer can join the scheme.

Which of the statements given above is/are correct?

(a) 1 only

(b) 2 only

(c) Both 1 and 2

https://t.me/UPSC_PDF Website ➡ https://upscpdf.com https://t.me/UPSC_PDF

Website ➡ https://upscpdf.com

Current Economics MCQs (Jan

2019 to Feb 20) + Eco Survey 2020

WriteToBeIAS.com Prelims 2020 Online Classes

by CA Rahul Kumar

Telegram https://t.me/writetobeias1 Youtube https://tinyurl.com/yx22y2yl Page 49

(d) Neither 1 nor 2

Ans: a

Exp: The traders in the age group of 18-40 years with an annual turnover, not exceeding Rs.1.5 crore

and who are not members of EPFO/ESIC/NPS/PM-SYM or an income tax payer, can join the scheme.

Under the scheme, 50% monthly contribution is payable by the beneficiary and equal matching

contribution is paid by the Central Government. Subscribers, after attaining the age of 60 years, are

eligible for a monthly minimum assured pension of Rs. 3,000.

https://t.me/UPSC_PDF Website ➡ https://upscpdf.com https://t.me/UPSC_PDF

Website ➡ https://upscpdf.com

Current Economics MCQs (Jan

2019 to Feb 20) + Eco Survey 2020

WriteToBeIAS.com Prelims 2020 Online Classes

by CA Rahul Kumar

Telegram https://t.me/writetobeias1 Youtube https://tinyurl.com/yx22y2yl Page 50

Q3. With Reference to the Fifteenth Finance Commission, which of the following statements is/are

correct?

1. The Fifteenth Finance Commission was constituted on 27 November 2015.

2. Terms of reference includes recommending monitorable performance criteria for important

national flagship programmes.

3. Terms of reference includes examining the possibility of setting up a permanent non lapsable

funding for India’s defence needs.

(a) 1 and 2 only

(b) 2 only

(c) 3 only

(d) 2 and 3 only

Ans: d

Exp: The Fifteenth Finance Commission was constituted on 27 November 2017 against the backdrop

of the abolition of Planning Commission (as also of the distinction between Plan and non-Plan

expenditure) and the introduction of the goods and services tax (GST), which has fundamentally

redefined federal fiscal relations. The Terms of Reference of the current Commission have some

distinctive features, including recommending monitorable performance criteria for important national

flagship programmes and examining the possibility of setting up a permanent non lapsable funding

for India’s defence needs. The reorganisation of the State of Jammu and Kashmir into two Union

Territories – one of Jammu and Kashmir and one of Ladakh – presents a new dynamic. On the whole

the Finance Commission faces new challenges in the process of the evolution of our federal polity.

As an important Constitutional entity, the Commission is committed to balancing competing claims

and priorities among all three tiers of government in a credible manner.

https://pib.gov.in/PressReleseDetail.aspx?PRID=1573388

Q4. Consider the following:

1. Gender Responsive Budgeting (GRB) was adopted by India in 2009-10.

2. Overall, the Gender Budget Allocation for 2019-20 is close to 10 percent as a proportion of total

expenditure.

Which of the statements given above is/are correct?

(a) 1 only

(b) 2 only

(c) Both1and 2

(d) Neither1nor 2

Ans: d

Exp: Gender Responsive Budgeting (GRB) was adoption by India in 2005-06. Overall, the Gender

Budget Allocation for 2019-20 is close to 5 percent as a proportion of total expenditure.

http://yojana.gov.in/FULLPDFYOJANAAUGUST2019.pdf

Q5. Consider the following statements about a scheme launched by the Government of India: It was

launched to provide social security during old age and to protect elderly persons aged 60 years and

above against a future fall in their interest income due to uncertain market conditions. The scheme

enables old age income security for senior citizens through provision of assured pension / return

linked to the subscription amount based on government guarantee to Life Insurance Corporation of

India (LIC).Identify the scheme.

(a) Pradhan Mantri Swasthya Suraksha Yojana

(b) Pradhan Mantri Vaya Vandana Yojana

(c) Liveability Index Programme

(d) Rashtriya Vayoshri Yojana

https://t.me/UPSC_PDF Website ➡ https://upscpdf.com https://t.me/UPSC_PDF

Website ➡ https://upscpdf.com

Current Economics MCQs (Jan

2019 to Feb 20) + Eco Survey 2020

WriteToBeIAS.com Prelims 2020 Online Classes

by CA Rahul Kumar

Telegram https://t.me/writetobeias1 Youtube https://tinyurl.com/yx22y2yl Page 51

Ans: b

Exp: The Pradhan Mantri Vaya Vandana Yojana (PNVVY) was launched in May 2017 to provide

social security during old age. This is a simplified version of the VPBY and will be implemented

by the Life Insurance Corporation (LIC) of India. Under the scheme, on payment of an initial

lump sum amount ranging from Rs 1,50,000 for a minimum pension of Rs 1000 per month to a

maximum of Rs 7,50,000/- for a maximum pension of Rs 5,000 per month, subscribers will get an

assured pension based on a guaranteed rate of return of 8% per annum payable

monthly/quarterly/half- yearly/annually. The duration of the scheme will be for a period of ten years

and the scheme is opened for subscription for a period of one year i.e. from 4th May, 2017 to 3rd

May, 2018.

http://yojana.gov.in/19-3-19%20%20March%20Yojana%20Final.pdf

Q6. Which of the following indicates most approximate figure for Budget 2019-20 of Railways:-

(a) 55,000 Crore

(b) 65,000 Crore

(c) 75,000 Crore

(d) 85,000 Crore

Ans: b

Exp: In the Interim Budget 2019-20, the Railways has been allocated Rs.64,587 crore. The Railways’

overall capital expenditure programme is of Rs.1,58,658 crore. This was stated by the Union Minister

for Finance, Corporate Affairs, Railways & Coal, Shri Piyush Goyal, while presenting the Interim

Budget 2019-20 in Parliament today.

http://www.pib.nic.in/PressReleseDetail.aspx?PRID=1562148

Q7. ‘Non-Functional Upgradation (NFU)’ is related to

(a) Aircraft upgradation

(b) Seventh Central Pay commission

(c) Submarine upgradation

(d) Skills Training

Ans: b

Exp: The Armed Forces have been demanding the grant of NFU based on recommendations of the

Central Pay Commission like the Grade A Services and even the Central Armed Police Forces now

to have parity in pay scales.

http://pib.nic.in/PressReleseDetail.aspx?PRID=1569457

Q8. Which of the following indicates most approximate figure for Budget 2019-20 of Defence:-

(a) 3,00,000 Crore

(b) 4,00,000 Crore

(c) 5,00,000 Crore

(d) 6,00,000 Crore

Ans: a

Exp: Rs. 3,05,296 crore have been provided in the Budget Estimates for 2019-20, compared to Rs.

2,82,733 crore provided in 2018-19 Budget Estimates. The figures were revised to Rs. 2,85,423 crore

in the Revised Estimates of 2018-19. While presenting the Interim Budget 2019-20 in Parliament

today the Union Minister for Finance, Corporate Affairs, Railways and Coal Sh. Piyush Goyal said

“Our Defence Budget will be crossing Rs.3,00,000 crore for the first time in 2019-20. For securing

our borders and to maintain preparedness of the highest order, if necessary, additional funds would

be provided.”

http://www.pib.nic.in/PressReleseDetail.aspx?PRID=1562149

https://t.me/UPSC_PDF Website ➡ https://upscpdf.com https://t.me/UPSC_PDF

Website ➡ https://upscpdf.com

Current Economics MCQs (Jan

2019 to Feb 20) + Eco Survey 2020

WriteToBeIAS.com Prelims 2020 Online Classes

by CA Rahul Kumar

Telegram https://t.me/writetobeias1 Youtube https://tinyurl.com/yx22y2yl Page 52

TAXATION

Q1. With reference to Advance Pricing Agreement, which of the following statements is/are correct?

1. It is an ahead-of-time agreement between a taxpayer and a tax authority on an appropriate transfer

pricing methodology (TPM) for a set of transactions at issue over a fixed period of time

2. Taxpayers may enter into APAs with more than one tax authority – i.e., bilateral or multilateral

APAs

Select the correct answer using the code given below:

(a) 1 only

(b) 2 only

(c) Both 1 and 2

(d) Neither 1nor 2

Ans: c

Exp:

Advance Pricing Agreement- In advance pricing agreement (APA) is an ahead-of-time agreement

between a taxpayer and a tax authority on an appropriate transfer pricing methodology (TPM) for a

set of transactions at issue over a fixed period of time

Taxpayers may enter into APAs with more than one tax authority – i.e., bilateral or multilateral APAs

http://pib.nic.in/newsite/PrintRelease.aspx?relid=189634

https://www.ey.com/gl/en/services/tax/international-tax/guide-to-advance-pricing-agreements--apa-

---apas--the-basic-elements

Q2. ‘Country-by-Country (CbC) Report’ recently seen in news is related to

(a) International Terrorism

(b) Sea Piracy

(c) Tax administrations

(d) None of the above

Ans: c

Exp: The BEPS Action 13 report (Transfer Pricing Documentation and Country-by-Country

Reporting) provides a template for multinational enterprises (MNEs) to report annually and for

each tax jurisdiction in which they do business the information set out therein. This report is called

the Country-by-Country (CbC) Report.

http://pib.nic.in/PressReleseDetail.aspx?PRID=1568894

Q3. Consider following statements:

1. The accounting of GST collections in the Central Government is done on the cash basis.

2. A constitutional amendment to fix the proportion of shareable taxes going to states has been

enacted.

Which of the statements given above is/are correct?

(a) 1 only

(b) 2 only

(c) Both 1 and 2

(d) Neither 1 nor 2

Ans: a

Exp: The Centre should bring in a constitutional amendment to fix the proportion of shareable taxes

going to states, said former chairman of the Prime Minister’s Economic Advisory Council C.

Rangarajan. The accounting of GST collections in the Central Government is done on the cash basis.

https://www.thehindu.com/business/bring-in-constitutional-amendment-on-gst-revenue-sharing-

proportion-rangarajan/article26475474.ece

https://t.me/UPSC_PDF Website ➡ https://upscpdf.com https://t.me/UPSC_PDF

Website ➡ https://upscpdf.com

Current Economics MCQs (Jan

2019 to Feb 20) + Eco Survey 2020

WriteToBeIAS.com Prelims 2020 Online Classes

by CA Rahul Kumar

Telegram https://t.me/writetobeias1 Youtube https://tinyurl.com/yx22y2yl Page 53

Q4. Consider following statements:

1. The TIR Convention facilitates the international carriage of goods from one or more customs

offices of departure to one or more customs offices of destination.

2. The vehicle remains sealed throughout the TIR transport and goods are not inspected at border

crossings.

3. India gets its second TIR shipment via Chabahar Port from Afghanistan

Which of the statements given above is/are correct?

(a) 1 and 2 only

(b) 2 and 3 only

(c) 1 and 3 only

(d) 1, 2 and 3

Ans: a

Exp: The TIR Convention facilitates the international carriage of goods from one or more customs

offices of departure to one or more customs offices of destination. The vehicle remains sealed

throughout the TIR transport and goods are not inspected at border crossings. India gets first TIR

shipment via Chabahar Port from Afghanistan.

https://www.thehindu.com/business/india-gets-first-tir-shipment-via-chabahar-port-from-

afghanistan/article26525062.ece

Q5. Consider following statements-

1. Goods & Services Tax Council is a constitutional body for making recommendations to the Union

and State Government on issues related to Goods and Service Tax.

2. The GST Council is chaired by the Prime Minister

Which of the statements given above is/are correct?

(a) 1 only

(b) 2 only

(c) Both 1 and 2

(d) Neither 1 nor 2

Ans: a

Exp: Goods & Services Tax Council is a constitutional body for making recommendations to the

Union and State Government on issues related to Goods and Service Tax. The GST Council is chaired

by the Union Finance Minister and other members are the Union State Minister of Revenue or

Finance and Ministers in-charge of Finance or Taxation of all the States.

https://www.thehindu.com/business/gst-council-to-discuss-realty-rates/article26314481.ece

Q6. ‘Angel Tax’, sometimes seen in news is

(a) Income tax payable on capital raised by unlisted companies via issue of shares where the share

price is seen in excess of the fair market value of the shares sold.

(b) Income tax rebate to Startups

(c) Income tax on companies with FDI investments above 25 percent

(d) None of the above

Ans: a

Exp: Angel tax is a term used to refer to the income tax payable on capital raised by unlisted

companies via issue of shares where the share price is seen in excess of the fair market value of the

shares sold.

https://www.thehindu.com/business/Industry/startups-to-be-listed-for-angel-tax-

exemption/article26218956.ece

https://t.me/UPSC_PDF Website ➡ https://upscpdf.com https://t.me/UPSC_PDF

Website ➡ https://upscpdf.com

Current Economics MCQs (Jan

2019 to Feb 20) + Eco Survey 2020

WriteToBeIAS.com Prelims 2020 Online Classes

by CA Rahul Kumar

Telegram https://t.me/writetobeias1 Youtube https://tinyurl.com/yx22y2yl Page 54

Q7. Country-by-Country (CbC) report sometimes seen in news is a key step in making India

compliant with the Base Erosion and Profit Shifting (BEPS) project.

Which one of the following statements best describes the term?

Select one:

(a) It is the annual publication of OECD on tax evasion achieved by multinational enterprises by

shifting the profits

(b) A report required to be published by multinational enterprises annually and for each tax

jurisdiction in which they do business

(c) A report that contains the revised standards of transfer pricing documentation

(d) It is the guidelines by OECD for developing countries for securing revenues by realigning taxation

with economic activities and value creation

Ans: a

Exp:

o It is a report required to be published by multinational enterprises (MNEs) annually and for

each tax jurisdiction in which they do business.

o The template for the same was provided by the BEPS Action 13 report.

o The Income Tax Act requires Indian subsidiaries of multinational companiesto provide details

of key financial statements from other jurisdictions where they operate.

o This helps the IT department to better monitor such companies, and track its revenue and tax

compliance or evasion.

https://www.thehindu.com/business/india-in-pact-to-ease-us-firms-compliance/article26656510.ece

Q8. Consider the following statements:

1. Angel Tax is the income tax payable on capital raised by listed companies.

2. Investments of up to 25 crore in an eligible company will be exempt from the angel tax.

Which of the statements given above is/are correct?

(a) 1 only

(b) 2 only

(c) Both 1 and 2

(d) Neither 1 nor 2

Ans: b

Exp:

o Angel Tax is a term used to refer to the income tax payable on capital raised by unlisted

companies via issue of shares where the share price is seen in excess of the fair market value

of the shares sold.

o Investments of up to `25 crore (earlier `10 crore) in an eligible company will be exempt

from the angel tax and any scrutiny to do with its applicability.

o Investments made by a listed company of a net worth of at least `100 croreor a turnover of at

least Rs. 250 crore would also be exempt.

https://www.thehindu.com/business/Industry/startups-to-be-listed-for-angel-tax-

exemption/article26218956.ece

Q9. WRONG QUESTION – SO DELETED IT

Q10. Consider the following statements: -

1. The annual turnover limit under which companies would be exempt from GST has been increased

to Rs. 40 lakh for the North Eastern and hill states, from the earlier limit of Rs. 20

https://t.me/UPSC_PDF Website ➡ https://upscpdf.com https://t.me/UPSC_PDF

Website ➡ https://upscpdf.com

Current Economics MCQs (Jan

2019 to Feb 20) + Eco Survey 2020

WriteToBeIAS.com Prelims 2020 Online Classes

by CA Rahul Kumar

Telegram https://t.me/writetobeias1 Youtube https://tinyurl.com/yx22y2yl Page 55

lakh.

2. The threshold limit for Composition Scheme to small service providers has been increased from

an annual turnover of up to Rs. 20 Lakh to an annual turnover of up to Rs. 40 Lakh.

Which of the statements given above is/are correct?

Select the correct answer using the code given below:

(a) 1 only

(b) 2 only

(c) Both 1 and 2

(d) Neither 1 nor 2

Ans: d

Exp: The Council also raised the annual turnover limit under which companies would be exempt

from GST to Rs. 40 lakh for most States and Rs. 20 lakh for the North Eastern and hill states,

from the earlier limit of Rs. 20 lakh and Rs. 10 lakh, respectively. Composition Scheme to small

service providers with an annual turnover of up to Rs. 50 lakh, at a tax rate of 6%.

https://www.thehindu.com/todays-paper/tp-business/gst-burden-on-small-businesses-

eased/article25964627.ece

Q11. Consider the following statements: -

1. Income Tax Act is amended only during the main Budget.

2. Finance Bill and Appropriation Bill both are Money Bills.

Which of the statements given above is/are correct?

Select the correct answer using the code given below:

(a) 1 only

(b) 2 only

(c) Both 1 and 2

(d) Neither 1 nor 2

Ans: c

Exp: as a matter of convention, the Income Tax Act is amended only during the main Budget.

https://www.thehindu.com/todays-paper/tp-business/views-divided-on-vote-on-account-option-for-

central-government/article25975496.ece

Q12. The annual GST return form for normal taxpayers/ businesses registered under the Goods and

Services Tax (GST) regime is:

(a) GSTR-9

(b) GSTR-9A

(c) GSTR-9B

(d) GSTR-9C

Ans: a

Exp: The Central Board of Indirect Taxes and Customs (CBIC) on December 31, 2018, notified form

GSTR-9, GSTR-9A and GSTR-9C. GSTR-9 is the annual return form for normal taxpayers,

GSTR-9A is for taxpayers under composition scheme, while GSTR-9C is a reconciliation statement.

https://www.thehindu.com/todays-paper/tp-business/govt-notifies-new-gst-return-

forms/article25885554.ece

Q13.Consider the following statements:

1. e-way bill is a document required to be carried by a person in charge of the conveyance carrying

any consignment of goods of value exceeding fifty thousand rupees

2. Only registered person can enrol and generate the e-way bill for movement of goods for his/ her

own use

https://t.me/UPSC_PDF Website ➡ https://upscpdf.com https://t.me/UPSC_PDF

Website ➡ https://upscpdf.com

Current Economics MCQs (Jan

2019 to Feb 20) + Eco Survey 2020

WriteToBeIAS.com Prelims 2020 Online Classes

by CA Rahul Kumar

Telegram https://t.me/writetobeias1 Youtube https://tinyurl.com/yx22y2yl Page 56

Which of the above statements is/are correct?

(a) 1 only

(b) 2 only

(c) Both 1 and 2

(d) Neither 1 nor 2

Ans: a

Exp: The consignor or consignee, as a registered person or a transporter of the goods can generate

the e-way bill. The unregistered transporter can enrol on the common portal and generate the e-way

bill for movement of goods for his clients. Any person can also enrol and generate thee-way bill for

movement of goods for his/ her own use. e-way bill is a document required to be carried by a person

in charge of the conveyance carrying any consignment of goods of value exceeding fifty thousand

rupees as mandated by the Government in terms of Section 68 of the Goods and Services Tax Act

read with Rule 138 of the rules framed there under. It is generated from the GST Common Portal for

e-Way bill system by the registered persons or transporters who cause movement of goods of

consignment before commencement of such movement.

https://pib.gov.in/PressReleseDetail.aspx?PRID=1587688

Q14. The term ‘Base Erosion and Profit Shifting’ is sometimes seen in the news in the context of

(a) mining operation by multinational companies in resource-rich but backward areas

(b) curbing of the tax evasion by multinational companies

(c) exploitation of genetic resources of a country by multinational companies

(d) lack of consideration of environmental costs in the planning and implementation of developmental

projects

Ans: b

Exp: Domestic tax base erosion and profit shifting (BEPS) due to multinational enterprises exploiting

gaps and mismatches between different countries' tax systems affects all countries. Developing

countries' higher reliance on corporate income tax means they suffer from BEPS disproportionately.

https://www.thehindu.com/opinion/a-tax-policy-that-could-work/article29675502.ece

Q16.Goods and Service Tax Network (GSTN) is

(a) Public sector entity

(b) Private sector Entity

(c) Non-profit organization

(d) None of the above

Ans: a

Exp: ICICI Bank has exited the company following the government’s decision to make GST Network

into a public sector entity last year. The Centre will own 50 % stake in the GST Network and the rest

will be held by States on a pro-rata basis, in the new structure

https://www.thehindu.com/business/icici-bank-exits-gst-network/article29801409.ece

Q17.The GST Council consists of the:

1. Union Finance Minister

2. Union Minister of State (Finance)

3. The Minister in charge of Finance or Taxation or any other Minister, nominated by each state

government.

4. RBI governor

Select the correct answer using the code given below.

https://t.me/UPSC_PDF Website ➡ https://upscpdf.com https://t.me/UPSC_PDF

Website ➡ https://upscpdf.com

Current Economics MCQs (Jan

2019 to Feb 20) + Eco Survey 2020

WriteToBeIAS.com Prelims 2020 Online Classes

by CA Rahul Kumar

Telegram https://t.me/writetobeias1 Youtube https://tinyurl.com/yx22y2yl Page 57

(a) 1 and 2 only

(b) 1, 2 and 3 only

(c) 2, 3and 4 only

(d) 1, 2, 3 and 4

Ans: b

Exp: The GST Council consists of the following members: The Union Finance Minister (as

Chairman). The Union Minister of State in-charge of Revenue or Finance. The Minister in charge of

Finance or Taxation or any other Minister, nominated by each State Government to the GST Council.

https://www.thehindu.com/news/national/kerala/showdown-likely-at-gst-council-

meet/article30330794.ece

Q18.With reference to FASTag, consider the following:

1. It is issued separately for each vehicle, regardless of ownership.

2. It is valid for a period of four years.

3. NPCI does not issue FASTag.

Which of the above statements are correct?

(a) 2 and 3

(b) 1 and 2

(c) 1 and 3

(d) 3 only

Ans: c

Exp: It is valid for a period of five years, after which you can apply for re-issuance. NPCI does not

issue FASTag. NPCI has developed NETC System which facilities to make the toll payment

electronically.

Source: https://www.business-standard.com/about/what-is-fastag

https://www.npci.org.in/netc-consumer

Q19.With reference HSN Code, consider the following statements:

1. It is a five-digit identification code developed by the World Customs Organization (WCO)

2. The Ministry of Commerce and Industry recently allocated a separate Harmonised System (HS)

code for Khadi.

Which of the statements given above is/are correct?

(a) 1only

(b) 2only

(c) Both 1 and 2

(d) Neither 1 nor 2

Ans: b

Exp: Harmonised System, or simply ‘HS’ is a six-digit identification code developed by the World

Customs Organization (WCO). It is called the “universal economic language” for goods. It is a

multipurpose international product nomenclature. The system currently comprises of around 5,000

commodity groups → HS Code is also known as HSN Code in India. Goods are classified into

Harmonized System of Nomenclature or HSN. It is widely used for taxation purposes by helping to

identify the rate of tax applicable to aspecific product in a country that is under review. It canals obe

used in calculations that involve claiming benefits. It is used to monitor and control the import and

export of commodities.

Source:https://indianexpress.com/article/explained/explained-khadi-gets-hs-code-but-what-does-

that-mean-6113268/

Q20.Consider the following statements:

https://t.me/UPSC_PDF Website ➡ https://upscpdf.com https://t.me/UPSC_PDF

Website ➡ https://upscpdf.com

Current Economics MCQs (Jan

2019 to Feb 20) + Eco Survey 2020

WriteToBeIAS.com Prelims 2020 Online Classes

by CA Rahul Kumar

Telegram https://t.me/writetobeias1 Youtube https://tinyurl.com/yx22y2yl Page 58

1. GST is proposed to be a dual levy where the Central Government will levy and collect Central

GST and the State will levy and collect State GST on intra-state supply of goods or services.

2. GST Council dictates tax rate, tax exemption and tax laws, keeping in mind special rates and

provisions for some states.

3. The Centre does not levy and collect Integrated GST (IGST) on inter-state supply of goods or

services.

Which of the statements given above is/are correct?

(a) 2 and 3

(b) 1 and 2

(c) 1 and 3

(d) 1, 2 and 3

Ans: b

Exp: Goods and Services Tax (GST) is governed by the GST Council. Article 279 (1) of the amended

Indian Constitution states that the GST Council has to be constituted by the President within 60 days

of the commencement of the Article 279A. According to the article, GST Council will be a joint

forum for the Centre and the States. It consists of the following members: Union Finance Minister

will be the Chairperson; as a member, the Union Minister of State will be in charge of Revenue of

Finance, the minister in charge of finance or taxation or any other minister nominated by each

State government, as members. The Centre also levies and collects Integrated GST (IGST) on inter-

state supply of goods or services.

Source:https://indianexpress.com/article/explained/explained-voting-at-the-gst-council-6206947/

https://cleartax.in/s/gst-council

Q21. ICEDASH and ATITHI are two new IT initiatives related to:

(a) Ministry of Commerce and Industry

(b) Ministry of Finance

(c) Ministry of Corporate Affairs

(d) Ministry of External Affairs

Ans: b

Exp: Two new IT Initiatives-ICEDASH & ATITHI were launched by the Ministry of Finance for

improved monitoring and pace of Customs clearance of imported goods and facilitating arriving

international passengers. ICEDASH: Ease of Doing Business monitoring dashboard of the Indian

Customs helping publicsee the daily Customs clearance times of import cargo. ATITHI: Easy to use

mobile app for international travellers to file the Customs declaration in advance.

Source:https://pib.gov.in/newsite/PrintRelease.aspx?relid=194236

Q22. “GAFA tax" recently seen in news is related to

(a) Tax on large internet and technology companies

(b) Tax on food products with trans fats

(c) Tax on carbon emissions

(d) Tax on clearing forests by firms

Ans. a

Exp: France will introduce its own tax on large internet and technology companies from January 1.

Source:https://www.livemint.com/Politics/Zag39IyZPanr0mNFR0VFbL/France-to-introduce-tax-

on-global-internet-technology-firms.html

Q23. Consider following statements:

https://t.me/UPSC_PDF Website ➡ https://upscpdf.com https://t.me/UPSC_PDF

Website ➡ https://upscpdf.com

Current Economics MCQs (Jan

2019 to Feb 20) + Eco Survey 2020

WriteToBeIAS.com Prelims 2020 Online Classes

by CA Rahul Kumar

Telegram https://t.me/writetobeias1 Youtube https://tinyurl.com/yx22y2yl Page 59

1. The Central Board of Indirect Taxes and Customs (CBIC) determines the Rate of Exchange of

conversion of the Foreign Currencies specified into Indian currency or vice versa, relating to imported

and export goods.

2. Central Board of Indirect Taxes and Customs is a part of the Department of Expenditure under

the Ministry of Finance, Government of India.

Which of the statements given above is/are correct?

(a) 1 only

(b) 2 only

(c) Both 1 and 2

(d) Neither 1 nor 2

Ans: a

Exp: Central Board of Indirect Taxes and Customs (erstwhile Central Board of Excise & Customs)

is a part of the Department of Revenue under the Ministry of Finance, Government of India. It deals

with the tasks of formulation of policy concerning levy and collection of Customs, Central Excise

duties, Central Goods & Services Tax and IGST, prevention of smuggling and administration of

matters relating to Customs, Central Excise, Central Goods & Services Tax, IGST and Narcotics to

the extent under CBIC's purview. The Board is the administrative authority for its subordinate

organizations, including Custom Houses, Central Excise and Central GST Commissionerate’s and

the Central Revenues Control Laboratory.

In exercise of the powers conferred by Section 14 of the Customs Act, 1962 (52 of 1962), and in

supersession of the notification of the Central Board of Indirect Taxes and Customs (CBIC)

No.37/2019-CUSTOMS (N.T.), dated 16th May, 2019 except as respects things done or omitted to

be done before such super-session, the Central Board of Indirect Taxes and Customs (CBIC) hereby

determines that the Rate of Exchange of conversion of each of the Foreign Currencies specified in

column (2) of each of Schedule I and Schedule II annexed hereto, into Indian currency or vice versa,

shall, with effect from 7th June, 2019, be the rate mentioned against it in the corresponding entry in

column (3) thereof, for the purpose of the said Section, relating to imported and export goods.

https://pib.gov.in/PressReleseDetail.aspx?PRID=1573583

Q24. Consider the following:

1. The National Anti-profiteering Authority (NAA) is the statutory mechanism under GST law to

check the unfair profiteering activities by the registered suppliers under GST law.

2. Any consumer or organisation experiencing the non-reduction in the price of the goods or service

despite reduction in the rate of tax can file the complaint with proper prima facie evidences.

3. Only Online complaint facility is available.

Which of the statements given above is/are correct?

(a) 1 only

(b) 1 and 2 only

(c) 1 and 3 only

(d) All of the above

Ans: b

Exp: The National Anti-profiteering Authority (NAA) is the statutory mechanism under GST law to

check the unfair profiteering activities by the registered suppliers under GST law. Any consumer or

organisation experiencing the non-reduction in the price of the goods or service despite reduction in

the rate of tax can file the complaint with proper prima facie evidences. There are multiple ways

through which aggrieved consumers or recipients of goods and services can register their complaints

against profiteering.

https://pib.gov.in/PressReleseDetail.aspx?PRID=1575285

https://t.me/UPSC_PDF Website ➡ https://upscpdf.com https://t.me/UPSC_PDF

Website ➡ https://upscpdf.com

Current Economics MCQs (Jan

2019 to Feb 20) + Eco Survey 2020

WriteToBeIAS.com Prelims 2020 Online Classes

by CA Rahul Kumar

Telegram https://t.me/writetobeias1 Youtube https://tinyurl.com/yx22y2yl Page 60

Q25. The term ‘Base Erosion and Profit Shifting’ is sometimes seen in the news in the context of

(a) Mining operation by multinational companies in resource-rich but backward areas

(b) Curbing of the tax evasion by multinational companies

(c) Exploitation of genetic resources of a country by multinational companies

(d) Lack of consideration of environmental costs in the planning and implementation of

developmental projects

Ans: b

Exp: The Union Cabinet, has approved the ratification of the Multilateral Convention to Implement

Tax Treaty Related Measures to Prevent Base Erosion and Profit Shifting (MLI)

Impact: The Convention will modify India's treaties in order to curb revenue loss through treaty abuse

and base erosion and profit shifting strategies by ensuring that profits are taxed where substantive

economic activities generating the profits are carried out and where value is created.

https://pib.gov.in/PressReleseDetail.aspx?PRID=1574095

Q26. Consider the following:

1. Unlike a tax, a cess is levied to meet a specific purpose; its proceeds cannot be spent on any kind

of government expenditure.

2. Recently flood cess was introduced by the state of Karnataka.

Which of the statements given above is/are correct?

(a) 1 only

(b) 2 only

(c) Both 1 and 2

(d) Neither 1 nor 2

Ans: a

Exp: The State government of Kerala has issued orders for introducing flood cess from June 1. Unlike

a tax, a cess is levied to meet a specific purpose; its proceeds cannot be spent on any kind of

government expenditure.

https://www.thehindu.com/opinion/op-ed/the-forgotten-funds/article28078252.ece

INFLATION

Q1. Which of the following brings out the ‘Consumer Price Index Number for the Industrial

Workers’?

a) The Reserve Bank of India

b) The Department of Economic Affairs

c) The Labour Bureau

d) The department of Personnel and Training

Ans: c

Exp: Labour Bureau under the Ministry of Labour and Employment is responsible for the collation

and publication of statistics and related information on wages, earnings, productivity, absenteeism,

labour turn-over, industrial relations etc.

https://pib.gov.in/PressReleseDetail.aspx?PRID=1573232

Q2. Sudden disruption in oil supply to India can cause

1. disruption in economic growth

2. increase Current Account Deficit

3.Increase inflation

(a) 1 and 2

(b) 1 and 3

https://t.me/UPSC_PDF Website ➡ https://upscpdf.com https://t.me/UPSC_PDF

Website ➡ https://upscpdf.com

Current Economics MCQs (Jan

2019 to Feb 20) + Eco Survey 2020

WriteToBeIAS.com Prelims 2020 Online Classes

by CA Rahul Kumar

Telegram https://t.me/writetobeias1 Youtube https://tinyurl.com/yx22y2yl Page 61

(c) 2 and 3

(d) all

Ans: d

Exp: Adverse impact on economic growth-due to an extended period of high oil prices. Saudi Arabia

is India second biggest oil supplier after Iraq. Widen the current account deficit: A$ 10 rise in Brent

will lift India’s annualised import bill by $ 15 billion and a 10 % rise in oil prices widens India’s

current account deficit by 0.4-0.5 percent of GDP. It further leads to weakening of rupee, which

ultimately squeeze the government’s ability to spend on social sector schemes or sops to revive the

economy. Inflation: Oil rise here would also affect many industries, including manufacturing and

aviation, and can accelerate inflation.

https://www.indiatoday.in/world/story/saudi-arabia-aramco-oil-facility-attacked-what-makes-drone-

attacks-so-dangerous-1600169-2019-09-17

Q3.Consider the following statements:-

1. Wholesale Price Inflation (WPI) number is a better measurement of what is largely happening with

consumer prices.

2. Reserve Bank of India uses WPI inflation to manage Monetary Policy in the country.

Which statement/statements given above is/are correct?

Select the correct answer using the codes given below.

(a) 1 only

(b) 2 only

(c) Both 1 and 2

(d) None

Ans: d

Exp: While earlier the Reserve Bank of India used WPI inflation to manage monetary policy

expectations, it is now the CPI inflation which is largely taken into account. Consumer PriceInflation

number in mind. It is a better measurement of what is largely happening with consumer prices.

https://www.thehindu.com/todays-paper/tp-opinion/inflation-conundrum/article26065472.ece

Q4. Through Asset liability management (ALM), which of the following central risks can be

managed:-

1. Interest Rate Risk

2. Liquidity Risk

3. Foreign currency risk

4. Currency risk

Select the correct answer using the codes given below.

(a) 1, 2 and 4 only

(b) 2 only

(c) 1, 3 and 4 only

(d) 1, 2, 3 and 4

Ans: d

Exp: All the risks mentioned are managed.

https://www.thehindu.com/todays-paper/tp-business/inflation-volatility-is-a-challenge-says-rbi-

governor/article26032628.ece

https://t.me/UPSC_PDF Website ➡ https://upscpdf.com https://t.me/UPSC_PDF

Website ➡ https://upscpdf.com

Current Economics MCQs (Jan

2019 to Feb 20) + Eco Survey 2020

WriteToBeIAS.com Prelims 2020 Online Classes

by CA Rahul Kumar

Telegram https://t.me/writetobeias1 Youtube https://tinyurl.com/yx22y2yl Page 62

https://t.me/UPSC_PDF Website ➡ https://upscpdf.com https://t.me/UPSC_PDF

Website ➡ https://upscpdf.com

Current Economics MCQs (Jan

2019 to Feb 20) + Eco Survey 2020

WriteToBeIAS.com Prelims 2020 Online Classes

by CA Rahul Kumar

Telegram https://t.me/writetobeias1 Youtube https://tinyurl.com/yx22y2yl Page 63

UNEMPLOYMENT AND POVERTY

Q1. Regarding “Pradhan Mantri Awas Yojana (Urban)” scheme, which of the following is not one of

its component?

(a) Slum rehabilitation of Slum Dwellers with participation of private developers using land as a

resource

(b) Promotion of Affordable Housing for weaker section through credit linked subsidy

(c) Affordable Housing in Partnership with Public sectors only

(d) Subsidy for beneficiary-led individual house construction /enhancement

Ans: c

Exp:

• The Pradhan Mantri Awas Yojana (Urban) Programme launched by the Ministry of Housing and

Urban Poverty Alleviation (MoHUPA), in Mission mode envisions provision of Housing for All by

2022, when the Nation completes 75 years of its Independence.

• The Mission seeks to address the housing requirement of urban poor including slum dwellers

through following programme verticals:

• Affordable Housing in Partnership with Public & Private sectors

• Slum rehabilitation of Slum Dwellers with participation of private developers using land as a

resource

• Promotion of Affordable Housing for weaker section through credit linked subsidy

• Subsidy for beneficiary-led individual house construction /enhancement

http://vikaspedia.in/health/sanitation-and-hygiene/social-welfare/urban-poverty-alleviation-

1/schemes-urban-poverty-alleviation/pradhan-mantri-awas-yojana-housing-for-all-urban

http://yojana.gov.in/Recent_archive_English_2018.asp

Q3. The Special Data Dissemination Standard (SDDS) were established by?

(a) IMF

(b) World Bank

(c) RBI

(d) WTO

Ans: a

Exp: Monthly measurement of the unemployment rate is one of the requirements of the Special Data

Dissemination Standard (SDDS) of the International Monetary Fund (IMF).

https://www.thehindu.com/opinion/lead/surveying-indias-

unemploymentnumbers/article26218615.ece

Q4. ‘SWEEKAR’ and ‘SWAGAT’ recently in news are

(a) Indigenously developed payment system

(b) New variety of apples

(c) High Yielding Variety of Wheat

(d) None of the above

Ans: a

Exp: India’s First Indigenously Developed Payment Eco-system for transport consisting of NCMC

Card, SWEEKAR (Swachalit Kiraya: Automatic Fare Collection System) and SWAGAT (Swachalit

Gate) is based on NCMC Standards.

http://www.pib.nic.in/PressReleseDetail.aspx?PRID=1567345

Q5. Consider following statements about Pradhan Mantri Shram Yogi Maandhan (PM-SYM):

1. It ensures old age protection for Unorganised Workers.

https://t.me/UPSC_PDF Website ➡ https://upscpdf.com https://t.me/UPSC_PDF

Website ➡ https://upscpdf.com

Current Economics MCQs (Jan

2019 to Feb 20) + Eco Survey 2020

WriteToBeIAS.com Prelims 2020 Online Classes

by CA Rahul Kumar

Telegram https://t.me/writetobeias1 Youtube https://tinyurl.com/yx22y2yl Page 64

2. During the receipt of pension, if the subscriber dies, the spouse of the beneficiary will not get the

pension received by the beneficiary as family pension.

Which of the statements given above is/are correct?

(a) 1 only

(b) 2 only

(c) Both 1 and 2

(d) Neither 1 nor 2

Ans: a

Exp: It ensures old age protection for Unorganised Workers. During the receipt of pension, if the

subscriber dies, the spouse of the beneficiary shall be entitled to receive 50% of the pension received

by the beneficiary as family pension.

http://www.pib.nic.in/PressReleseDetail.aspx?PRID=1567494

Q6.‘Hank Yarn Obligation’ is

(a) Subsidy to Silk Farmers

(b) Subsidy to Handloom Weavers

(c) Availability of coiled or wrapped yarn to handloom weavers

(d) None of the above

Ans: c

Exp: Now every producer of yarn who packs yarn for civil consumption will pack yarn in hank form

in each quarterly period commencing from January-March 2019 and every subsequent quarterly

period, in proportion of not less than 30% of total yarn packed by him during each quarterly period

for civil consumption. However, not less than 80% of the yarn required to be packed in hank form

shall be of counts 80s and below. Previously, this proportion was 40% of total yarn packed.

http://www.pib.nic.in/PressReleseDetail.aspx?PRID=1568058

Q7. Consider following statements related to Pradhan Mantri Shram Yogi Maan-dhan (PM-SYM)

Yojana:

1. It is Pension yojana for Unorganised Sector workers with monthly income of rupees 15000 or

below.

2. It will give assured monthly pension of rupees 5000 from the age of 60 years.

Which of the statements given above is/are correct?

(a) 1 only

(b) 2 only

(c) Both 1 and 2

(d) Neither 1 nor 2

Ans: a

Exp: It is Pension yojana for Unorganised Sector workers with monthly income of rupees 15000 or

below. It will give assured monthly pension of rupees 3000 from the age of 60 years.

http://yojana.gov.in/19-3-19%20%20March%20Yojana%20Final.pdf

Q8. The new concept of ‘learning poverty’ has been introduced by:

(a) UNDP

(b) UNICEF

(c) World Bank

(d) PISA

Ans: c

Exp: World Bank has released the report “Ending Learning Poverty”. Learning Poverty is defined as

the percentage of 10-year-olds who cannot read and understand a simple story.

https://t.me/UPSC_PDF Website ➡ https://upscpdf.com https://t.me/UPSC_PDF

Website ➡ https://upscpdf.com

Current Economics MCQs (Jan

2019 to Feb 20) + Eco Survey 2020

WriteToBeIAS.com Prelims 2020 Online Classes

by CA Rahul Kumar

Telegram https://t.me/writetobeias1 Youtube https://tinyurl.com/yx22y2yl Page 65

Source:https://timesofindia.indiatimes.com/blogs/toi-editorials/read-it-right-fix-learning-poverty-to-

tackle-actual-poverty/

Q9. The New Code on Wages will amalgamate which of the following acts:

1. Payment of Wages Act, 1936

2. Payment of Bonus Act, 1965

3. Employees State Insurance Act, 1948

4. Equal Remuneration Act, 1976

Which of the statements given above is/are correct?

(a)1 and 4 only

(b)1, 2 and 4 only

(c)3 and 4 only

(d)1, 2, 3 and 4

Ans: b

Exp: The Centre will soon notify the rules that will create the mechanisms to fix a floor wage that

would then under gird the minimum wages for different categories of workers— unskilled, semi-

skilled, skilled and highly skilled; that the States and Central government would have to set and

enforce. The new code will amalgamate the Payment of Wages Act, 1936, the Minimum Wages

Act, 1948, the Payment of Bonus Act, 1965, and the Equal Remuneration Act, 1976.

Source:https://economictimes.indiatimes.com/news/economy/policy/rajya-sabha-passes-wage-code-

bill/articleshow/70501009.cms?from=mdr

Q10.With reference to the Minimum Wages Act, 1948, consider the following:

1. Under the Act, both Centre and the State governments are responsible in respect of their scheduled

employments within their jurisdictions to fix and revise minimum wages.

2. The Act is legally non-binding but statutory.

3. Period to revise the minimum wages is five years.

Which of the statements given above is/are correct?

Select the correct answer using the code given below:

(a) 1 and 3

(b) 1 only

(c) 2 and 3

(d) 1, 2 and 3

Ans: d

Exp: Under the Act, both Centre and the State governments are responsible in respect of their

scheduled employments within their jurisdictions to fix and revise minimum wages. The Act is legally

non-binding but statutory. Payment of wages below the minimum wage rate amounts to forced labour.

Source:https://economictimes.indiatimes.com/news/economy/policy/decoded-what-code-on-wages-

mean-for-the-50-crore-workers-it-aims-to-benefit/articleshow/70515807.cms?from=mdr

https://legalbites.in/law-notes-object-validity-features-minimum-wages-act-1948/

Q11.Recently, JEEVANKAUSHAL– curriculum for life skills is launched by

(a) ministry of human resources and development

(b) ministry of skill development and entrepreneurship

(c) ministry of rural development

(d) Prime Minister’s Office

Ans: a

https://t.me/UPSC_PDF Website ➡ https://upscpdf.com https://t.me/UPSC_PDF

Website ➡ https://upscpdf.com

Current Economics MCQs (Jan

2019 to Feb 20) + Eco Survey 2020

WriteToBeIAS.com Prelims 2020 Online Classes

by CA Rahul Kumar

Telegram https://t.me/writetobeias1 Youtube https://tinyurl.com/yx22y2yl Page 66

Exp: Recently, the Ministry of Human Resource Development launched the Curriculum for Life

Skills (Jeevan Kaushal) designed by University Grants Commission (UGC), for the development of

life skills in every individual, as an essential part of wholesome learning.

https://pib.gov.in/PressReleaseIframePage.aspx?PRID=1584801

Q12. SANKALP scheme is related to

(a) Skill development

(b) Rural development

(c) Women empowerment

(d) Agriculture

Ans: a

Exp: SANKALP is an outcome-oriented centrally sponsored programme of Ministry of Skill

Development & Entrepreneurship (MSDE) with a special focus on decentralised planning and quality

improvement. It focuses on the overall skilling ecosystem covering both Central & State agencies.

SANKALP aims to implement the mandate of the National Skill Development Mission (NSDM).

https://pib.gov.in/newsite/PrintRelease.aspx?relid=192464

Q13. Consider the following statements

1. SC/ST households

2. No literate adult above 25 years

3. Female headed household without a male member between the age of 16-59

4. Landless households as manual casual labour

5. Household without any adult member holding a government job

Which of the following fall under deprivation category of SECC 2011?

(a) 1,2,3,5

(b) 2,3,4,5

(c) 1,2,3,4

(d) All

Ans: c

Exp: SECC 2011 has become the source to identify beneficiaries under many government schemes

like MGNREGA in order to provide services as per deprivation. In addition to above, houses with

kutcha walls and kutcha roofs, households with only disabled members without an adult member,

households without a literate also fall under deprivation category.

Source: Yojana September

Q14. “Time to Care” Report released by Oxfam International focuses on:

(a) Alleviation of global poverty

(b) Reducing maternal mortality rate

(c) Eradicating child labour

(d) Non-communicable diseases like mental illness

Ans: a

Exp:

The report is released by Oxfam International. It focuses on the alleviation of global poverty. The

world had 2,153 billionaires in the world in 2019. The number of billionaires has doubled in the last

decade, despite their combined wealth having declined in 2018→ World’s richest 1% have more than

twice as much wealth as 6.9 billion people.

Source:https://economictimes.indiatimes.com/news/economy/indicators/wealth-of-indias-richest-1-

more-than-4-times-of-total-for-70-poorest-oxfam/articleshow/73416122.cms?from=mdr

https://t.me/UPSC_PDF Website ➡ https://upscpdf.com https://t.me/UPSC_PDF

Website ➡ https://upscpdf.com

Current Economics MCQs (Jan

2019 to Feb 20) + Eco Survey 2020

WriteToBeIAS.com Prelims 2020 Online Classes

by CA Rahul Kumar

Telegram https://t.me/writetobeias1 Youtube https://tinyurl.com/yx22y2yl Page 67

Q15. Match the following state government schemes related to skill development.

Scheme- State

1. Samvardhan Yojana w-MadhyaPradesh

2. Kushal Yuva Programme x-Bihar

3. Kushalkar.com y-Karnataka

4. SEEKHO-SIKHAO z-Haryana

Select the correct answer from the following

(a) 1-w, 2-y, 3-x, 4-z

(b) 1-w, 2-x, 3-y, 4-z

(c) 1-x, 2-w, 3-z, 4-y

(d) 1-z, 2-w, 3-y, 4-x

Ans: b

Exp: Other state government schemes related to skill development

Kaushalya Vardhan Kendra–Gujarat

SURYA, SAKSHAM–Haryana

Mukhya Mantri Shram Shakti Yojana–Bihar

Source: Yojana September

Q16. Oxfam International is

(a) an agency of the United Nations to help refugees of civil wars

(b) a global Human Rights Movement

(c) a group of charitable organizations focusing on the alleviation of global poverty

(d) aninter-governmentalagencytocatertomedicalemergenciesinwar-ravagedregions

Ans: c

Exp: Oxfam is a confederation of 19 independent charitable organizations focusing on the alleviation

of global poverty, founded in 1942 and led by Oxfam International. It is a major non profit group

with an extensive collection of operations.

https://www.thehindu.com/news/national/assam-tea-estates-violating-labour-

laws/article29649803.ece

Q17. `SANKALP’, an initiative of the Government of India, aims at

(a) Promoting the Self Help Groups in rural areas

(b) Providing financial and technical assistance to young start-up entrepreneurs

(c) District level skilling ecosystem

(d) Providing affordable and quality education to the citizens for free

Ans: c

Exp: Skill India’s SANKALP Scheme to focus on district level skilling ecosystem through seamless

convergence and coordination

https://pib.gov.in/PressReleseDetail.aspx?PRID=1581204

Q18. Consider the following statements:

1. The 45th World Skills Competition was held in Kazan, Russia in August 2019.

2. India is a member to World Skills International.

Which of the statements given above is/are correct?

(a) 1 only

(b) 2 only

(c) Both 1 and 2

(d) Neither 1 no r2

Ans: c

https://t.me/UPSC_PDF Website ➡ https://upscpdf.com https://t.me/UPSC_PDF

Website ➡ https://upscpdf.com

Current Economics MCQs (Jan

2019 to Feb 20) + Eco Survey 2020

WriteToBeIAS.com Prelims 2020 Online Classes

by CA Rahul Kumar

Telegram https://t.me/writetobeias1 Youtube https://tinyurl.com/yx22y2yl Page 68

Exp: The 48-member Indian Team representing the country at World Skills Kazan 2019 was given a

grand send-off at a ceremony in the capital today, organized by National Skill Development

Corporation (NSDC) under the aegis of Ministry of Skill Development and Entrepreneurship

(MSDE), to motivate the participants as they gear up to pit their skills against the best in the world.

India is the sixth largest team that will take part in the competition.

https://pib.gov.in/PressReleseDetail.aspx?PRID=1582263

Q19. Periodic Labour Force Survey (PLFS) is released by

(a) Labour bureau

(b) The Reserve Bank of India

(c) NSSO

(d) The Department of Economic Affairs

Ans: c

Exp: This Ministry of Statistics & Programme Implementation has launched a new regular

employment-unemployment survey, namely, Periodic Labour Force Survey (PLFS) during April,

2017 with certain changes in survey methodology, data collection mechanism and sampling design

vis-à-vis the earlier quinquennial (once in every five years) Employment and Unemployment surveys

of NSSO. The PLFS has been launched with an objective of measuring quarterly changes of various

labour market statistical indicators in urban areas as well as generating annual estimates of these

indicators both for rural and urban areas, which can be used for policy making.

https://pib.gov.in/PressReleseDetail.aspx?PRID=1575853

Q20. Consider the following statements.

1. Inflation has declined in almost all the countries around the world.

2. However in India, inflation has been showing an uptick.

Select the correct code

a. 1 only

b. 2 only

c. Both 1 and 2

d. Neither 1 nor 2

Ans: c

Exp: The global economy has been witnessing a steep decline in inflation over the past five decades

(World Bank, 2019). Inflation has declined in almost all the countries around the world. Emerging

market economies have also experienced a remarkable decline in inflation over the same period.

In India, inflation has been witnessing moderation since 2014. However, recently inflation has shown

an uptick.

https://www.indiabudget.gov.in/economicsurvey/doc/echapter_vol2.pdf

Q21. Consider the following statements.

1. Headline inflation based on CPI-C has been sliding on an upward path since 2014.

2. The average CPI-C headline inflation has increased continuously.

Select the correct code

a. 1 only

b. 2 only

c. Both 1 and 2

d. Neither 1 nor 2

Ans: d

Exp: Headline inflation based on CPI-C has been sliding on a downward path since 2014

https://t.me/UPSC_PDF Website ➡ https://upscpdf.com https://t.me/UPSC_PDF

Website ➡ https://upscpdf.com

Current Economics MCQs (Jan

2019 to Feb 20) + Eco Survey 2020

WriteToBeIAS.com Prelims 2020 Online Classes

by CA Rahul Kumar

Telegram https://t.me/writetobeias1 Youtube https://tinyurl.com/yx22y2yl Page 69

The average CPI-C headline inflation, which was 5.9 per cent in 2014- 15, has fallen continuously to

around 3.4 per cent in 2018-19.

https://www.indiabudget.gov.in/economicsurvey/doc/echapter_vol2.pdf

Q22. Consider the following statements.

1. CPI-Rural inflation has been consistently above CPI-Urban inflation

2. The divergence has been mainly on account of the differential rates of food inflation between

rural and urban areas witnessed during this period

Select the correct code

a. 1 only

b. 2 only

c. Both 1 and 2

d. Neither 1 nor 2

Ans: b

Exp: Since July 2018, CPI-Urban inflation, has been consistently above CPI-Rural inflation.

The divergence has been mainly on account of the differential rates of food inflation between rural

and urban areas witnessed during this period.

https://www.indiabudget.gov.in/economicsurvey/doc/echapter_vol2.pdf

Q23. Consider the following statements.

1. The overall inflation rate has been quite low in almost all the States.

2. Daman & Diu have the lowest inflation rate amongst all the States.

Select the correct code

a. 1 only

b. 2 only

c. Both 1 and 2

d. Neither 1 nor 2

Ans: c

Exp: CPI-C inflation has continued to be highly variable across States. Inflation ranged between (-

)0.04 per cent to 8.1 per cent across States/UTs in financial year (FY) 2019-20 (April-December)

compared to (-)1.3 per cent to 9.1 per cent in FY 2018-19 (AprilDecember). However, the overall

inflation rate has been quite low in almost all the States.

Nineteen States/UTs had inflation rate lower than All India average for FY 2019-20 (April-

December) with Daman & Diu having the lowest inflation followed by Bihar and Chhattisgarh

https://www.indiabudget.gov.in/economicsurvey/doc/echapter_vol2.pdf

Q24. Consider the following statements about CPI-C inflation in 2019-20.

1. Food and beverages emerged as the main contributor to CPI-C inflation

2. The major driver of CPI-C inflation was the miscellaneous group.

Select the correct code

a. 1 only

b. 2 only

c. Both 1 and 2

d. Neither 1 nor 2

Ans: a

Exp: During 2018-19, the major driver of CPI-C inflation was the miscellaneous group. Compared to

2017-18, the contribution of food and beverages to total inflation was lower in 2018-19. However,

during 2019-20 (April- December), food and beverages emerged as the main contributor to CPI-C

https://t.me/UPSC_PDF Website ➡ https://upscpdf.com https://t.me/UPSC_PDF

Website ➡ https://upscpdf.com

Current Economics MCQs (Jan

2019 to Feb 20) + Eco Survey 2020

WriteToBeIAS.com Prelims 2020 Online Classes

by CA Rahul Kumar

Telegram https://t.me/writetobeias1 Youtube https://tinyurl.com/yx22y2yl Page 70

inflation, with 54 per cent of the inflation during this period attributable to this group. Miscellaneous

group was the second largest contributor to inflation during this period.

https://www.indiabudget.gov.in/economicsurvey/doc/echapter_vol2.pdf

Q25. Which of the following are reasons for a high spread between the wholesale and retail prices ?

1. high transaction costs

2. weak infrastructure

3. information systems

4. poor marketing facilities

5. huge margins of middleman

Select the correct code

a. 1, 2, 3, 5

b. 2, 3, 4, 5

c. 1, 3, 4, 5

d. 1, 2, 3, 4, 5

Ans: d

Exp: The reasons for such a high spread between the wholesale and retail prices could be due to

several reasons such as high transaction costs, weak infrastructure and information systems, poor

marketing facilities, huge margins of middleman etc.

https://www.indiabudget.gov.in/economicsurvey/doc/echapter_vol2.pdf

Q10.Consider the following statements

1. Expenditure on social services, as a proportion of GDP, has increased by 1.5%.

2. Total formal employment in the economy increased to 9.98%

Select the correct code

a. 1 only

b. 2 only

c. Both 1 and 2

d. Neither 1 nor 2

Ans: c

Exp: As a proportion of GDP, the expenditure on social services has registered an increase of more

than 1 percentage points during the period from 2014-15 to 2018-19 (BE), from 6.2 per cent in 2014-

15 to 7.3 per cent in 2018-19 (BE).

Total formal employment in the economy increased from 8% in 2011-12 to 9.98% in 2017-18, stated

the survey released today.

https://pib.gov.in/Pressreleaseshare.aspx?PRID=1577032

https://economictimes.indiatimes.com/small-biz/sme-sector/formal-jobs-on-the-up-stands-at-9-98-

in-2017-18-economic-survey-2019-20/articleshow/73802278.cms?from=mdr

Q12.Consider the following statements

1. Least expenditure in the various components of education is for the course fees.

2. Students pursuing education in private aided institutions are spending significantly higher as

compared to government institutions

Select the correct code

a. 1 only

b. 2 only

c. Both 1 and 2

d. Neither 1 nor 2

https://t.me/UPSC_PDF Website ➡ https://upscpdf.com https://t.me/UPSC_PDF

Website ➡ https://upscpdf.com

Current Economics MCQs (Jan

2019 to Feb 20) + Eco Survey 2020

WriteToBeIAS.com Prelims 2020 Online Classes

by CA Rahul Kumar

Telegram https://t.me/writetobeias1 Youtube https://tinyurl.com/yx22y2yl Page 71

Ans: b Exp: Of the total expenses incurred on the education of every student, over half of the amount is

spent on tuition and exam fees — and that excludes course material, books and uniform.

https://www.businessinsider.in/education/news/average-education-expenditure-in-india-increases-

fourfold-to-8331-per-

student/articleshow/72282009.cms?utm_source=contentofinterest&utm_medium=text&utm_campa

ign=cppst

POPULATION AND DEMOGRAPHY

Q1. Consider the following:

1. World Population Prospects is released by United Nations.

2. Around 2050, India is projected to overtake China as the world’s most populous country.

Which of the statements given above is/are correct?

(a) 1 only

(b) 2 only

(c) Both 1 and 2

(d) Neither 1 nor 2

Ans: a

Exp: The world’s population is expected to increase by 2 billion persons in the next 30 years, from

7.7 billion currently to 9.7 billion in 2050, according to a new United Nations report launched today.

Around 2027, India is projected to overtake China as the world’s most populous country.

https://www.thehindu.com/opinion/editorial/a-stable-planet/article28102991.ece

INTERNATIONAL INSTITUTIONS

Q1. ‘Libor’ is

(a) Interbank Interest Rate

(b) London crude oil variety

(c) Coming Olympics mascot

(d) None of the above

Ans: a

Exp: The London Interbank Offered Rate (Libor)

https://www.thehindu.com/business/Industry/libor-transition-opens-up-avenues-for-it-

majors/article26935586.ece

Q2. Consider following statements:

1. The SWIFT is a global member-owned cooperative.

2. It is headquartered in London.

3. It developed a secure electronic messaging service and common standards to facilitate cross- border

payments.

Which of the statements given above is/are correct?

Select the correct answer using the code given below:

(a) 1 only

(b) 1 and 2 only

https://t.me/UPSC_PDF Website ➡ https://upscpdf.com https://t.me/UPSC_PDF

Website ➡ https://upscpdf.com

Current Economics MCQs (Jan

2019 to Feb 20) + Eco Survey 2020

WriteToBeIAS.com Prelims 2020 Online Classes

by CA Rahul Kumar

Telegram https://t.me/writetobeias1 Youtube https://tinyurl.com/yx22y2yl Page 72

(c) 1 and 3 only

(d) 1, 2 and 3

Ans: c

Exp: The SWIFT is a global member-owned cooperative. It is headquartered in Brussels, Belgium.

It developed a secure electronic messaging service and common standards to facilitate cross-border

payments.

https://www.thehindu.com/business/the-lowdown-on-swift-and-bank-fraud/article22846010.ece

Q3.Consider following statements

1. WTO has an Appellate Body which hears appeals in disputes brought by WTO Members.

2. Dispute Settlement Body (DSB) is the supreme body for disputes settlement at WTO.

Which of the statements given above is/are correct?

(a) 1 only

(b) 2 only

(c) Both 1 and 2

(d) Neither 1 nor 2

Ans: c

Exp: The General Council convenes as the Dispute Settlement Body (DSB) to deal with disputes

between WTO members.

http://www.pib.nic.in/PressReleseDetail.aspx?PRID=1562519

Q4. Consider the following -

1. Minimum Regulatory Capital Requirements based on Risk Weighted Assets (RWAs)

2. Supervisory Review Process

3. Market Discipline

Which of the above are pillars of BASEL III for banking sector?

Select the correct answer using the code given below:

(a) 1 and 2 only

(b) 2 and 3 only

(c) 1 and 3 only

(d) 1, 2 and 3

Ans: d

Exp: All the above are three pillars of BASEL III.

https://www.thehindu.com/todays-paper/tp-business/rbi-notifies-deferment-of-capital-buffer-

norms-as-per-boards-call/article25964632.ece

Q5. Consider the following statements:-

1. World Economy Outlook (WEO) is issued by World Bank.

2. India is projected to grow ahead of China as WEO-2019

Which statement/statements given above is/are correct?

Select the correct answer using the codes given below.

(a) 1 only

(b) 2 only

(c) Both 1 and 2

(d) None

Ans: b

Exp: World Economy Outlook is issued by International Monetary Fund (IMF). India is projected

to grow at 7.5 per cent in 2019 and 7.7 per cent in 2020, an impressive over one

percentage point ahead of China’s estimated growth of 6.2 per cent in these two years.

https://t.me/UPSC_PDF Website ➡ https://upscpdf.com https://t.me/UPSC_PDF

Website ➡ https://upscpdf.com

Current Economics MCQs (Jan

2019 to Feb 20) + Eco Survey 2020

WriteToBeIAS.com Prelims 2020 Online Classes

by CA Rahul Kumar

Telegram https://t.me/writetobeias1 Youtube https://tinyurl.com/yx22y2yl Page 73

Q5. Consider following statements

1. The Organization of the Petroleum Exporting Countries (OPEC) is headquartered in Vienna,

Austria.

2. India is one of the 14 Members of the OPEC.

Which of the statements given above is/are correct?

(a) 1 only

(b) 2 only

(c) Both 1 and 2

(d) Neither 1 nor 2

Ans: a

Exp: The Organization of the Petroleum Exporting Countries is an intergovernmental organization

of 14 nations, founded in 1960 in Baghdad by the first five members (Iran, Iraq, Kuwait, Saudi Arabia,

and Venezuela), and headquartered since 1965 in Vienna, Austria. The current OPEC members are

the following: Algeria, Angola, Ecuador, Equatorial Guinea, Gabon, Iran, Iraq, Kuwait, Libya,

Nigeria, the Republic of the Congo, Saudi Arabia (the de facto leader), United Arab Emirates, and

Venezuela. Indonesia and Qatar are former members.

http://www.pib.nic.in/PressReleseDetail.aspx?PRID=1562541

Q6. In the BIMSTEC, an initiative of seven countries, which of the following is/are a participant/

participants?

1. Bangladesh

2. Bhutan

3. China

4. Myanmar

5. Thailand

Select the correct answer using the code given below.

(a) 1 only

(b) 2, 3 and 4

(c) 1 and 3

(d) 1, 2,4 and 5

Ans: d

Exp: The Bay of Bengal Initiative for Multi-Sectoral Technical and Economic Cooperation

(BIMSTEC) is a regional organization comprising seven Member States lying in the littoral and

adjacent areas of the Bay of Bengal constituting a contiguous regional unity. This sub-regional

organization came into being on 6 June 1997 through the Bangkok Declaration. It constitutes seven

Member States: five deriving from South Asia, including Bangladesh, Bhutan, India, Nepal, Sri

Lanka, and two from Southeast Asia, including Myanmar and Thailand. Initially, the economic bloc

was formed with four Member States with the acronym ‘BIST-EC’ (Bangladesh, India, Sri Lanka

and Thailand Economic Cooperation). Following the inclusion of Myanmar on 22 December 1997

during a special Ministerial Meeting in Bangkok, the Group was renamed ‘BIMST-EC’ (Bangladesh,

India, Myanmar, Sri Lanka and Thailand Economic Cooperation). With the admission of Nepal and

Bhutan at the 6th Ministerial Meeting (February 2004, Thailand), the name of the grouping was

changed to ‘Bay of Bengal Initiative for Multi-Sectoral Technical and Economic Cooperation’

(BIMSTEC)

https://www.thehindu.com/opinion/op-ed/the-importance-of-being-neighbourly/article27819079.ece

Q7. Which one of the following issues the ‘World Economic outlook’ report periodically?

(a) International Monetary Fund

https://t.me/UPSC_PDF Website ➡ https://upscpdf.com https://t.me/UPSC_PDF

Website ➡ https://upscpdf.com

Current Economics MCQs (Jan

2019 to Feb 20) + Eco Survey 2020

WriteToBeIAS.com Prelims 2020 Online Classes

by CA Rahul Kumar

Telegram https://t.me/writetobeias1 Youtube https://tinyurl.com/yx22y2yl Page 74

(b) The European Bank for Reconstruction and Development

(c) The US Federal Reserve Bank

(d) The World Bank

Ans: a

Exp: The IMF in its latest World Economic Outlook projected India’s growth rate at 6.1% in 2019

and noted that the Indian economy is expected to pick up the next year at 7.0% in 2020.

https://www.thehindu.com/business/Economy/imf-cuts-indias-growth-projection-to-61-in-

2019/article29692020.ece

Q8. ‘Illuminating Inequalities’ released in 2019 is related to

(a) UN Development Programme

(b) World Economic Forum

(c) Amnesty International

(d)Oxfam India

Ans. a

Exp: The 2019 Global Multidimensional Poverty Index (MPI) data and publication "Illuminating

Inequalities" released on 11 July 2019 shed light on the number of people experiencing poverty at

regional, national and sub-national levels, and reveal inequalities across countries and among the poor

themselves. It is jointly developed by the United Nations Development Programme (UNDP)

and the Oxford Poverty and Human Development Initiative (OPHI) at the University of

Oxford, the 2019 global MPI offers data for 101 countries, covering 76 percent of the global

population.

Source:http://hdr.undp.org/en/2019-MPI

Q9. The term ‘Regulatory Consistency Assessment Programme (RCAP)’ is sometimes mentioned

in media with reference to

(a) Basel Norms

(b) FDI

(c) Insurance

(d) Participatory Notes

Ans. a

Exp: An assessment of compliance with Basel Norms was recently conducted by the Regulatory

Consistency Assessment Programme (RCAP). RCAP is part of the Basel committee

Source: https://www.bis.org/bcbs/implementation/rcap_role.htm

Q10.Recently India was part of the ‘Summit on Financial Markets and the World Economy’ (2019)

that took place in which of the following countries?

(a) India

(b) Brazil

(c) Japan

(d) France

Ans. c

Exp:The G20 Summit is formally known as the “Summit on Financial Markets and the World

Economy”. It was recently held at Osaka, Japan.

Source:https://www.japan.go.jp/g20japan/

Q11. Which one of the following issues the “Special Data Dissemination Standard (SDDS)”

(a) World Bank

https://t.me/UPSC_PDF Website ➡ https://upscpdf.com https://t.me/UPSC_PDF

Website ➡ https://upscpdf.com

Current Economics MCQs (Jan

2019 to Feb 20) + Eco Survey 2020

WriteToBeIAS.com Prelims 2020 Online Classes

by CA Rahul Kumar

Telegram https://t.me/writetobeias1 Youtube https://tinyurl.com/yx22y2yl Page 75

(b) International Monetary Fund

(c) United Nations

(d) The Asian Development Bank

Ans: b

Exp. India has subscribed to the Special Data Dissemination Standard (SDDS) of the International

Monetary Fund (IMF) and an Advance Release Calendar is decided for release of estimates. The IMF

had raised certain issues on the usage of double deflation in the Indian GDP series and India has

informed IMF that the existing data availability does not permit its application in India at present.

https://pib.gov.in/PressReleseDetail.aspx?PRID=1573808

Q12.Consider the following

1. PISA was introduced by the Organisation for Economic Cooperation Development (OECD) to test

the learning levels of 15-year-olds in reading, mathematics and science.

2. India is one of participant of the programme.

Which of the statements given above is/are correct?

(a) 1 only

(b) 2 only

(c) Both 1 and 2

(d) Neither 1 nor 2

Ans: a

Exp: Government of India has decided to participate in the Programme for International Students

Assessment (PISA) to be conducted by the Organization for Economic Cooperation and Development

(OECD) in 2021. PISA — introduced by the Organisation for Economic Cooperation Development

(OECD) — tests the learning levels of 15-year-olds in reading, mathematics and science.

https://pib.gov.in/PressReleseDetail.aspx?PRID=1575471

Q13. Consider the following statements:

1. The Bank of International Settlement is owned by 60 central banks, representing countries from

around the world that together account for about 95% of world GDP.

2. Its head office is in Bern, Switzerland.

Which of the above statements is/are correct?

(a) 1 only

(b) 2 only

(c) Both 1 and 2

(d) Neither 1 nor 2

Ans: a

Exp: Established in 1930, the BIS is owned by 60 central banks, representing countries from around

the world that together account for about 95% of world GDP. Its head office is in Basel, Switzerland

and it has two representative offices: in Hong Kong SAR and in Mexico City.

https://www.thehindu.com/business/sebi-simplifies-norms-for-foreign-

investors/article29214341.ece

Q14. Consider the following about Dispute settlement Body of WTO

1. The General Council is WTO’s highest decision-making body and it also meets as the DSB.

2. Decisions are taken here by the reverse consensus method.

3. Its decisions are final and binding

Which among the following statements is/are correct?

(a) 1 and 2

(b) 2 and 3

https://t.me/UPSC_PDF Website ➡ https://upscpdf.com https://t.me/UPSC_PDF

Website ➡ https://upscpdf.com

Current Economics MCQs (Jan

2019 to Feb 20) + Eco Survey 2020

WriteToBeIAS.com Prelims 2020 Online Classes

by CA Rahul Kumar

Telegram https://t.me/writetobeias1 Youtube https://tinyurl.com/yx22y2yl Page 76

(c) 1 and 3

(d) all

Ans: a

Exp: Dispute Settlement Body- The General Council is WTO’s highest decision-making body and it

also meets as the DSB. It is essentially a political body and it administers rules and procedure of the

DSU. Decisions are taken hereby the reverse consensus method. That is, the decision is adopted

unless there is consensus against it.

Appellate Body is a seven-member permanent organ that adjudicates appeals within the DSS.

Members are appointed by the DSB for four-year terms. It follows the positive consensus mechanism.

https://www.thehindu.com/opinion/op-ed/the-problem-at-the-wto/article24609374.ece

Q15. Which of the following organizations brings out the publication known as ‘Ease of doing

business' ?

(a) The International Monetary Fund

(b) The United Nations Development Programme

(c) The World Economic Forum

(d) The World Bank

Ans: d

Exp: Ease of doing business index (1=most business-friendly regulations) from The World Bank

https://pib.gov.in/PressReleseDetail.aspx?PRID=1586891

Q16. The “Global Risks Report” is published by the

(a) International Monetary Fund

(b) United Nations Conference on Trade and Development

(c) World Economic Forum

(d) World Bank

Ans: c

Exp:

The 15thedition of the World Economic Forum’s Global Risks Report is published as critical risks are

manifesting. The global economy is facing an increased risk of stagnation, climate change is striking

harder and more rapidly than expected, and fragmented cyberspace threatens the full potential of next-

generation technologies—all while citizens world wide protest political and economic conditions and

voice concerns about systems that exacerbate inequality. The challenges before us demand immediate

collective action, but fractures within the global community appear to only be widening. Stake holders

need to act quickly and with purpose within an unsettled global landscape.

https://www.thehindu.com/business/Economy/in-charts-international-monetary-funds-world-

economic-outlook/article29734182.ece

Q17. Which of the following organizations brings out the publication known as ‘Global social

mobility index'?

(a) The International Monetary Fund

(b) The United Nations Development Programme

(c) The World Economic Forum

(d) The World Bank

Ans: c

Exp: The World Economic Forum’s Global Social Mobility Index provides a new, holistic assessment

of 82 global economies according to their performance on five key dimensions of social mobility

distributed over 10 pillars: 1. Health; 2. Education (access, quality and equity, lifelong learning); 3.

https://t.me/UPSC_PDF Website ➡ https://upscpdf.com https://t.me/UPSC_PDF

Website ➡ https://upscpdf.com

Current Economics MCQs (Jan

2019 to Feb 20) + Eco Survey 2020

WriteToBeIAS.com Prelims 2020 Online Classes

by CA Rahul Kumar

Telegram https://t.me/writetobeias1 Youtube https://tinyurl.com/yx22y2yl Page 77

Technology; 4. Work (opportunities, wages, conditions); 5. Protection and Institutions (social

protection and inclusive institutions).

The Global Social Mobility Index, which benchmarks 82 global economies, is designed to provide

policymakers with a means to identify areas for improving social mobility and promoting equally

shared opportunities in their economies, regardless of their development.

https://www.thehindu.com/business/india-ranks-low-at-76th-place-on-global-social-mobility-

index/article30607184.ece

Q18. Lisbon Treaty is related to

(a) Extradition of prisoners

(b) European Union

(c) Climate change

(d) None of the above

Ans: b

Exp: The Treaty of Lisbon (initially known as the Reform Treaty) is an international agreement that

amends the two treaties which form the constitutional basis of the European Union (EU). The Treaty

of Lisbon was signed by the EU member states on 13 December 2007, and entered into force on 1

December 2009.

https://www.thehindu.com/news/international/britains-last-day-in-eu-here-is-the-chronology-of-

brexit/article30703156.ece

Q19. Consider the following statements:

1. The “New Development Bank” International organization was jointly founded by the IBSA

Countries.

2. All countries have equal shareholding and voting rights.

Which of the above statements is/are correct?

(a) 1 only

(b) 2 only

(c) Both 1 and 2

(d) Neither 1 nor 2

Ans: b

Exp: The NEW DEVELOPMENT BANK International organization jointly founded by the BRICS

Countries. All countries have equal shareholding and voting rights.

https://pib.gov.in/PressReleseDetail.aspx?PRID=1587688

Q20. New Southern Policy is strategy of foreign relations of which of the following countries?

(a) South Korea

(b) India

(c) China

(d) America

Ans: a

Exp: NSP is part of South Korean government’s broader strategy of promoting a ‘Northeast Asia

Plus Community for Responsibility-sharing’ (NEAPC). The New Southern Policy intends to build

stronger economic linkages with countries to Korea’s south in Southeast Asia and India. The New

Northern Policy, meanwhile, focuses on countries to Korea’s north including Russia, Mongolia and

Central Asian countries.

https://www.thehindu.com/news/international/s-korea-wants-to-elevate-ties-with-

india/article24988305.ece

https://www.thehindu.com/news/national/india-south-korea-seal-logistics-pact/article29354781.ece

https://t.me/UPSC_PDF Website ➡ https://upscpdf.com https://t.me/UPSC_PDF

Website ➡ https://upscpdf.com

Current Economics MCQs (Jan

2019 to Feb 20) + Eco Survey 2020

WriteToBeIAS.com Prelims 2020 Online Classes

by CA Rahul Kumar

Telegram https://t.me/writetobeias1 Youtube https://tinyurl.com/yx22y2yl Page 78

Q21. The term “Far East” often in news refer to

(a) Eastern part of Russia

(b) Eastern part of Pacific Ocean

(c) Eastern part of North East India

(d) None

Ans: a

Exp: Recently, the 20th India-Russia annual summit and the fifth meeting of the Eastern Economic

Forum (EEF) was held in Vladivostok, Russia. The focus of the visit was on the development of the

Far East for which India has extended a $ 1 billion line of credit.

https://economictimes.indiatimes.com/news/economy/foreign-trade/india-promotes-long-term-big-

ticket-investments-at-eastern-economic-forum/articleshow/70971960.cms?from=mdr

Q22. Global Microscope on Financial Inclusion Report is published by:

(a) IMF

(b) The Economist Intelligence Unit

(c) World Economic Forum

(d) INSEAD

Ans: b

Exp: It is released by the Economist Intelligence Unit. It features 11 new gender focussed indicators

that measure financial inclusion for both women as well as men. India is among top nations with most

conducive environment for financial inclusion in terms of allowing non-banks to issuee-money,

proportionate customer due diligence and effective consumer protection. South Africa, India, Mexico,

Tanzania and Uruguay were among the top countries that safeguarde-money via some sort of deposit

insurance or protection.

Source:https://www.thehindubusinessline.com/economy/india-among-top-nations-with-most-

conducive-environment-for-financial-inclusion-report/article29839466.ece

Q23. With reference to Tropical Forest Alliance 2020, consider the following statements:

1. It was founded in 2012 at Rio+20.

2. This project is part of the World Economic Forum’s Shaping the Future of Global Public Goods

Platform.

Which of the statements given above is/are correct?

(a) 2 only

(b) 1 only

(c) Both 1 and 2

(d) Neither 1 nor 2

Ans: c

Exp: The Tropical Forest Alliance is a global public-private partnership dedicated to collaborative

action to realize sustainable rural development and better growth opportunities based on reduced

deforestation and sustainable land use management in tropical forest countries

Source: https://www.weforum.org/projects/tfa-2020

Q24. Global Investment Trend Monitor Report was recently released by:

(a) UNCTAD

(b) IMF

(c) WTO

(d) World Bank

Ans: a

https://t.me/UPSC_PDF Website ➡ https://upscpdf.com https://t.me/UPSC_PDF

Website ➡ https://upscpdf.com

Current Economics MCQs (Jan

2019 to Feb 20) + Eco Survey 2020

WriteToBeIAS.com Prelims 2020 Online Classes

by CA Rahul Kumar

Telegram https://t.me/writetobeias1 Youtube https://tinyurl.com/yx22y2yl Page 79

Exp: The United Nations Conference on Trade and Development (UNCTAD) said that India was

among the top 10 recipients of Foreign Direct Investment (FDI) in 2019, attracting $49 billion in

inflows, a 16 % increase from the previous year, driving the FDI growth in South Asia. The majority

went into services industries, including information technology. UNCTAD, in its Global Investment

Trend Monitor report said that the global foreign direct investment remained flat in 2019 at $1.39

trillion,a 1 % decline from a revised $ 1.41 trillion in 2018.

Source:https://economictimes.indiatimes.com/news/economy/indicators/india-attracted-49-billion-

fdi-in-2019-among-top-10-recipients-of-overseas-investment-unctad/articleshow/73441481.cms

Q25. India is set to sign ‘Agreement on Reciprocal Logistics Support (ARLS)’ with which of the

following countries?

(a) USA

(b) UK

(c) Japan

(d) Russia

Ans: d

Exp: India and Russia are expected to conclude a mutual logistics agreement and review the setting

up of joint ventures for manufacturing spares for Russian defence platforms in India. The ARLS is

an arrangement which will allow access to each other’s military facilities for supplies and fuel,

expanding the logistics support and operational turn around of the Indian military. This will be

especially beneficial for the Indian Navy, which has a large number of Russian origin ships, that will

get access to Russian ports for supplies and refueling. This will be crucial for joint exercises. The air

force too will benefit by finding it easier to deploy aircraft for the same purpose.

Source:https://www.thehindu.com/news/national/india-russia-to-conclude-mutual-logistics-

agreement/article29881623.ece

Q26. Women, Business and the Law (WBL) is a project collecting unique data on the laws and

regulations that restrict women’s economic opportunities. It is an initiative of:

(a) OECD

(b) World Bank Group

(c) UN Women

(d) UNDP

Ans: b

Exp: Women, Business and the Law (WBL) is a World Bank Group project collecting unique data

on the laws and regulations that restrict women’s economic opportunities (in 190 countries). India

gained 4% year-on-year on its score on the World Bank study of WBL 2020. The country scored 74.4

out of the perfect score of 100 on the WBL Index.

Source:https://economictimes.indiatimes.com/news/economy/indicators/india-commands-the-top-

score-on-the-women-business-and-law-index-2020-in-south-

asia/articleshow/73517164.cms?from=mdr

Q27. Global Talent Competitiveness Index (GTCI) 2020 was published by:

(a) World Bank

(b) Oxfam

(c) INSEAD

(d) IMD World Competitiveness Center

Ans: c

https://t.me/UPSC_PDF Website ➡ https://upscpdf.com https://t.me/UPSC_PDF

Website ➡ https://upscpdf.com

Current Economics MCQs (Jan

2019 to Feb 20) + Eco Survey 2020

WriteToBeIAS.com Prelims 2020 Online Classes

by CA Rahul Kumar

Telegram https://t.me/writetobeias1 Youtube https://tinyurl.com/yx22y2yl Page 80

Exp: GTCI report compiled by INSEAD, in collaboration with human resource firm Addeco and

Google. India has climbed eight places to 72nd rank in the 2020 GTCI, which was topped by

Switzerland, the US and Singapore.

Source:https://www.livemint.com/news/india/india-climbs-eight-places-to-rank-72-among-132-

countries-in-global-talent-index-11579787080354.html

Q11.Human Development Index is published by

a. World Bank

b. UNESCO

c. WHO

d. UNDP

Ans: d

INDUSTRY

Q1. With which of the following country India has signed MoU on Cooperation in the field of

Geology and Mineral Resources

(a) Bolivia

(b) Argentina

(c) Peru

(d) South Africa

Ans: a

Exp: MoU on Cooperation in the field of Geology and Mineral Resources

• It will provide an institutional mechanism between India and Bolivia for cooperation in the field of

Mineral Resources.

• The MoU provides for exchange of information on resources, laws and policy, organization of

seminars to exchange views on development strategies, encouragement of transfer of technologies

between the two countries, promotion of value addition, which will enable taking up of activities like

documentation and dissemination etc.

http://pib.nic.in/newsite/PrintRelease.aspx?relid=189750

Q2. Regarding Index of Industrial Production (IIP), which of the following statements are true?

1. It is an annual production index, which is also considered as a lead indicator of the industrial

performance.

2. It is compiled and published by the central statistical organization (CSO).

3. The current base year of IIP is 2011-2012.

Select the correct answer using the code given below:

(a) 1 and 2 only

(b) 2 and 3 only

(c) 1 and 3 only

(d) 1, 2 and 3

Ans: b

Exp: It is a monthly production index, which is also considered as a lead indicator of the industrial

performance.

http://pib.nic.in/newsite/PrintRelease.aspx?relid=189879

http://www.arthapedia.in/index.php?title=Index_of_Industrial_Production_(IIP)_-_2011-12_series

https://t.me/UPSC_PDF Website ➡ https://upscpdf.com https://t.me/UPSC_PDF

Website ➡ https://upscpdf.com

Current Economics MCQs (Jan

2019 to Feb 20) + Eco Survey 2020

WriteToBeIAS.com Prelims 2020 Online Classes

by CA Rahul Kumar

Telegram https://t.me/writetobeias1 Youtube https://tinyurl.com/yx22y2yl Page 81

Q3. With reference to “Faster Adoption and Manufacturing of Electric Vehicles II (FAME II)

scheme”, which of the following statements is/are correct?

1. Under it, Emphasis is on electrification of the public transportation that includes shared transport.

2. The scheme proposes for establishment of charging infrastructure

Select the correct answer using the code given below:

(a) 1 only

(b) 2 only

(c) Both 1 and 2

(d) Neither 1nor 2

Ans: c

Exp: Faster Adoption and Manufacturing of Electric Vehicles II (FAME II) scheme

o Emphasis is on electrification of the public transportation that includes shared transport.

o Establishment of Charging stations are also proposed on major highways connecting major

city clusters.

o On such highways, charging stations will be established on both sides of the road at an interval

of about 25 km each.

http://pib.nic.in/newsite/PrintRelease.aspx?relid=189644

Q4.Consider the following statements:

1. National Automotive Testing and R&D Infrastructure Project (NATRiP), represents a unique

joining of hands between the Government of India, a number of State Governments and Indian

Automotive Industry to create a state of the art Testing, Validation and R&D infrastructure in the

country.

2. Indian Automotive Industry is a Sunshine industry in India contributing to over 7.5% to country’s

GDP

Which of the statements given above is/are correct?

(a) 1 only

(b) 2 only

(c) Both 1 and 2

(d) Neither 1 nor 2

Ans: c

Exp: Indian Automotive Industry: Sunshine industry in India contributing to over 7.5% to country’s

GDP

• Creation of new automotive test centres in the country under NATRIP project, is an important step

forward by the Government to support the need of the automotive industry in the backdrop of

tightening regulatory framework and technology shift driven by market forces.

https://www.natrip.in/

Q5. Consider the following statements:

1. FAGMIL is a Government of India Undertaking, under the administrative control of Department

of Fertilizers, Ministry of Chemicals and Fertilizers.

2. It is engaged in mining and marketing of mineral gypsum in Madhya Pradesh.

3. FAGMIL is Miniratna-II Company.

Which of the statements given above is/are correct?

(a) 1 and 2 only

(b) 2 and 3 only

(c) 1 and 3 only

https://t.me/UPSC_PDF Website ➡ https://upscpdf.com https://t.me/UPSC_PDF

Website ➡ https://upscpdf.com

Current Economics MCQs (Jan

2019 to Feb 20) + Eco Survey 2020

WriteToBeIAS.com Prelims 2020 Online Classes

by CA Rahul Kumar

Telegram https://t.me/writetobeias1 Youtube https://tinyurl.com/yx22y2yl Page 82

(d) 1, 2 and 3

Ans: c

Exp: FAGMIL is a Government of India Undertaking, under the administrative control of

Department of Fertilizers, Ministry of Chemicals and Fertilizers. It is engaged in mining and

marketing of mineral gypsum in Rajasthan. FAGMIL is Miniratna-II Company.

http://pib.nic.in/PressReleseDetail.aspx?PRID=1566246

Q6. Consider the following statements regarding “Turmeric Production in India”

1. India is the world’s second largest producer of turmeric

2. Maharashtra is the largest grower of turmeric in the country

Which of the statements given above is/are correct?

(a) 1 only

(b) 2 only

(c) Both 1 and 2

(d) Neither 1 nor 2

Ans: d

Exp: Turmeric Production in India

India is the world’s largest producer of turmeric (Curcuma longa), a perennial herbaceous plant of

the ginger family.

Telangana is the largest grower of turmeric in the country (followed by Maharashtra and Tamil Nadu).

• Recently Odisha’s Kandhamal Haldi (turmeric) has received GI tag.

https://www.thehindu.com/society/more-yellow-than-most/article26676344.ece

Q7. Consider the following statements:

1. India is the second largest producer of silk in the world.

2. Mulberry, Eri, Tussar and Muga are the main type of silk produced in the country.

Select the correct answer using the code given below:

(a) 1 only

(b) 2 only

(c) Both 1 and 2

(d) Neither 1 nor 2

Ans: c

Exp: India is the second largest producer of silk n the world. Mulberry, Eri, Tussar and Muga are the

main type of silk produced in the country.

http://yojana.gov.in/details_leadarticle.asp?CoverId=101

Q8. Consider the following statements:

1. India is the second largest producer of jute in the world.

2. India is the third largest manufacturer of cotton in the world.

Select the correct answer using the code given below:

(a) 1 only

(b) 2 only

(c) Both 1 and 2

(d) Neither 1 nor 2

Ans: b

Exp: India is the largest producer of jute in the world. India is the third largest manufacturer of cotton

in the world.

http://yojana.gov.in/details_leadarticle.asp?CoverId=101

https://t.me/UPSC_PDF Website ➡ https://upscpdf.com https://t.me/UPSC_PDF

Website ➡ https://upscpdf.com

Current Economics MCQs (Jan

2019 to Feb 20) + Eco Survey 2020

WriteToBeIAS.com Prelims 2020 Online Classes

by CA Rahul Kumar

Telegram https://t.me/writetobeias1 Youtube https://tinyurl.com/yx22y2yl Page 83

Q9. “Interconnect Usage Charge” recently seen in news is related to

(a) Train services

(b) Airport services

(c) Internet services

(d) Telecom services

Ans: d

Exp: Traffic symmetry indicates that telecom operator will not have any outstanding balance of

interconnect usage charges (IUC) against other networks.

https://www.thehindu.com/business/Industry/delay-in-implementation-of-zero-call-connect-

charges-to-hurt-service-affordability-jio/article29981799.ece

Q11.What does Viability Gap Fund mean?

(a) A long-term start-up capital provided to new entrepreneurs

(b) Funds provided to industries at times of incurring losses

(c) Funds provided for replacement and renovation of industries

(d) Viability Gap Funding is a certain percentage of capital cost paid by the government

Ans: d

Exp: Viability Gap Funding (VGF) is an incentive scheme in which a certain percentage of total

capital cost is paid by the government to make the project economically viable.

https://pib.gov.in/PressReleseDetail.aspx?PRID=1594751

Q12. Consider the following statements:

1. Paris Convention for the Protection of Industrial Property, adopted in 1883, applies to industrial

property in the widest sense, including patents, trademarks, industrial designs etc.

2. It is administered by the World trade Organisation.

Which of the statements given above is/are correct?

(a) 1 only

(b) 2 only

(c) Both 1 and 2

(d) Neither 1 nor 2

Ans: a

Exp:The Paris Convention is a multilateral treaty dealing with the protection of industrial property

in the widest sense. It is administered by the World Intellectual Property Organization (WIPO),

one of the specialised agencies of the United Nations dealing with the protection and promotion of

intellectual property rights.

Source:https://economictimes.indiatimes.com/industry/cons-products/garments-/-textiles/kvic-

seeks-international-trademark-for-khadi-items/articleshow/73507291.cms?from=mdr

Q13. Consider the following statements:

1. Ministry of Heavy Industries and Public Enterprises has launched mission purvodaya.

2. It aims to develop eastern India by accelerated development of roadways and energy sector.

Which of the statements given above is/are correct?

(a) 1only

(b) 2only

(c) Both 1 and 2

(d) Neither 1 nor 2

Ans:d

https://t.me/UPSC_PDF Website ➡ https://upscpdf.com https://t.me/UPSC_PDF

Website ➡ https://upscpdf.com

Current Economics MCQs (Jan

2019 to Feb 20) + Eco Survey 2020

WriteToBeIAS.com Prelims 2020 Online Classes

by CA Rahul Kumar

Telegram https://t.me/writetobeias1 Youtube https://tinyurl.com/yx22y2yl Page 84

Exp:Ministry of Steel has launched PURVODAYA: Accelerated development of eastern India

through integrated steel hub in Kolkata, West Bengal. It aims to develop eastern India by accelerated

development of steel sector through an integrated steelhub

Source:https://pib.gov.in/newsite/PrintRelease.aspx?relid=197372

Q14. Consider the following statements:

1. The country is 4th largest consumers of finished steel in world.

2. India is the largest crude steel producer.

Which of the above statements is/are correct?

(a) 1 only

(b) 2 only

(c) Both 1 and 2

(d) Neither 1 nor 2

Ans: d

Exp: The country is 3rd largest consumers of finished steel in world. India was the 2nd largest crude

steel producer.

https://pib.gov.in/PressReleseDetail.aspx?PRID=1597299

Q15. PMI or a Purchasing Managers’ Index (PMI) is an indicator of business activity in

(a) Manufacturing sector

(b) Services sector

(c) Both a and b

(d) None of the above

Ans: c

Exp: PMI or a Purchasing Managers’ Index (PMI) is an indicator of business activity -- both in the

manufacturing and services sectors. It is a survey-based measures that asks the respondents about

changes in their perception of some key business variables from the month before. It is calculated

separately for the manufacturing and services sectors and then a composite index is constructed.

https://www.thehindu.com/business/markets/manufacturing-pmi-remains-flat-at-514-in-

september/article29567701.ece

Q16. In India, in the overall Index of Industrial Production, the Indices of Eight Core Industries have

a combined weight of 37-90%. Which of the following are among those Eight Core Industries?

1. Cement

2. Fertilizers

3. Natural gas

4. Refinery products

5. Textiles

Select the correct answer using the codes given below :

(a) 1 and 5 only

(b) 2, 3 and 4 only

(c) 1, 2, 3 and 4 only

(d) 1, 2, 3, 4 and 5

Ans: c

Exp: The 8 core industries are their relative weight in IIP is as below:

Coal (weight: 4.38 %); Crude Oil (weight: 5.22 %); Natural Gas (weight: 1.71 %); Refinery

Products (weight: 5.94%); Fertilizers (weight: 1.25%); Steel (weight: 6.68%); Cement (weight:

2.41%); Electricity (weight: 10.32%)

https://www.thehindu.com/opinion/editorial/twin-troubles/article30300279.ece

https://t.me/UPSC_PDF Website ➡ https://upscpdf.com https://t.me/UPSC_PDF

Website ➡ https://upscpdf.com

Current Economics MCQs (Jan

2019 to Feb 20) + Eco Survey 2020

WriteToBeIAS.com Prelims 2020 Online Classes

by CA Rahul Kumar

Telegram https://t.me/writetobeias1 Youtube https://tinyurl.com/yx22y2yl Page 85

Q17. Which of the following are the objectives of National Broadband Mission

1. Develop a Broadband Readiness Index (BRI) to measure the availability of digital communications

infrastructure.

2. Broadband access to all villages by 2030

3. Develop innovative implementation models for Right of Way (RoW).

Select the correct answer using the code given below.

(a) 1 only

(b) 1 and 3 only

(c) 1 and 2 only

(d) 1, 2 and 3

Ans: b

Exp: Develop a Broadband Readiness Index (BRI) to measure the availability of digital

communications infrastructure and conducive policy ecosystem within a State/UT. Broadband access

to all villages by 2022 Develop innovative implementation models for Right of Way (RoW).

https://www.thehindu.com/news/national/govt-unveils-7-lakh-cr-national-broadband-

mission/article30332256.ece

Q18. ‘Project Zero’ recently seen in news is associated with:

(a) Land degradation and desertification

(b) Blocking the sale of counterfeit goods

(c) Decreasing carbon emissions

(d) Net neutrality

Ans: b

Exp: In a bid to ensure that customers receive authentic goods when shopping on Amazon, it has

announced to bring "Project Zero" to India. "Project Zero" introduces additional proactive

mechanisms and powerful tools to identify, block and remove counterfeits.

Source:https://www.livemint.com/industry/retail/amazon-launches-project-zero-in-india-to-block-

counterfeit-goods-11573582620572.html

Q19.With reference to the Mineral Laws (Amendment) Ordinance 2020, consider the following

statements:

1. Through the ordinance India has opened up the coal sector completely for commercial mining for

all local and global firms after easing restrictions on end-use and prior experience in auctions

2. It has amended the Coal Mines (Special Provisions) Act, 2015, as well as the Mines and Minerals

(Development and Regulation) Act, 1957.

Which of the statements given above is/are correct?

(a) 1 only

(b) Both 1 and 2

(c) 2 only

(d) Neither 1 nor 2

Ans: b

Exp: Union Cabinet has approved promulgation of Mineral Laws (Amendment) Ordinance 2020 to

amend the Coal Mines (Special Provisions) Act, 2015, as well as the Mines and Minerals

(Development and Regulation) Act, 1957. The amendment to the latter was required to begin auctions

of iron ore mining leases before they expire in March this year. The ordinance puts an end to captive

coal block auctions infuture and will have to be adopted in the upcoming Budget session once it is

cleared by the President. The government proposes to kick-start commercial coal mining auction

process with release of bid rules and consultations with stakeholders.

https://t.me/UPSC_PDF Website ➡ https://upscpdf.com https://t.me/UPSC_PDF

Website ➡ https://upscpdf.com

Current Economics MCQs (Jan

2019 to Feb 20) + Eco Survey 2020

WriteToBeIAS.com Prelims 2020 Online Classes

by CA Rahul Kumar

Telegram https://t.me/writetobeias1 Youtube https://tinyurl.com/yx22y2yl Page 86

Source:https://economictimes.indiatimes.com/industry/indl-goods/svs/metals-mining/govtapproves-

ordinance-to-auction-coal-mines/articleshow/73153778.cms?from=mdr

Q20. AGR (as per the definition of the Department of Telecom) often seen in news includes which

among the following:

1. Spectrum usage charges

2. Dividend

3. Interest on deposits

4. Profit on sale of assets

Which of the statements given above is/are correct?

(a) 1, 2 and 3 only

(b) 1, 2 and 4 only

(c) 1, 3 and 4 only

(d) 1, 2, 3 and 4

Ans: d

Exp: The Adjusted Gross Revenue (AGR) calculation is what the government and telecom majors

have had a disagreement over since 2005. The telecom companies argued that AGR should include

income only from telecom operations. The Department of Telecommmunications (DoT)- disagreed

and said it should also include non-telecom incomes such as the sale of assets, interest on deposits,

rents, etc. Telecom operators are required to pay licence fee and spectrum charges in the form of

‘revenue share’ to the Centre. The revenue amount used to calculate this revenue share is termed as

the AGR. According to the DoT, the calculations should incorporate all revenues earned by a telecom

company– including from non-telecom sources such as deposit interests and sale of assets. The

companies, however, have been of the view that AGR should comprise the revenues generated from

telecom services only and non-telecom revenues should be kept out of it.

Source:https://www.thehindubusinessline.com/opinion/columns/slate/all-you-wanted-to-know-

about-agr/article30008124.ece

https://indianexpress.com/article/explained/explained-what-is-agr-how-will-it-impact-airtel-

vodafone-idea-6086416/

Q21. Consider the following statements:

1. The recently setup National Start-up Advisory Council will be chaired by the Minster of Skill

Development and Entrepreneurship.

2. It aims to foster a culture of innovation amongst citizens and students in particular.

Which of the statements given above is/are correct?

(a) 1 only

(b) 2 only

(c) Both 1 and 2

(d) Neither 1 nor 2

Ans: b

Exp: Chairman of the National Start-up Advisory Council-Minister of Commerce & Industry. The

Council will suggest measures to foster a culture of innovation amongst citizens and students in

particular, promote innovation in all sectors of economy across the country, including semi-urban and

rural areas, support creative and innovative ideas through incubation and research and development

to transform them into valuable products, processes or solutions to improve productivity and

efficiency and create an environment of absorption of innovation in industry.

Source:https://pib.gov.in/newsite/PrintRelease.aspx?relid=197539

https://t.me/UPSC_PDF Website ➡ https://upscpdf.com https://t.me/UPSC_PDF

Website ➡ https://upscpdf.com

Current Economics MCQs (Jan

2019 to Feb 20) + Eco Survey 2020

WriteToBeIAS.com Prelims 2020 Online Classes

by CA Rahul Kumar

Telegram https://t.me/writetobeias1 Youtube https://tinyurl.com/yx22y2yl Page 87

https://www.livemint.com/news/india/govt-sets-up-national-startup-advisory-council-

11579614976654.html

Q22. Consider the following statements:

1. To recognize young people as critical drivers of sustainable development, Atal Innovation Mission

(AIM), NITI Aayog and United Nations Development Programme (UNDP) India launched Youth

Co: Lab.

2. It aims at accelerating social entrepreneurship and innovation in young India.

Which of the statements given above is/are correct?

(a) 1 only

(b) Both 1 and 2

(c) 2 only

(d) Neither 1 nor 2

Ans: b

Exp: In a latest initiative to recognize young people as critical drivers of sustainable development,

Atal Innovation Mission (AIM), NITI Aayog and United Nations Development Programme (UNDP)

India launched Youth Co:Lab which aims at accelerating social entrepreneurship and innovation in

young India. He first phase of Youth Co:Lab will focus on six SDGs: SDG 5(Gender

Equality),SDG6(Clean Water and Sanitation),SDG 7(Affordable and Clean Energy), SDG 8(Decent

Work and Economic Growth), SDG 12(Sustainable Consumption and Production) and SDG 13

(Climate Action). Youth Co:Lab will convene social innovation challenges at the national and sub-

national level,which will invite young people in the age group of 18-29 years and start-ups to show

case their proposed ideas and solutions to tackle some of the region’s biggest social challenges.

Source: https://pib.gov.in/newsite/PrintRelease.aspx?relid=193626

Q23. Consider the following statements:

1.Hyperloop is the name given to a technology where in vacuum is used to transport people very fast.

2.High speed in a hyperloop is achieved as the pods, which carry passengers, move using magnetic

levitation.

Which of the statements given above is/are correct?

(a)1 only

(b)2 only

(c)Both 1 and 2

(d)Neither 1 nor 2

Ans: c

Exp: In July 2012, entrepreneur Elon Musk, co-founder of Tesla, Inc and several other firms, unveiled

his vision of the new transport system, which he called a ‘Hyperloop’. Musk envisioned a transport

system which would never crash and would be immune to weather’s vagaries. It would also be thrice

or four times as fast as a bullet train, with an average speed that would be twice of an aircraft. With

this system, the travelling time between Los Angeles and San Francisco could be reduced to 30

minutes ( it is currently three hours in high-speed trains). The high speed is achieved as the pods,

which carry passengers, move using magnetic levitation. The speed increases further due to the near-

vacuum conditions inside the tubes, which reduce resistance to the pod as it travels within the tube.

Musk open-sourced this idea and called upon companies and individuals, with the right resources, to

take it forward.

Source:https://www.thehindu.com/business/richard-branson-to-meet-uddhav-thackeray-over-

mumbai-pune-hyperloop-project/article30277276.ece

https://t.me/UPSC_PDF Website ➡ https://upscpdf.com https://t.me/UPSC_PDF

Website ➡ https://upscpdf.com

Current Economics MCQs (Jan

2019 to Feb 20) + Eco Survey 2020

WriteToBeIAS.com Prelims 2020 Online Classes

by CA Rahul Kumar

Telegram https://t.me/writetobeias1 Youtube https://tinyurl.com/yx22y2yl Page 88

https://indianexpress.com/article/explained/explained-after-hype-over-hyperloop-why-theres-a-

question-mark-over-ultra-modern-project-6223864/

Q24. Consider the following statements:

1. Khanij Bidesh India Ltd. (KABIL) is to be set up with the participation of three Central Public

Sector Enterprises namely, National Aluminium Company Ltd. (NALCO), Hindustan Copper Ltd.

(HCL) and Mineral Exploration Company Ltd. (MECL).

2. KABIL is to ensure a consistent supply of critical and strategic minerals to Indian domestic and

international market.

Which of the above statements is/are correct?

(a) 1only

(b) 2only

(c) Both1and2

(d) Neither1nor2

Ans: a

Exp: A joint venture company namely Khanij Bidesh India Ltd. (KABIL) is to be set up with the

participation of three Central Public Sector Enterprises namely, National Aluminium Company Ltd.

(NALCO), Hindustan Copper Ltd. (HCL) and Mineral Exploration Company Ltd. (MECL). The

Minister of Coal, Mines and Parliamentary Affairs said that the objective of constituting KABIL is

to ensure a consistent supply of critical and strategic minerals to Indian domestic market. While

KABIL would ensure mineral security of the Nation, it would also help in realizing the overall

objective of import substitution, he said.

https://pib.gov.in/PressReleseDetail.aspx?PRID=1581058

Q25. In India the overall Index of Industrial Production, the Indices of Eighth Core Industries have

combined weight of 37.90 %.Which of the following are among those Eight Core Industries?

1. Cement

2. Fertilizers

3. Natural Gas

4. Refinery products

5. Textiles

Select the correct answer using the codes given below:

(a) 1 and 5only

(b) 2, 3 and 4only

(c) 1, 2, 3 and 4only

(d) 1, 2, 3, 4 and 5

Ans: c

Exp: Core industries can be defined as main or key industries of the economy. In most countries,

these industries are backbone of all other industries. In India, there are eight core sectors comprising

of coal, crude oil, natural gas, petroleum refinery products, fertilisers, steel, cement and electricity.

https://www.thehindubusinessline.com/economy/infrastructure-sector-grows-at-slowest-pace-in-

four-years/article28773648.ece

Q26. National Manufacturing Competitiveness Programme (NMCP) is implemented by the

(a) Ministry of Micro, Small and Medium Enterprises (MSMEs)

(b) Ministry of Corporate Affairs

(c) Ministry of Commerce and Industry

(d) Ministry of Heavy Industries and Enterprises

Ans. a

https://t.me/UPSC_PDF Website ➡ https://upscpdf.com https://t.me/UPSC_PDF

Website ➡ https://upscpdf.com

Current Economics MCQs (Jan

2019 to Feb 20) + Eco Survey 2020

WriteToBeIAS.com Prelims 2020 Online Classes

by CA Rahul Kumar

Telegram https://t.me/writetobeias1 Youtube https://tinyurl.com/yx22y2yl Page 89

Exp: National Manufacturing Competitiveness Programme (NMCP) aims to support MSMEs in

improving their competitiveness. Ministry of Micro, Small and Medium Enterprises (MSMEs)

implements Credit Linked Capital Subsidy-Technological Up-gradation Scheme (CLCS-TUS) to

support MSMEs in their technology up-gradation endeavours.

https://pib.gov.in/newsite/PrintRelease.aspx?relid=112836

Q27. Consider the following statements

1. In India, the GI tag is governed by the Geographical Indication of Goods (Registration and

Protection) Act which came into being in 1999.

2. A GI tag is valid for twenty years.

3. Jeeraphool rice from Chhattisgarh became the country’s first product to bag a GI tag.

4. Ministry of commerce’s department for promotion of industry and internal trade is responsible for

registering GIs in India.

Which of the statements given above is/are correct?

Select the correct answer using the code given below:

(a) 1, 2 and 4 only

(b) 1and 4 only

(c) 2, 3 and 4 only

(d) 1, 2, 3 and 4

Ans. b

Exp: The tag is valid for a decade, after which it can be renewed for another 10 years. Darjeeling tea

became the country’s first product to bag a GI tag.

Source:https://www.thehindubusinessline.com/blink/know/india-plays-gi-tag/article29467112.ece#

Q28. National Productivity Council (NPC) comes under the

(a) Ministry of Skill Development and Entrepreneurship

(b) NITI Aayog

(c) Department for Promotion of Industry and Internal Trade

(d) Ministry of Micro, Small and Medium Enterprises

Ans. c

Source:https://dipp.gov.in/about-us/autonomous-institutionss/national-productivity-council

Q29. Consider the following:

1. Small start-ups with turn over upto Rs.2.5 crore will get the promised tax holiday.

2. They will be eligible for deduction of 100 percent of income of a negligible start-up for 3 years out

of 7 years from the year of its incorporation.

Which of the statements given above is/are correct?

(a) 1 only

(b) 2 only

(c) Both 1 and 2

(d) Neither 1 nor 2

Ans: b

Exp: Small start-ups with turn over upto Rs.25 crore will get the promised tax holiday. They will be

eligible for deduction of 100 percent of income of an eligible start-up for 3 years out of 7 years from

the year of its incorporation.

https://pib.gov.in/PressReleseDetail.aspx?PRID=1582654

https://t.me/UPSC_PDF Website ➡ https://upscpdf.com https://t.me/UPSC_PDF

Website ➡ https://upscpdf.com

Current Economics MCQs (Jan

2019 to Feb 20) + Eco Survey 2020

WriteToBeIAS.com Prelims 2020 Online Classes

by CA Rahul Kumar

Telegram https://t.me/writetobeias1 Youtube https://tinyurl.com/yx22y2yl Page 90

AGRICULTURE

Q1. With reference to National Agriculture Market (e-NAM), consider the following

1. It is an online trading portal that was launched by the Government of India on 14th April, 2015.

2. It aims to create an online transparent competitive price discovery system to facilitate farmers with

remunerative prices for their produce.

3. Small Farmers Agribusiness Consortium (SFAC) is the lead agency for implementing eNAM.

Which of the statements given above is/are correct?

Select the correct answer using the code given below:

(a) 2 only

(b) 1 and 2

(c) 1 and 3

(d) 2 and 3

Ans. d

Exp: It is an online trading portal that was launched on 14th April, 2016. Small Farmers Agribusiness

Consortium (SFAC) is the lead agency for implementing eNAM under the aegis of Ministry of

Agriculture and Farmers’ Welfare.

Source: https://enam.gov.in/web/

Q2. Consider the following

1. Ministry of Agriculture and Farmers Welfare has distributed more than one lakh bee-boxes among

farmers and unemployed youths across the country under its 'Honey Mission' initiative.

2. 'Honey Mission' was launched in August 2017.

3. Honey exports from India grew 19% year-on-year in 2018-19 to $105 million and 13% in the

previous fiscal to $101 million amid increasing demand.

4. Punjab, West Bengal, Uttar Pradesh and Bihar are among leading honey producing states in India.

Which of the statements given above is/are correct?

Select the correct answer using the code given below:

(a) 1and 4 only

(b) 1, 2 and 3

(c) 2, 3 and 4

(d) 4 only

Ans. c

Exp: Khadi and Village Industries Commission (KVIC), under the Ministry of Micro, Small

and Medium Enterprises has distributed more than one lakh bee-boxes among farmers and

unemployed youths across the country in less than two years under its 'Honey Mission' initiative.

Source: https://www.business-standard.com/article/pti-stories/kvic-distributes-over-1-lakh-bee-

boxes-under-honey-mission-119050601239_1.html

https://economictimes.indiatimes.com/news/economy/foreign-trade/honey-may-turn-steeter-for-

indian-exporters/articleshow/70804913.cms?from=mdr

Q3.Consider the following statements

Under Kisan Credit Card (KCC) facility,

1. Fisheries and animal husbandry farmers are also included

2. Interest subvention is available to eligible farmers @ 2% per annum at the time of disbursal of loan

and additional interest subvention @ 4% per annum in case of prompt repayment

Which of the following statements given above is/are correct?

(a) 1 only

(b) 2 only

https://t.me/UPSC_PDF Website ➡ https://upscpdf.com https://t.me/UPSC_PDF

Website ➡ https://upscpdf.com

Current Economics MCQs (Jan

2019 to Feb 20) + Eco Survey 2020

WriteToBeIAS.com Prelims 2020 Online Classes

by CA Rahul Kumar

Telegram https://t.me/writetobeias1 Youtube https://tinyurl.com/yx22y2yl Page 91

(c) Both 1 and 2

(d) Neither 1 nor 2

Ans. a

Exp:Under Kisan Credit Card (KCC) facility, interest subvention is available @ 2% per annum at

the time of disbursal of loan and additional interest subvention @ 3% per annum in case of prompt

repayment as Prompt Repayment Incentive.

Source:https://pib.gov.in/PressReleaseIframePage.aspx?PRID=1576660

Q4. “Crop Cutting Experiments (CCEs)” recently seen in news is related to

(a) Technology to cut crop in cost effective manner

(b) Accurately estimate the yield of a crop or region during a given cultivation cycle

(c) Used for transgenic crops

(d) None of the above

Ans: b

Exp: Crop Cutting Experiments or CCE, refer to an assessment method employed by governments

and agricultural bodies to accurately estimate the yield of a crop or region during a given cultivation

cycle. The traditional method of CCE is based on the yield component method where sample locations

are selected based on a random sampling of the total area under study.

https://pib.gov.in/PressReleseDetail.aspx?PRID=1575248

Q5. Which of the following brings out the ‘Advance Estimates of production of major crops?

(a) Ministry of Food Processing Industries

(b) Department of Agriculture, Cooperation and Farmers Welfare

(c) Department of Consumer Affairs

(d) None of the above

Ans: b

Exp: The 3rd Advance Estimates of production of major crops for 2018-19 have been released by the

Department of Agriculture, Cooperation and Farmers Welfare on 03rdJune, 2019. The assessment of

production of different crops is based on the feedback received from States and validated with

information available from other sources

https://pib.gov.in/PressReleseDetail.aspx?PRID=1573283

Q6. Consider following statements:

1. FCI is a nodal Agency for procurement of Pulses and Oilseeds.

2. The Central Government extends price support for only procurement of wheat and paddy through

the FCI and State Agencies.

Which of the statements given above is/are correct?

(a) 1 only

(b) 2 only

(c) Both 1 and 2

(d) Neither 1 nor 2

Ans: a

Exp: The Central Government extends price support for procurement of wheat, paddy and coarse

grains through the FCI and State Agencies. All the food grains conforming to the prescribed

specifications are procured by the public procurement agencies at the Minimum Support Price (MSP)

plus incentive bonus announced, if any. As decided by the CCEA, FCI has also been nominated as

an additional nodal Agency for procurement of Pulses and Oilseeds. The Department of Agriculture

and Co-0operation is drawing up guidelines /MoU for FCI to procure the said items.

https://pib.gov.in/PressReleseDetail.aspx?PRID=1573682

https://t.me/UPSC_PDF Website ➡ https://upscpdf.com https://t.me/UPSC_PDF

Website ➡ https://upscpdf.com

Current Economics MCQs (Jan

2019 to Feb 20) + Eco Survey 2020

WriteToBeIAS.com Prelims 2020 Online Classes

by CA Rahul Kumar

Telegram https://t.me/writetobeias1 Youtube https://tinyurl.com/yx22y2yl Page 92

Q7.Consider the following:

1. Nitrogen

2. Phosphorus

3. Potash

4. Urea

5. Sulphur

For which of the above Department of Fertilizers fixes Nutrient Based Subsidy Rates?

(a) 1, 2, 3 and 4

(b) 1, 2, 3 and 5

(c) 1, 2, 4 and 5

(d) 1, 2, 3, 4 and 5

Ans: b

Exp: The Cabinet Committee on Economic Affairs, chaired by Prime Minister Shri Narendra Modi

has approved the proposal of the Department of Fertilizers for fixation of Nutrient Based Subsidy

Rates for P&K Fertilizers for the year 2019-20. The approved rates for NBS effective from the date

of notification will be as under:

Per Kg Subsidy rates (in Rs.)

N(Nitrogen) P(Phosphorus) K(Potash) S(Sulphur)

18.901 15.216 11.124 3.562

https://www.thehindubusinessline.com/economy/nutrient-based-subsidy-rate-for-suphur-hiked-by-

31-per-cent/article28773421.ece

Q8. With reference to “PM Kisan Maan Dhan Yojana” consider the following:

1. The scheme is voluntary and contributory for farmers in the entry age group of 18 to 40 years.

2. Monthly pension of Rs.5000/- will be provided to on attaining the age of 60 years.

3. The farmers will have to make a monthly contribution of Rs.55 to Rs.200, depending on their age

of entry.

Select the correct answer using the codes given below:

(a) 1 and 2 only

(b) 1 and 3 only

(c) 1only

(d) 1, 2 and 3

Ans: b

Exp: The scheme is voluntary and contributory for farmers in the entry age group of 18 to 40 years

and a monthly pension of Rs. 3000/- will be provided to them on attaining the age of 60 years. The

farmers will have to make a monthly contribution of Rs. 55 to Rs. 200 , depending on their age of

entry, in the Pension Fund till they reach the retirement date i.e. the age of 60 years. The Central

Government will also make an equal contribution of the same amount in the pension fund. The spouse

is also eligible to get a separate pension of Rs.3000/-upon making separate contributions to the Fund.

The Life Insurance Corporation of India (LIC) shall be the Pension Fund Manager and responsible

for Pension payout.

https://pib.gov.in/PressReleseDetail.aspx?PRID=1581638

Q9. With reference to Price stabilization fund consider the following

https://t.me/UPSC_PDF Website ➡ https://upscpdf.com https://t.me/UPSC_PDF

Website ➡ https://upscpdf.com

Current Economics MCQs (Jan

2019 to Feb 20) + Eco Survey 2020

WriteToBeIAS.com Prelims 2020 Online Classes

by CA Rahul Kumar

Telegram https://t.me/writetobeias1 Youtube https://tinyurl.com/yx22y2yl Page 93

1. It is to promote direct purchase from farmers/farmers’ associations at farm gate / Mandi.

2. Safal will act as Fund Manager.

3. It will act to maintain a strategic bufferstock that would discourage hoarding and unscrupulous

speculation.

Select the correct answer using the codes given below:

(a) 1 only

(b) 2 and 3 only

(c) 1 and 3 only

(d) 1, 2 and 3

Ans: c

Exp: It is to promote direct purchase from farmers/ farmers’ associations at farm gate / Mandi. Small

Farmers Agri- Business Consortium (SFAC), will act as Fund Manager. It will act to maintain a

strategic buffer stock that would discourage hoarding and unscrupulous speculation.

https://pib.gov.in/PressReleseDetail.aspx?PRID=1582500

Q10. Consider following statements:

1. PM KISAN is a Central Sector scheme with 50% funding from Government of India.

2. An income support of Rs.6000/- per year in three equal instalments will be provided to small and

marginal farmer families having combined land holding/ownership of upto 2 hectares.

3. Definition of family for the scheme is husband, wife and minor children.

Which of the statements given above is/are correct?

(a) 1 and 2 only

(b) 2 and 3 only

(c) 1 and 3 only

(d) 1, 2 and 3

Ans: b

Exp: PM KISAN is a Central Sector scheme with 100% funding from Government of India. An

income support of Rs.6000/- per year in three equal instalments will be provided to small and

marginal farmer families having combined land holding/ownership of upto 2 hectares. Definition of

family for the scheme is husband, wife and minor children.

Q11. Consider the following:

1. All farmers – individuals/Joint cultivator owners

2. Tenant farmers, oral lessees and share croppers

3. SHGs or Joint liability groups including tenant farmers

Which of the above is/are eligible for Kisan Credit Card?

(a) 1 and 2 only

(b) 2 and 3 only

(c) 1 and 3 only

(d) 1, 2 and 3

Ans: d

Exp: Eligibility- All farmers – individuals/Joint cultivator owners. Tenant farmers, oral lessees and

share croppers etc. SHGs or Joint liability groups including tenant farmers.

http://yojana.gov.in/19-3-19%20%20March%20Yojana%20Final.pdf

http://yojana.gov.in/19-3-19%20%20March%20Yojana%20Final.pdf

Q12. Consider following statements:

1. India is one of the first countries in the world to develop a comprehensive Cooling Action plan.

2. Its aim is to reduce cooling energy requirements by 25% to 40% by 2037-38.

https://t.me/UPSC_PDF Website ➡ https://upscpdf.com https://t.me/UPSC_PDF

Website ➡ https://upscpdf.com

Current Economics MCQs (Jan

2019 to Feb 20) + Eco Survey 2020

WriteToBeIAS.com Prelims 2020 Online Classes

by CA Rahul Kumar

Telegram https://t.me/writetobeias1 Youtube https://tinyurl.com/yx22y2yl Page 94

3. It will help in Doubling Farmers Income.

Which of the statements given above is/are correct?

(a) 1 and 2 only

(b) 2 and 3 only

(c) 1 and 3 only

(d) 1, 2 and 3

Ans: d

Exp: India is one of the first countries in the world to develop a comprehensive Cooling Action plan

which has a long term vision to address the cooling requirement across sectors and lists out actions

which can help reduce the cooling demand. Cooling requirement is cross sectoral and an essential

part for economic growth and is required across different sectors of the economy such as residential

and commercial buildings, cold-chain, refrigeration, transport and industries The following benefits

would accrue to the society over and above the environmental benefits: (i) Thermal comfort for all –

provision for cooling for EWS and LIG housing, (ii) Sustainable cooling – low GHG emissions

related to cooling, (iii) Doubling Farmers Income – better cold chain infrastructure – better value of

produce to farmers, less wastage of produce, (iv) Skilled workforce for better livelihoods and

environmental protection, (v) Make in India – domestic manufacturing of air-conditioning and related

cooling equipment’s, (vi) Robust R&D on alternative cooling technologies – to provide push to

innovation in cooling sector.

http://www.pib.nic.in/PressReleseDetail.aspx?PRID=1568328

Q13. ‘Transport and Marketing Assistance (TMA)’ Scheme is related to

(a) Export under Make in India

(b) International component of freight and marketing of agricultural produce

(c) Export for Coir Industry

(d) None of the above

Ans: b

Exp: The “Transport and Marketing Assistance” (TMA) for specified agriculture products scheme

aims to provide assistance for the international component of freight and marketing of agricultural

produce which is likely to mitigate disadvantage of higher cost of transportation of export of specified

agriculture products due to trans-shipment and to promote brand recognition for Indian agricultural

products in the specified overseas markets.

http://www.pib.nic.in/PressReleseDetail.aspx?PRID=1567543

Q14. Consider following statements about Pradhan Mantri Kisan Samman Nidhi (PM-KISAN):

1. Under this programme, vulnerable landholding farmer families, having cultivable land up to 2

hectares, will be provided direct income support.

2. This programme will be funded by State Governments and Government of India in the ratio of

50:50.

Which of the statements given above is/are correct?

(a) 1 only

(b) 2 only

(c) Both 1 and 2

(d) Neither 1 nor 2

Ans: a

Exp: This programme will be funded by Government of India. Under this programme, vulnerable

landholding farmer families, having cultivable land upto 2 hectares, will be provided direct income

support at the rate of Rs. 6,000 per year. This income support will be transferred directly into the bank

accounts of beneficiary farmers, in three equal instalments of Rs. 2,000 each. This programme will

https://t.me/UPSC_PDF Website ➡ https://upscpdf.com https://t.me/UPSC_PDF

Website ➡ https://upscpdf.com

Current Economics MCQs (Jan

2019 to Feb 20) + Eco Survey 2020

WriteToBeIAS.com Prelims 2020 Online Classes

by CA Rahul Kumar

Telegram https://t.me/writetobeias1 Youtube https://tinyurl.com/yx22y2yl Page 95

be funded by Government of India. Around 12 crore small and marginal farmer families are expected

to benefit from this. The programme would be made effective from 1st December 2018 and the first

installment for the period upto 31st March 2019 would be paid during this year itself. This programme

will entail an annual expenditure of Rs.75,000 crore”

http://www.pib.nic.in/PressReleseDetail.aspx?PRID=1562184

Q15. Consider following statements w.r.t. “Pradhan Mantri Kisan Samman Nidhi (PM-KISAN):

1. Under the scheme PM-Kissan only Marginal farmers (SMF) families having cultivable land upto

one hectare will be provided income support.

2. The income support of Rs.6000/- per year will be provided through account payee cheque in six

equal instalments.

Which of the statements given above is/are correct?

(a) 1 only

(b) 2 only

(c) Both 1 and 2

(d) Neither 1 nor 2

Ans: d

Exp: Under the scheme PM-Kissan all Small and Marginal farmers (SMF) families having cultivable

land upto two hectares will be provided income support of Rs.6000/- per year directly into their bank

accounts, in three equal instalments.

http://www.pib.nic.in/PressReleseDetail.aspx?PRID=1563259

Q16.Consider following statements w.r.t. agriculture Census:

1. Farmers having below 2-hectares land are categorised as Marginal Farmers.

2. Farmers having more than 5-hectares land are categorised as Large Farmers.

Which of the statements given above is/are correct?

(a) 1 only

(b) 2 only

(c) Both 1 and 2

(d) Neither 1 nor 2

Ans: a

Exp:

Q17. Vaikunth Mehta National Institute of Cooperative Management (VAMNICOM) is under

administrative control of :-

(a) Ministry of Human resource development

(b) Ministry of Agriculture

(c) Ministry of Rural development

(d) Ministry of finance

Ans: b

Exp: Option b is correct.

http://pib.nic.in/PressReleseDetail.aspx?PRID=1560001

Q18. Consider the following statements w.r.t. Electronic National Agriculture Market (e-NAM):

1. It is funded by the Central Government and State Governments in the ratio of 50:50.

2. It creates a national network of major mandis which can be accessed physically.

Which statement/statements given above is/are correct?

Select the correct answer using the codes given below.

(a) 1 only

https://t.me/UPSC_PDF Website ➡ https://upscpdf.com https://t.me/UPSC_PDF

Website ➡ https://upscpdf.com

Current Economics MCQs (Jan

2019 to Feb 20) + Eco Survey 2020

WriteToBeIAS.com Prelims 2020 Online Classes

by CA Rahul Kumar

Telegram https://t.me/writetobeias1 Youtube https://tinyurl.com/yx22y2yl Page 96

(b) 2 only

(c) Both 1 and 2

(d) None

Ans: d

Exp: Electronic National Agriculture Market (e-NAM) is a Pan-India electronic trading portal

launched in 2016 completely funded by the Central Government and implemented by SmallFarmers’;

Agribusiness Consortium (SFAC). It creates a national network of physical mandis which can be

accessed online thus enabling buyers, situated even outside the State, to participate in trading at the

local level.

http://yojana.gov.in/Yojana%20January2019.pdf

Q19.Which of the following are mechanisms of agriculture credit in India

(a)Priority sector lending

(b) interest subvention scheme

(c) kisan credit scheme

(d)all

Ans: d

Exp: simplequestion

Priority Sector Lending: PSL was introduced to ensure that vulnerable sections of the society get

access to credit and that there is an adequate flow of credit to employment intensive sectors like

agriculture and MSME.

Interest Subvention Scheme (ISS) was launched for short term crop loans in 2006-07. 2 % interest

subvention is given to farmers, which is reimbursed to banks (through RBI and NABARD).

Kisan Credit Card (KCC) Scheme, introduced in 1998, aimed at providing adequate and timely credit

with flexible and simplified procedure for agriculture related and also consumption requirements of

farmer households

https://rbidocs.rbi.org.in/rdocs/Content/PDFs/84140.pdf

Q20. Which of the following can be the impact of farm loan waivers?

1. can boost agricultural productivity in short-term

2. can impact banking sector balance sheet

3. can decrease credit to agriculture R&D

(a) all

(b) 1 and 2

(c) 2 and 3

(d) 1 and 3

Ans: a

Exp: Economic rationale for loan waivers comes from alleviating debt over hang of beneficiaries thus

enabling them to undertake productive investment and boost real economic activity (investment,

production and consumption). Creates moral hazardas loan waivers lead to poor loan performance

and destroy honest credit culture, as it incentivizes borrowers to default strategically in anticipation

of future bailouts. Waivers skew credit allocation as banks reallocate lending to lower risk borrower

segments. Frequent write-offs impact financial health of banking sector. Prompts banks to invest in

Rural Infrastructure Development Fund rather than reaching out to cultivator to meet their PSL

targets, reducing credit access.

Q21.Operation Greens taken by government aims to control prices of

(a) Tomato, onion, potato

(b) tomato, pulses, potato

https://t.me/UPSC_PDF Website ➡ https://upscpdf.com https://t.me/UPSC_PDF

Website ➡ https://upscpdf.com

Current Economics MCQs (Jan

2019 to Feb 20) + Eco Survey 2020

WriteToBeIAS.com Prelims 2020 Online Classes

by CA Rahul Kumar

Telegram https://t.me/writetobeias1 Youtube https://tinyurl.com/yx22y2yl Page 97

(c) onion, rice, potato

(d) onion, pulses, potato

Ans: a

Exp: Operation Greens: For enhancing production & reducing price volatility of fruits & vegetables

(initially for tomatoes, onions and potatoes). It will also promote Farmer Producers Organisations

(FPOs), agri-logistics, processing facilities, thus to achieve the aim of doubling the farmer Income

2022. https://www.indiatoday.in/business/story/onion-price-delhi-mumbai-onion-price-rise-causes-

1602489-2019-09-24

Q22. Consider the correct statement about Pradhan Mantri Kisan Mandhan Yojana.

(a) It is an old age pension scheme for all farmers

(b) It is an old age pension scheme for land holding small and marginal farmers

(c) It provides financial assistance of 6000 rs/annum for all farmers

(d) It provides financial assistance of 6000 rs/annum for land holding small and marginal farmers.

Ans: b

Exp: PM-KMY is an old age pension scheme for all and holding Small and Marginal Farmers (SMFs)

in the country with a view to provide social security net as they have minimal or no savings to provide

for old age and to support them in the event of consequent loss of livelihood.

Salient features: It is a voluntary and contribution-based pension scheme for farmers in the entry age

group of 18 to 40 years and a monthly pension of Rs. 3000 will be provided to them on attaining the

age of 60 years.

Financial assistance of 6000 rs/annum is provided under PM KISAN scheme.

https://pib.gov.in/newsite/PrintRelease.aspx?relid=193160

Q23.Consider the following statements regarding Pradhan Mantri Kisan Samman Nidhi (PM

KISAN).

1. PM KISAN is a Central Sector scheme with 100 % funding from Government of India

2. Under this programme, vulnerable and holding farmer families, having cultivable land upto 2 acres,

will be provided direct income support at the rate of Rs. 6,000 per year.

3. Definition of family for the scheme is husband, wife and minor children.

4. All Institutional Land holders are eligible beneficiaries.

Which of the above statements is/are correct?

(a) 1, 2, 3

(b) 1, 3

(c) 2, 3, 4

(d)1, 3, 4

Ans: b

Exp: PM KISAN is a Central Sector scheme with 100 % funding from Government of India. It has

become operational from 1.12.2018. Under the scheme an income support of Rs.6000/-per year in

three equal instalments will be provided to small and marginal farmer families having combined land

holding/ ownership of upto 2 hectares. Definition of family for the scheme is husband, wife and minor

children. State Government and UT Administration will identify the farmer families which are

eligible for support as per scheme guidelines. The fund will be directly transferred to the bank

accounts of the beneficiaries. All Institutional Landholders shall not be eligible for benefit under the

scheme.

https://www.pmkisan.gov.in/

Q24.Consider the following statements:

1. Cotton is a drought resistant crop ideal for arid climates.

https://t.me/UPSC_PDF Website ➡ https://upscpdf.com https://t.me/UPSC_PDF

Website ➡ https://upscpdf.com

Current Economics MCQs (Jan

2019 to Feb 20) + Eco Survey 2020

WriteToBeIAS.com Prelims 2020 Online Classes

by CA Rahul Kumar

Telegram https://t.me/writetobeias1 Youtube https://tinyurl.com/yx22y2yl Page 98

2. India under Cotton Technical Assistance Programme (TAP) for Africa cover 41 African countries.

Which of the above statements is/are correct?

(a) 1 only

(b) 2 only

(c) Both 1 and 2

(d) Neither 1 nor 2

Ans: a

Exp: Cotton is a drought–resistant crop ideal for arid climates. India will launch the second phase of

the Cotton Technical Assistance Programme (TAP) for Africa. In the five year long second phase,

the programme will be scaled up in size and coverage and will be introduced in five additional

countries, namely Mali, Ghana, Togo, Zambia and Tanzania. The Cotton TAP programme will now

cover 11 African countries including the C4 (Benin, Burkina Faso, Chad and Mali).

https://pib.gov.in/newsite/PrintRelease.aspx?relid=196149

Q25. Consider the following statements relating to Objectives of “Operation Greens”:

1. Price stabilisation for producers and consumers by proper production planning in the Tomato,

Onion and Potato (TOP) clusters.

2. Reduction in post-harvest losses by creation of farm gate infrastructure and creation of appropriate

storage capacity linking consumption centres.

3. Setting up of a market intelligence network to collect and collate real time data on demand and

supply and price of TOP crops.

Which of the above statements is/are correct?

(a) 2 and 3only

(b) 2 only

(c) 1 and 3 only

(d) 1, 2 and 3

Ans: d

Exp: Price stabilisation for producers and consumers by proper production planning in the Tomato,

Onion and Potato (TOP) clusters. Reduction in post-harvest losses by creation of farm gate

infrastructure and creation of appropriate storage capacity linking consumption centres. Setting up of

a market intelligence network to collect and collate real time data on demand and supply and price of

TOP crops.

https://pib.gov.in/PressReleseDetail.aspx?PRID=1587688

Q26.Consider following statements:

1. A new ration card will be required for availing the portability benefit under the government's

ambitious' one nation, one ration card' initiative

2. Under the initiative, eligible beneficiaries would be able to avail their entitled food grain under the

National Food Security Act (NFSA) from any Fair Price Shop in the country

Which of the statements given above is/are correct?

(a) 1 only

(b) 2 only

(c) Both 1 and 2

(d) Neither 1 nor 2

Ans: b

Exp: No new ration card will be required for availing the portability benefit under the government's

ambitious' one nation, one ration card' initiative. Under the initiative, eligible beneficiaries would be

able to avail their entitled foodgrain under the National Food Security Act (NFSA) from any Fair

Price Shop in the country

https://t.me/UPSC_PDF Website ➡ https://upscpdf.com https://t.me/UPSC_PDF

Website ➡ https://upscpdf.com

Current Economics MCQs (Jan

2019 to Feb 20) + Eco Survey 2020

WriteToBeIAS.com Prelims 2020 Online Classes

by CA Rahul Kumar

Telegram https://t.me/writetobeias1 Youtube https://tinyurl.com/yx22y2yl Page 99

https://www.thehindu.com/news/national/inter-state-portability-for-ration-cards-

launched/article29577579.ece

Q27. Consider the following statements:

1. Under Aadhaar enabled Fertilizers Distribution System (AeFDS), it is mandatory for the retailers

to sell subsidized fertilizers through PoS devices.

2. The Fertilizer sales are not monitored online .

Which of the above statements is/are correct?

(a) 1 only

(b) 2 only

(c) Both 1 and 2

(d) Neither 1 nor 2

Ans: a

Exp: Under Aadhaar enabled Fertilizers Distribution System (AeFDS), it is mandatory for the

retailers to sell subsidized fertilizers through PoS devices. The Fertilizer sales are monitored online

through web based Integrated Fertilizer Management System (iFMS). A study conducted by NITI

Aayog through M/s Microsave revealed that AeFDS has increased accountability of stakeholders

including fertilizer manufacturers, wholesalers and retailers.

The NITI Aayog study in the pilot districts revealed that diversion of fertilizers has reduced. However,

the extent of diversion of fertilizers for non-agriculture activities has not been quantified.

https://pib.gov.in/PressReleseDetail.aspx?PRID=1594672

Q28. Consider the following statements

1. India is self-sufficient in terms of fertilizer production and we do not Import fertilizers.

2. The MRP of urea is statutorily fixed by the Government.

Which of the above statements is/are correct?

(a) 1 only

(b) 2 only

(c) Both 1 and 2

(d) Neither 1 nor 2

Ans: b

Exp: The gap between demand (requirement) and indigenous production of fertilizers is met through

imports. The import for the season is also finalized well in advance to ensure timely availability.The

MRP of urea is statutorily fixed by the Government.

https://pib.gov.in/PressReleseDetail.aspx?PRID=1595326

Q29. What is/are the advantage/advantages of implementing the ‘National Agriculture Market’

scheme?

1. It is a pan-India electronic trading portal for agricultural commodities.

2. It provides the farmers access to nationwide market, with prices commensurate with the quality of

their produce.

Which of the above statements is/are correct?

(a) 1 only

(b) 2 only

(c) Both 1 and 2

(d) Neither 1 nor 2

Ans: c

Exp: e-NAM, which aims at integrating mandis on a virtual platform, is a reform linked scheme.

States/ Union Territories (UTs) who have carried out three mandatory reforms namely (i) single

https://t.me/UPSC_PDF Website ➡ https://upscpdf.com https://t.me/UPSC_PDF

Website ➡ https://upscpdf.com

Current Economics MCQs (Jan

2019 to Feb 20) + Eco Survey 2020

WriteToBeIAS.com Prelims 2020 Online Classes

by CA Rahul Kumar

Telegram https://t.me/writetobeias1 Youtube https://tinyurl.com/yx22y2yl Page 100

trading license to be valid across the State (ii) single point levy of market fee across the State and (iii)

provision for e-auction / e-trading as a mode of price discovery; are eligible for integrating their

wholesale regulated markets with e-NAM platform. Based on the proposals received from requisite

reformed States/UTs, the wholesale regulated markets are approved and integrated with e-NAM

platform.

https://pib.gov.in/PressReleseDetail.aspx?PRID=1596396

Q30. The economic cost of food grains to the Food Corporation of India is Minimum Support Price

and bonus (if any) paid to the farmers plus

(a) transportation cost only

(b) interest cost only

(c) procurement incidentals and distribution cost

(d) procurement incidentals and charges for godowns

Ans: c

Exp: The economic cost of food grains to the Food Corporation of India is Minimum Support Price

and bonus (if any) paid to the farmers plus procurement incidentals and distribution cost.

https://www.kumarbookcentre.com/Magazines---Current-Affairs/Yojana-December-2019-id-

4114886.html

Q31. PM-AASHA scheme is related to

(a) ensuring remunerative prices to farmers for their produce

(b) community health worker

(c) women’s empowerment scheme

(d) free LPG scheme for women

Ans: a

Exp: PM-AASHA’s main feature was a price support scheme whereby Central agencies would

procure pulses and oilseeds directly from farmers.

https://www.thehindu.com/business/agri-business/government-procured-just-3-of-pulses-seeds-

targeted-under-pm-aasha-scheme/article30169667.ece

Q32. Which one of the following best describes the main objective of ‘Seed Village Concept?

(a) Encouraging the farmers to use their own farm seeds and discouraging them to buy the seeds from

others

(b) Involving the farmers for training in quality seed production and thereby to make available quality

seeds to others at appropriate time and affordable cost

(c) Earmarking some villages exclusively for the production of certified seeds

(d) Identifying the entrepreneurs in village and providing them technology and finance to set up seed

companies Solution

Ans: b

Exp: A village, where in trained group of farmers are involved in production of seeds of various crops

and cater to their own needs, fellow farmers of the village and farmers of the neighbouring villages

in appropriate time and at an affordable cost is called “Seed Village”.

https://www.thehindu.com/opinion/lead/a-potential-seedbed-for-private-profits/article30195634.ece

Q33. Under the Pradhan Mantri Kisan SAMPADA Yojana, the following schemes will be

implemented:

1. Setting up of SEZ’s and mega food parks

2. Integrated Cold Chain and Value Addition Infrastructure

3. Food Safety and Quality Assurance Infrastructure

https://t.me/UPSC_PDF Website ➡ https://upscpdf.com https://t.me/UPSC_PDF

Website ➡ https://upscpdf.com

Current Economics MCQs (Jan

2019 to Feb 20) + Eco Survey 2020

WriteToBeIAS.com Prelims 2020 Online Classes

by CA Rahul Kumar

Telegram https://t.me/writetobeias1 Youtube https://tinyurl.com/yx22y2yl Page 101

Which of the statements given above is/are correct?

(a) 1 and 3

(b) 1 and 2 only

(c) 2 and 3 only

(d) 1 ,2 and 3

Ans: c

Exp: Pradhan Mantri Kisan SAMPADA Yojana (Scheme for Agro-Marine Processing and

Development of Agro-Processing Clusters) will be implemented by Ministry of Food Processing

Industries (MoFPI). Under PMKSY following schemes are implemented: Mega Food Parks;

Integrated Cold Chain and Value Addition Infrastructure; Creation/ Expansion of Food Processing &

Preservation Capacities; Infrastructure for Agro-processing Clusters;

Creation of Backward and Forward Linkages; Food Safety and Quality Assurance Infrastructure; and

Human Resources and Institutions.

Source:https://pib.gov.in/PressReleasePage.aspx?PRID=1575745

Q34. Consider the following statements:

1. Zero Budget Natural Farming (ZBNF), which is a set of farming methods, and also a grassroots

peasant movement, has spread to various states in India.

2. Four pillars of ZNBF are Jivamrita, Bijamrita, Acchadana and Whapasa.

Which of the statements given above is/are correct?

(a) 1 only

(b) 2 only

(c) Both 1 and 2

(d) None

Ans: c

Exp: Zero Budget Natural Farming (ZBNF), which is a set of farming methods, and also a grassroots

peasant movement, has spread to various states in India. It has attained wide success in southern India,

especially the southern Indian state of Karnataka where it first evolved. The four pillars of ZNBF are

Jivamrita/jeevamrutha, Bijamrita/beejamrutha, Acchadana–Mulching and Whapasa–moisture.

Source:http://www.fao.org/3/a-bl990e.pdf

https://www.thehindubusinessline.com/specials/india-file/ground-results-validate-zero-budget-

natural-farming-subhash-palekar/article29723018.ece

Q35. With reference to the Agricultural and Processed Food Products Export Development Authority

(APEDA), consider the following statements:

1. It was established by the Government of India under the Agricultural and Processed Food Products

Export Development Authority Act passed by the Parliament in December, 1985.

2. It replaced the Processed Food Export Promotion Council (PFEPC).

Which of the statements given above is/are correct?

(a) 1 only

(b) 2 only

(c) Both 1 and 2

(d) Neither 1 nor 2

Ans: c

Exp: The Agricultural and Processed Food Products Export Development Authority (APEDA) was

established by the Government of India under the Agricultural and Processed Food Products Export

Development Authority Act passed by the Parliament in December, 1985. The Act (2 of 1986) came

into effect from 13th February, 1986 by a notification issued in the Gazette of India: Extraordinary:

https://t.me/UPSC_PDF Website ➡ https://upscpdf.com https://t.me/UPSC_PDF

Website ➡ https://upscpdf.com

Current Economics MCQs (Jan

2019 to Feb 20) + Eco Survey 2020

WriteToBeIAS.com Prelims 2020 Online Classes

by CA Rahul Kumar

Telegram https://t.me/writetobeias1 Youtube https://tinyurl.com/yx22y2yl Page 102

Part-II [Sec. 3(ii): 13.2.1986). The Authority replaced the Processed Food Export Promotion Council

(PFEPC).

http://apeda.gov.in/apedawebsite/about_apeda/about_apeda.htm

Q36. Consider the following statements

1. The share of agriculture and allied sectors in the total GVA of the country has been declining.

2. The share of agriculture and allied sectors in its contribution to national income has gradually

increased.

Select the correct code.

a. 1 only

b. 2 only

c. Both 1 and 2

d. Neither 1 nor 2

Ans: a

Exp: The share of agriculture and allied sectors in the Gross Value Added (GVA) of the country at

current prices has declined from 18.2 per cent in 2014-15 to 16.5 per cent in 2019-20

https://www.indiabudget.gov.in/economicsurvey/doc/echapter_vol2.pdf

Q37.Consider the following statements

1. Farm mechanization market in India has been decelerating.

2. Overall farm mechanization in India is lower compared to other countries.

Select the correct code.

a. 1 only

b. 2 only

c. Both 1 and 2

d. Neither 1 nor 2

Ans: b

Exp: Farm mechanization market in India has been growing at a CAGR of 7.53 per cent during 2016-

2018 due to thrust given by various government policies.

https://www.indiabudget.gov.in/economicsurvey/doc/echapter_vol2.pdf

Q38.The Economic Cost of foodgrains consists of

a. Pooled cost of grains

b. Procurement incidentals

c. Cost of distribution

d. All the above

Ans: d

Exp: The Economic Cost of foodgrains consists of three components, namely, pooled cost of grains,

procurement incidentals and the cost of distribution.

https://www.indiabudget.gov.in/economicsurvey/doc/echapter_vol2.pdf

Q39. Consider the following statements

1. The regional distribution of agricultural credit in India is highly skewed.

2. It is observed that credit is the least in North Eastern, Hilly and Eastern States.

3. It is observed that credit is the highest in Punjab and Uttar Pradesh

Select the correct code.

a. 1 only

b. 1 and 2 only

c. 1 and 3 only

https://t.me/UPSC_PDF Website ➡ https://upscpdf.com https://t.me/UPSC_PDF

Website ➡ https://upscpdf.com

Current Economics MCQs (Jan

2019 to Feb 20) + Eco Survey 2020

WriteToBeIAS.com Prelims 2020 Online Classes

by CA Rahul Kumar

Telegram https://t.me/writetobeias1 Youtube https://tinyurl.com/yx22y2yl Page 103

d. All the above

Ans: c

Exp: The regional distribution of agricultural credit in India is highly skewed (Figure 3). It is observed

that credit is low in North Eastern, Hilly and Eastern States. The share of North Eastern States has

been less than one percent in total agricultural credit disbursement.

https://www.indiabudget.gov.in/economicsurvey/doc/echapter_vol2.pdf

Q40. Pradhan Mantri Krishi Sinchayee Yojana (PMKSY) is amalgamation of which of the following

schemes?

1. Accelerated Irrigation Benefits Programme (AIBP)

2. Har Khet Ko Pani (HKKP)

3. Per Drop More Crop (PDMC)

4. Watershed Development (WD)

5. Command Area Development (CAD)

Select the answer from the codes given below:

(a) 1, 2, 3 and 4 only

(b) 1, 2, 3 and 5 only

(c) 1, 2, 4 and 5 only

(d) 1, 2, 3, 4 and 5

Ans: d

Exp: Pradhan Mantri Krishi Sinchayee Yojana (PMKSY) is amalgamation of various schemes viz.

Accelerated Irrigation Benefits Programme (AIBP), PMKSY –Har Khet Ko Pani (HKKP) including

Command Area Development (CAD), Surface Minor Irrigation (SMI) and Repair, Renovation and

Restoration (RRR) of Water Bodies [Implemented by MoWR, R&D; GR], PMKSY - Per Drop More

Crop (PDMC) [Implemented by Ministry of Agriculture & Farmers Welfare] and PMKSY –

Watershed Development (WD) [Implemented by Department of Land Resources].

http://www.pib.nic.in/PressReleseDetail.aspx?PRID=1562545

INFRASTRUCTURE

Q1. With reference to ‘National Investment and Infrastructure Fund’, which of the following

statements is/are correct?

1. It is an organ of NITI Aayog.

2. It has a corpus of Rs. 4, 00,000 crore at present.

Select the correct answer using the code given below:

(a) 1 only

(b) 2 only

(c) Both 1 and 2

(d) Neither 1 nor 2

Ans: d

Exp: NIIF has been structured as a fund of funds and set up as Category II Alternate Investment Fund

(AIF) under the Securities and Exchange Board of India (SEBI) Regulations. Total corpus of the fund

is Rs. 40000 Crore.

https://www.thehindu.com/business/roadis-niif-to-invest-2-billion/article26726029.ece

Q2. Consider following statements about National Rural Economic Transformation Project

(NRETP):

https://t.me/UPSC_PDF Website ➡ https://upscpdf.com https://t.me/UPSC_PDF

Website ➡ https://upscpdf.com

Current Economics MCQs (Jan

2019 to Feb 20) + Eco Survey 2020

WriteToBeIAS.com Prelims 2020 Online Classes

by CA Rahul Kumar

Telegram https://t.me/writetobeias1 Youtube https://tinyurl.com/yx22y2yl Page 104

1. It will help women in rural households shift to a new generation of economic initiatives by

developing viable enterprise for farm and non-farm products.

2. It is funded by World Bank.

Which of the statements given above is/are correct?

(a) 1 only

(b) 2 only

(c) Both 1 and 2

(d) Neither 1 nor 2

Ans: c

Exp: The World Bank and the Government of India signed here today in New Delhi a $250 Million

Agreement for the National Rural Economic Transformation Project (NRETP) which will help

women in rural households shift to a new generation of economic initiatives by developing viable

enterprise for farm and non-farm products.

http://www.pib.nic.in/PressReleseDetail.aspx?PRID=1567572

Q3. With reference to Bharat Mala Project, consider the following statements :

1. Under programme, roads will be built along borders with Bhutan and Nepal and bottlenecks on

existing Golden Quadrilateral highway network will be removed.

2. It envisages construction of roads along India’s borders, coastal areas, ports, religious and tourist

places as well as over 100 district headquarters.

3. The BharatMala project will include economic corridors, inter-corridor and feeder route, national

corridors efficiency improvement, border roads and international connectivity, coastal roads and port

connectivity and greenfield expressway

Which of the statements given above is/are correct?

(a) 1 and 3 only

(b) 2 only

(c) 2 and 3 only

(d) 1, 2 and 3

Ans: d

Exp: The BharatMala project will include economic corridors (9,000 km), inter-corridor and feeder

route (6,000 km), national corridors efficiency improvement (5,000 km), border roads and

international connectivity (2,000 km), coastal roads and port connectivity (2,000 km) and Greenfield

expressway (800 km).

https://www.thehindu.com/opinion/editorial/highway-hurdle/article26836892.ece

Q4. Consider the following statements:

1. NITI Aayog recently unveiled the Infrastructure Investment Trust (InvITs) roadmap.

2. Infrastructure Investment Trust (InvITs) are investment scheme similar to mutual funds that allow

investment from individuals and institutional investors in infrastructure projects to earn a portion of

the income as return.

Which of the statements given above is/are correct?

(a) Both1 and 2

(b) 1 only

(c) 2 only

(d) Neither 1 nor 2

Ans: c

Exp:National Highway Authority of India (NHAI) recently unveiled the Infrastructure Investment

Trust (InvITs) roadmap. Union Cabinet in December 2019 authorised NHAI to set up InvIT. The trust

https://t.me/UPSC_PDF Website ➡ https://upscpdf.com https://t.me/UPSC_PDF

Website ➡ https://upscpdf.com

Current Economics MCQs (Jan

2019 to Feb 20) + Eco Survey 2020

WriteToBeIAS.com Prelims 2020 Online Classes

by CA Rahul Kumar

Telegram https://t.me/writetobeias1 Youtube https://tinyurl.com/yx22y2yl Page 105

has enabled the NHAI to monetise completed national highways with toll collection record of atleast

one year and meet its funding requirements.

Source:https://www.livemint.com/news/india/gadkari-unveils-invit-road-map-for-nhai-initial-target-

20-000-cr-11580236271034.html

Q5. Consider the following statements related to “Bharatmala Project”:

1. Economic corridors

2. Inter-corridor and feeder roads

3. Border and International connectivity roads

4. Coastal and port connectivity roads

5. Expressways

Which of the above are included under the project?

Select the correct answer using the code given below

(a) 1,2 and 3 only

(b) 2,3and 4 only

(c) 1,2,3 and 4 only

(d) All of the above

Ans: d

Exp: The Government of India had approved BharatmalaPariyojana Phase-I in October 2017 with

an aggregate length of about 34,800 km (including 10,000 km residual NHDP stretches) at an

estimated outlay of Rs. 5,35,000 crore for development of about 9,000 km length of Economic

corridors, about 6,000 km length of Inter-corridor and feeder roads, about 5,000 km length of National

Corridors Efficiency improvements, about 2,000 km length of Border and International connectivity

roads, about 2,000 km length of Coastal and port connectivity roads, and about 800 km length of

Expressways.

https://pib.gov.in/PressReleseDetail.aspx?PRID=1595501

Q6.“Mattala Airport” is located in

(a) India

(b) Srilanka

(c) Maldives

(d) Seychelles

Ans: b

Exp: Sri Lanka’s civil aviation authority will develop the southern airport in Mattala, near the

strategically located Hambantota port, President Gotabaya Rajapaksa said on Thursday, ruling out

earlier proposals of a joint venture with India. However, he invited Indian companies, especially in

the IT sector, to invest in Sri Lanka.

https://www.thehindu.com/news/international/sri-lanka-not-india-will-develop-mattala-airport-

gotabaya-rajapaksa/article30351142.ece

Q7. India Smart Grid Forum (ISGF) is a public private partnership initiative of:

(a)Ministry of Coal

(b)Ministry of Communications

(c)Ministry of Power

(d)Ministry of Electronics and Information Technology

Ans:c

Exp: India Smart Grid Forum (ISGF) is a public private partnership initiative of Ministry of Power

(MoP), Government of India for accelerated development of smart grid technologies in the Indian

power sector. ISGF was set up in 2010 to provide a mechanism through which academia, industry,

https://t.me/UPSC_PDF Website ➡ https://upscpdf.com https://t.me/UPSC_PDF

Website ➡ https://upscpdf.com

Current Economics MCQs (Jan

2019 to Feb 20) + Eco Survey 2020

WriteToBeIAS.com Prelims 2020 Online Classes

by CA Rahul Kumar

Telegram https://t.me/writetobeias1 Youtube https://tinyurl.com/yx22y2yl Page 106

utilities and other stakeholders could participate in the development of Indian smart grid systems and

provide relevant inputs to the government’s decision-making.

Source:http://www.isgw.in/isgf/

Q8. “Chattogram” and “Mongla” recently seen in news are

(a) Coal mines

(b) Uranium mines

(c) Port

(d) Oil field

Ans: c

Exp: Foreign policy initiative regarding use of Chattogram and Mongla Ports in Bangladesh for

movement of goods to and from India will bring revolutionary cost-cutting in shipments to and from

the North Eastern States.

https://pib.gov.in/PressReleseDetail.aspx?PRID=1586891

Q9.India recently inaugurated the infrastructure facilities constructed to support the proposed oil

refinery project in

(a) Mongolia

(b) Bhutan

(c) Nepal

(d) Afghanistan

Ans: a

Exp: Minister of Petroleum & Natural Gas and Steel Shri Dharmendra Pradhan today participated in

the ceremony for commissioning the infrastructure facilities constructed to support the proposed oil

refinery project in Mongolia. Prime Minister of Mongolia, H. E .Mr. Ukhnaagi in Khurelsukh, six

Cabinet Ministers & Governor of Dornogovi Province, Mr. T. Enkhtuvshin also participated in the

ceremony.

https://pib.gov.in/PressReleseDetail.aspx?PRID=1586891

Q10. Consider the following:

1. National Investment and Infrastructure Fund Limited (NIIFL) is an investor-owned fund manager,

anchored by the Government of India (GoI) in collaboration with leading global and domestic

institutional investors.

2. The funds are registered as angel funds with the Securities and Exchange Board of India (SEBI).

Which of the statements given above is/are correct?

(a) 1only

(b) 2only

(c) Both1and 2

(d) Neither1nor2

Ans: a

Exp: National Investment and Infrastructure Fund Limited (NIIFL) is an investor-owned fund

manager, anchored by the Government of India (GoI) in collaboration with leading global and

domestic institutional investors. The funds are registered as Alternative Investment Fund (AIF) with

the Securities and Exchange Board of India (SEBI).

https://www.thehindu.com/business/niif-to-get-up-to-2-bn-from-australiansuper-ontario-

teachers/article28838821.ece

Q15. “FASTag” is a device that employs

(a) Radio Frequency

https://t.me/UPSC_PDF Website ➡ https://upscpdf.com https://t.me/UPSC_PDF

Website ➡ https://upscpdf.com

Current Economics MCQs (Jan

2019 to Feb 20) + Eco Survey 2020

WriteToBeIAS.com Prelims 2020 Online Classes

by CA Rahul Kumar

Telegram https://t.me/writetobeias1 Youtube https://tinyurl.com/yx22y2yl Page 107

(b) Microwave Frequency

(c) Infrared Frequency

(d) None of the above

Ans: a

Exp: NHAI has rolled out program for Electronic Toll Collection on Toll Plazas on National

Highways to be called FASTag.

FASTag is a device that employs Radio Frequency Identification (RFID) technology for making

toll payments directly from the prepaid account linked to it. It is affixed on the windscreen of your

vehicle and enables you to drive through toll plazas. FASTag has a validity of 5 years and after

purchasing it, you only need to recharge/top up the FASTag as per your requirement.

FASTag offers near non-stop movement of vehicles through toll plazas and the convenience of

cashless payment of toll fee with nation-wide interoperable Electronic Toll Collection Services.

https://www.thehindubusinessline.com/economy/logistics/nhai-readies-infrastructure-to-accept-

tolls-only-throughfastag-from-december-1/article29680320.ece

SOCIAL SECTOR

Q1.Consider the following statements related to ‘RashtriyaVayoshri Yojana’ (RVY):

1. Only for senior citizens, belonging to BPL category.

2. To provide physical aids and assisted living devices.

Which of the above statements is/are correct?

(a) 1 only

(b) 2 only

(c) Both 1 and 2

(d) Neither 1 nor 2

Ans: c

Exp: A Central Sector Scheme namely ‘Rashtriya Vayoshri Yojana’ (RVY) is being implemented by

D/o Social Justice and Empowerment (M/o SJ&E) with an objective to provide to senior citizens,

belonging to BPL category and suffering from age-related disabilities/ infirmities, with such physical

aids and assisted living devices which can restore near normalcy in their bodily functions.

https://pib.gov.in/PressReleseDetail.aspx?PRID=1595713

Q2.With reference to Bharatiya POSHAN Krishi Kosh, consider the following statements:

1. It was announced by the Ministry of Women and Child Development along with Bill & Melinda

Gates Foundation (BMGF) in November, 2019.

2. It has two components–Development of a Food Atlas and Documentation of promising practices

for Jan-Andolan for POSHAN Abhiyaan.

Which of the statements given above is/are correct?

(a)2 only

(b)1 only

(c)Both 1 and 2

(d)Neither 1 nor 2

Ans: c

Exp: Bhartiya Poshan Krishi Kosh” project has two components–Development of a Food Atlas and

Documentation of promising practices for Jan-Andolan for POSHAN Abhiyaan. The Agro-Food

Atlas is to act as a repository of diverse crops across 127 agro-climatic zones of the country having

three parts – crops currently being grown, agro-ecological conditions (soil, organic carbon content,

ground water availability etc) and guidance on how a greater diversity of crops could be encouraged

in a particular district or block to promote dietary diversity and nutrition. The project includes diverse

data sources like National Sample Survey, Agri-Census, Soil Health Cards, ISRO’s Advanced Wide

https://t.me/UPSC_PDF Website ➡ https://upscpdf.com https://t.me/UPSC_PDF

Website ➡ https://upscpdf.com

Current Economics MCQs (Jan

2019 to Feb 20) + Eco Survey 2020

WriteToBeIAS.com Prelims 2020 Online Classes

by CA Rahul Kumar

Telegram https://t.me/writetobeias1 Youtube https://tinyurl.com/yx22y2yl Page 108

Field Sensor (A WiFS) and National Aeronautics and Space Administration’s (NASA’s) Moderate

Resolution Imaging Spectro-Radiometer. The project also documents social, behavioural and cultural

practices that promote and reinforce healthy dietary behaviours. Identification of promising practices

with the help of a multi-disciplinary group of experts and developments of a tool kit to disseminate

best strategies for Social and Behavioural Change Communication, specific to population groups in

those regions is also a part of it.

Source:https://pib.gov.in/Pressreleaseshare.aspx?PRID=1595250

Q3.Consider the following statements:

1. Jal Jeevan Mission aims to give functional tap-water to every household in the country by 2022.

2. Water is part of the concurrent list.

Which of the statements given above is/are correct?

(a) 1 only

(b) 2 only

(c) Both 1 and 2

(d) Neither 1 nor 2

Ans: d

Exp: Jal Jeevan Mission aims to give functional tap-water to every house hold in the country by 2024.

Water is a State subject.

Source:https://www.thehindubusinessline.com/economy/jal-jeevan-mission-aims-to-give-tap-water-

to-all-households-by-2024/article29568145.ece

https://economictimes.indiatimes.com/news/politics-and-nation/pm-modi-to-release-operational-

guidelines-for-jal-jeevan-mission/articleshow/72960681.cms

Q4.Consider the following statements

1. Internet connectivity

2. Water conservation

3. SHGs

4. Sports Youth club culture

Which of the following are covered under Mission Antyodaya clusters?

(a) 1,2,3

(b) 2,3,4

(c) 1,3,4

(d) All

Ans: d

Exp: Mission Antyodaya is a convergence framework for measurable effective outcomes on

parameters that transform lives and livelihoods. Mission Antyodaya encourages partnerships with

network of professionals, institutions and enterprises to further accelerate the transformation of rural

livelihoods. Self Help Groups (SHG) are enablers to convergence approach due to their social capital

and their proven capacity for social mobilisation. The thrust is not only on physical infrastructure but

also on social infrastructure with strengthening of agriculture, horticulture, animal husbandry

activities with utmost priority given to SHG expansion in Mission Antyodaya clusters. Capacity for

financial and social audit would also be built at grass root level. This would be accomplished by

convergence of frontline worker teams, cluster resource persons (CRPs) and professionals at the

Gram Panchayats/ Cluster level. Cluster facilitation team would also be provided at the block level.

The aim is to make villages self-sufficient by focussing on almost all services-connectivity (road,

LPG, internet), women empowerment via SHGs, transport, Youth clubs, water conservation,

nutrition, financial credit etc.

Source: yojana September

https://t.me/UPSC_PDF Website ➡ https://upscpdf.com https://t.me/UPSC_PDF

Website ➡ https://upscpdf.com

Current Economics MCQs (Jan

2019 to Feb 20) + Eco Survey 2020

WriteToBeIAS.com Prelims 2020 Online Classes

by CA Rahul Kumar

Telegram https://t.me/writetobeias1 Youtube https://tinyurl.com/yx22y2yl Page 109

Q5. ‘The International Migrant Stock 2019’ report is released by

(a) UN Department of Economic and Social Affairs (DESA)

(b) UN Development Programme

(c) UN General Assembly

(d) Amnesty International

Ans: a

Exp: Recently, UN Department of Economic and Social Affairs (DESA) released ‘The International

Migrant Stock 2019’ report. In 2019, the number of international migrants worldwide has reached

nearly 272 million, up from 221 million in 2010. India was the leading country of origin of

international migrants in 2019 with a 17.5 million strong diaspora, while Mexico was at second place.

https://www.thehindu.com/news/national/india-is-the-top-source-of-immigrants-across-the-

globe/article29452221.ece

Q6.Consider the following about Global Liveability Index

1. It is given by Economist Intelligence Unit

2. New Delhi and Mumbai ranked at 55 and 85 respectively among 140 cities

Which of the following statements is/are correct?

(a)1 only

(b)2 only

(c) Both

(d) None

Ans:a

Exp: The index is released by the Economist’s Intelligence Unit. It assesses 140

Cities and ranks them according to their performance in over 30 qualitative and quantitative factors

across five broad categories-Stability, Healthcare, Culture and Environment, Education, and

Infrastructure. New Delhi and Mumbai ranked 118th and 119th out of 140 cities in the Global

Liveability Index, 2019. No Indian city ranked below 100.

https://www.livemint.com/news/india/these-two-indian-cities-are-in-global-liveability-index-2019-

1567582279267.html

Q7. With reference to Arogya Sanjeevani Policy, consider the following statements:

1. It is a Standard Health Insurance Product (SHIP) to be rolled out from April 1, 2020, onwards.

2. It primarily focuses on women of BPL households.

Which of the statements given above is/are correct?

(a)1 only

(b)2 only

(c)Both 1 and 2

(d)Neither 1 nor 2

Ans: a

Exp: IRDAI has mandated all general and health insurance companies to offer a Standard Health

Insurance Product (SHIP). The SHIP is aimed at taking care of basic health needs, carrying similar

policy wordings and enabling seamless portability among the insurers. The SHIP of each insurer will

have the nomenclature of Arogya Sanjeevani Policy followed by the insurer’s name. For example,

Arogya Sanjeevani Policy SBI General insurance, Arogya Sanjeevani Policy Apollo Munich etc.

IRDAI has already issued guidelines and the insurers have to roll out Arogya Sanjeevani Policy from

April1, 2020, onwards.

https://t.me/UPSC_PDF Website ➡ https://upscpdf.com https://t.me/UPSC_PDF

Website ➡ https://upscpdf.com

Current Economics MCQs (Jan

2019 to Feb 20) + Eco Survey 2020

WriteToBeIAS.com Prelims 2020 Online Classes

by CA Rahul Kumar

Telegram https://t.me/writetobeias1 Youtube https://tinyurl.com/yx22y2yl Page 110

Source:https://economictimes.indiatimes.com/wealth/insure/health-insurance/what-is-arogya-

sanjeevani-health-insurance-policy/articleshow/74254954.cms?from=mdr

Q8. Consider the following

1. Vidya Lakshmi Portal provides single window for Students to access information and make

application for Educational Loans provided by Banks and Govt Scholarships.

2. A student can apply to a maximum of two Banks through Vidya Lakshmi portal using CELAF.

Which of the statements given above is/are correct?

(a) 1 only

(b) 2 only

(c) Both 1 and 2

(d) Neither 1 nor 2

Ans: a

Exp: Vidya Lakshmi Portal provides single window for Students to access information and make

application for Educational Loans provided by Banks and Govt Scholarships.

A student can apply to a maximum of three Banks through Vidya Lakshmi portal using CELAF.

https://pib.gov.in/PressReleseDetail.aspx?PRID=1575474

Q9. Consider the following:

1. Health & Nutrition

2. Education

3. Artificial Intelligence

4. Financial inclusion

5. Skill development

6. Basic Infrastructure

Which of the above are focus area for Transformation of Aspirational District Programme?

(a) 1, 2, 3 and 4

(b) 1, 2, 3 and 5

(c) 1, 2, 4, 5 and 6

(d) 1, 2, 3, 4 and 5

Ans: c

Exp: Focus area for Transformation of Aspirational District Programme are- Health & Nutrition,

Education, Financial inclusion & Skill development, Basic Infrastructure, Agriculture & Water

resources.

https://pib.gov.in/PressReleseDetail.aspx?PRID=1574715

Q13.Which of the following statements are true?

1. The share of regular wage/salaried employees has decreased as per usual status.

2. The proportion of women workers in the regular wage/salaried employees category has

increased.

3. Among the self-employed category, while the proportion of own account workers and

employers increased.

4. The distribution of workers in the casual labour category increased.

Select the correct code

a. 1 and 2 only

b. 2 and 3 only

c. 1 and 3 only

https://t.me/UPSC_PDF Website ➡ https://upscpdf.com https://t.me/UPSC_PDF

Website ➡ https://upscpdf.com

Current Economics MCQs (Jan

2019 to Feb 20) + Eco Survey 2020

WriteToBeIAS.com Prelims 2020 Online Classes

by CA Rahul Kumar

Telegram https://t.me/writetobeias1 Youtube https://tinyurl.com/yx22y2yl Page 111

d. 2 and 4 only

Ans: b Exp: The share of regular wage earners and salaried employees in the total urban workforce increased

marginally between April-June 2018 and January-March 2019 — from 48.3% to 50% — with women

faring better than men, the latest Periodic Labour Force Survey (PLFS) released by the National

Statistical Organisation (NSO) showed.

On the flip side, there was a decrease in the proportion of the self-employed – from 38.9% in April-

June 2018 to 37.7% in the latest period surveyed.

http://timesofindia.indiatimes.com/articleshow/72215099.cms?utm_source=contentofinterest&utm_

medium=text&utm_campaign=cppst

Q14.Consider the following statements

1. Female labour force participation rate for productive age-group shows a declining trend.

2. The rate of decline was also sharper in urban areas compared to rural areas

Select the correct code

a. 1 only

b. 2 only

c. Both 1 and 2

d. Neither 1 nor 2

Ans: a Exp: According to NSO-EUS and PLFS estimates, female labour force participation rate (LFPR) for

productive age-group (15- 59 years) as per usual status (ps+ss) shows a declining trend.

Though female LFPR is higher in rural areas as compared to urban areas, the rate of decline was also

sharper in rural areas compared to urban areas

https://www.indiabudget.gov.in/economicsurvey/doc/echapter_vol2.pdf

SERVICE SECTOR

Q1.Which of the following statements about the service sector are true?

a. It accounts for around 55% of the total size of the economy and GVA growth.

b. It accounts for two-thirds of total FDI inflows into India.

c. It accounts for about 38% of total exports.

d. All the above

Ans: d

Exp: The services sector’s significance in the Indian economy has continued to increase, with the

sector now accounting for around 55 per cent of total size of the economy and GVA growth, two-

thirds of total FDI inflows into India and about 38 per cent of total exports.

https://www.indiabudget.gov.in/economicsurvey/doc/vol2chapter/echap09_vol2.pdf

Q2.Consider the following statements

1. FDI equity inflows accounted for about two-thirds of the total gross FDI equity inflows into

India.

2. Bank credit to the services sector has continued to decelerate.

Select the correct code

a. 1 only

b. 2 only

https://t.me/UPSC_PDF Website ➡ https://upscpdf.com https://t.me/UPSC_PDF

Website ➡ https://upscpdf.com

Current Economics MCQs (Jan

2019 to Feb 20) + Eco Survey 2020

WriteToBeIAS.com Prelims 2020 Online Classes

by CA Rahul Kumar

Telegram https://t.me/writetobeias1 Youtube https://tinyurl.com/yx22y2yl Page 112

c. Both 1 and 2

d. Neither 1 nor 2

Ans: c

Exp: Gross FDI equity inflows jumped by 33 per cent YoY during AprilSeptember 2019 to reach

US$ 17.58 billion, accounting for about two-thirds of the total gross FDI equity inflows into India

during this period

Even the growth in rail freight, which was contracting in the past few months, has picked up in

November 2019. In contrast, bank credit to the services sector has continued to decelerate. The growth

in bank credit to services sector was 4.8 per cent as on November 2019 as compared to 28.1 per cent

a year ago

https://www.indiabudget.gov.in/economicsurvey/doc/vol2chapter/echap09_vol2.pdf

Q3.Consider the following statements

1. India now ranks 1st among the world’s largest commercial services exporters.

2. Despite India’s strong performance in services exports, India’s financial services exports have

remained stagnant

Select the correct code

a. 1 only

b. 2 only

c. Both 1 and 2

d. Neither 1 nor 2

Ans: b

Exp: India now ranks 8th among the world’s largest commercial services exporters and continues to

register strong growth performance relative to the other major services-exporting countries as well as

world services export growth

India’s financial services exports have remained stagnant, averaging about US$ 5 billion in the recent

years. As a result, the share of financial services exports in overall services exports has almost halved

from 4.2 per cent in 2011-12 to 2.3 per cent in 2018-19

https://www.indiabudget.gov.in/economicsurvey/doc/vol2chapter/echap09_vol2.pdf

Q4.Consider the following statements

1. The top five states attracting foreign tourists are Tamil Nadu, Maharashtra, Uttar Pradesh,

Delhi and Rajasthan.

2. The top five states attracting domestic tourists are Tamil Nadu, Uttar Pradesh, Karnataka,

Andhra Pradesh and Maharashtra.

Select the correct code

a. 1 only

b. 2 only

c. Both 1 and 2

d. Neither 1 nor 2

Ans:c

Exp: Looking at tourism trends at the state level, the top five states attracting domestic tourists are

Tamil Nadu, Uttar Pradesh, Karnataka, Andhra Pradesh and Maharashtra, accounting for nearly 65

per cent of the total domestic tourist visits in the country in 2018.

The top five states attracting foreign tourists are Tamil Nadu, Maharashtra, Uttar Pradesh, Delhi and

Rajasthan, accounting for about 67 per cent of the total foreign tourist visits in the country in 2018.

https://www.indiabudget.gov.in/economicsurvey/doc/vol2chapter/echap09_vol2.pdf

https://t.me/UPSC_PDF Website ➡ https://upscpdf.com https://t.me/UPSC_PDF

Website ➡ https://upscpdf.com

Current Economics MCQs (Jan

2019 to Feb 20) + Eco Survey 2020

WriteToBeIAS.com Prelims 2020 Online Classes

by CA Rahul Kumar

Telegram https://t.me/writetobeias1 Youtube https://tinyurl.com/yx22y2yl Page 113

SUSTAINABLE DEVELOPMENT

Q1. SATAT is an initiative aimed at

(a) Alternative towards Affordable Transportation

(b) Doubling farmers income

(c) Electricity to all

(d) None of the above

Ans: a

Exp: SATAT is an initiative aimed at providing a Sustainable Alternative Towards Affordable

Transportation as a developmental effort that would benefit both vehicle-users as well as farmers and

entrepreneurs.

http://pib.nic.in/PressReleseDetail.aspx?PRID=1566541

Q2. Swachh Bharat Mission (Grameen), (SBM-G ) assumes significance in the wake of cleanliness

in India. Which of the followings are key components of the implementation of SBM-G :

1. Construction of household toilets

2. Construction of community sanitary complexes

3. A revolving fund at the Village level

4. Funds for rural sanitary marts, where materials for the construction of toilets, etc., may be

purchased

5. Funds for solid and liquid waste management.

Select the correct answer using the code given below :

(a) 1, 3 and 4

(b) 2, 3, 4 and 5

(c) 2, 4 and 5

(d) 1, 2, 4 and 5

Ans: d

Exp:A revolving fund at the district level

http://pib.nic.in/newsite/PrintRelease.aspx?relid=189803

Q3. Consider the following statements with respect to Trade Related Analysis of Fauna andFlora in

Commerce (TRAFFIC)

1. It is a bureau under United Nations Environment Programme (UNEP).

2. It works globally on trade in wild animals and plants in the context of both biodiversity

conservation and sustainable development.

Which of the statement(s) given above is/are correct?

(a) 1 only

(b) 2 only

(c) Both 1 and 2

(d) Neither 1 nor 2

Ans: b

Exp:TRAFFIC is a leading non-governmental organization working globally on trade in wild animals

and plants in the context of both biodiversity conservation andsustainable development.

https://www.thehindu.com/sci-tech/energy-and-environment/social-media-used-for-illegal- wildlife-

trade-conservationists/article8309884.ece

Q4. Which of the following state recently set up the country's first International Women's Trade

Centre (iWTC), inconsonance with the United Nations Sustainable Development Goals (SDGs).

(a) Telangana

(b) Gujarat

https://t.me/UPSC_PDF Website ➡ https://upscpdf.com https://t.me/UPSC_PDF

Website ➡ https://upscpdf.com

Current Economics MCQs (Jan

2019 to Feb 20) + Eco Survey 2020

WriteToBeIAS.com Prelims 2020 Online Classes

by CA Rahul Kumar

Telegram https://t.me/writetobeias1 Youtube https://tinyurl.com/yx22y2yl Page 114

(c) Maharashtra

(d) Kerala

Ans: d

Exp: Kerala will set up the country's first International Women's Trade Centre (iWTC), in

consonance with the United Nations Sustainable Development Goals (SDGs), in Kozhikode. It is a

key project of the state's Gender Park “Vision 2020” under the Department of Social Justice. The first

phase is scheduled to be completed by 2021. The facility will have women's start-up and incubation

centre, business centers and offices, retail fashion and technology outlets, health and wellness centers,

performing arts centre, residential suite and day-care for seniors and children. iWTC will also have a

dedicated space for cultural expressions of women to promote and showcase performing arts and

cultural forms like dance, music and theatre.

https://www.thehindu.com/news/national/kerala/global-trade-centre-for-women-

soon/article29377470.ece

Q5.Which among the following international organisations has launched a global initiative to grow,

restore and conserve 1 trillion trees around the world?

(a)World Economic Forum

(b)UNEP

(c)IUCN

(d)World Wide Fund for Nature

Ans:a

Exp: World Economic Forum (WEF) at its Davos meeting has launched a global initiative to grow,

restore and conserve 1 trillion trees around the world-in a bid to restore biodiversity and help fight

climate change.

Source:https://in.reuters.com/article/davos-meeting-trump-climatechange/trump-says-u-s-to-join-

one-trillion-trees-initiative-idINKBN1ZK19M

https://www.weforum.org/agenda/2020/01/one-trillion-trees-world-economic-forum-launches-plan-

to-help-nature-and-the-climate/

Q6.Consider the following statements:

1.Green Credit scheme allows “forests” to be traded as a commodity.

2.It allows the Forest Department to outsource one of its responsibilities of reforesting to non-

government agencies.

Which of the statements given above is/are correct?

(a)Both 1 and 2

(b)1 only

(c)2 only

(d)Neither 1 nor 2

Ans:a

Exp: Forest Advisory Committee has approved the implementation of Green Credit scheme.

It allows “forests” to be traded as a commodity. It allows the Forest Department to outsource one of

its responsibilities of reforesting to non-government agencies. The scheme allows agencies—they

could be private companies, village forest communities—toidentify land and begin growing

plantations. After three years, they would be eligible to be considered as compensatory forest land if

they met the Forest Department’s criteria. An industry needing forest land could then approach the

agency and pay it for parcels of such forested land, and this would then be transferred to the Forest

Department and be recorded as forestland.

The participating agency will be free to trade its asset, that is plantation, in parcels, with project

proponents who need forest land.

https://t.me/UPSC_PDF Website ➡ https://upscpdf.com https://t.me/UPSC_PDF

Website ➡ https://upscpdf.com

Current Economics MCQs (Jan

2019 to Feb 20) + Eco Survey 2020

WriteToBeIAS.com Prelims 2020 Online Classes

by CA Rahul Kumar

Telegram https://t.me/writetobeias1 Youtube https://tinyurl.com/yx22y2yl Page 115

Source:https://www.thehindu.com/news/national/forest-committee-approves-scheme-to-trade-in-

forests/article30527065.ece

Q7. With reference to the State Energy Efficiency Index 2019, consider the following:

1. The index is developed by Bureau of Energy Efficiency (BEE) in association with Alliance for an

Energy Efficient Economy (AEEE)

2.Haryana, Karnataka and Kerala have topped the State Energy Efficiency Index 2019.

Which of the above statements are correct?

(a)2 only

(b)1 and 2

(c)1 only

(d)None of the above

Ans: b

Exp: This index tracks the progress of Energy Efficiency (EE) initiatives in 36 states and union

territories based on 97 significant indicators. It categorises states as ‘Front Runner’, ‘Achiever’,

‘Contender’ and ‘Aspirant’ based on their efforts and achievements towards energy efficiency

implementation. “The topper forming states in the State EE Index 2019-Haryana, Kerala and

Karnataka-are in the ‘Achiever’ category. Since there isn’t any ‘front runner’ state, it can be inferred

that a lot more can be done at the state level to realise energy savings from energy efficiency,” there

port on the index said.

Source:https://www.thehindubusinessline.com/economy/bee-launches-state-energy-efficiency-

index-2019/article30533640.ece

Q8. With reference to UNESCO Creative Cities Network (UCCN), consider the following statements:

1. The UNESCO Creative Cities Network (UCCN) was created in 2004 to promote cooperation with

and among cities that have identified creativity as a strategic factor for sustainable urban and rural

development.

2. Mumbai, Hyderabad and Chennai are the only Indian cities that have been designated as members

of UCCN so far.

3.Categories for recognition under UCCN include Crafts and Folk Arts, Design, Film, Gastronomy,

Music, Media Arts and Literature.

Which of the statements given above is/are correct?

(a)1 and 3 only

(b)1, 2 and 3

(c)3 only

(d)2 only

Ans:c

Exp:The UNESCO Creative Cities Network (UCCN) was created in 2004 to promote cooperation

with and among cities that have identified creativity as a strategic factor for sustainable urban

development. UNESCO has designated Mumbai as a member of UCCN in the field of FILM and

Hyderabad in the field of GASTRONOMY. Previously, 3 Indian cities were recognized as members

of UCCN, namely- Jaipur- Crafts and Folk Arts (2015), Varanasi-Creativecity of Music (2015) and

Chennai- Creativecity of Music (2017).

Source: https://pib.gov.in/newsite/PrintRelease.aspx?relid=194177

https://www.business-standard.com/article/pti-stories/mumbai-hyderabad-join-unesco-network-of-

creative-cities-119103101477_1.html

Q9.“Green Channel” ,recently in the news recently is related to

(a) Environment clearance for a project

https://t.me/UPSC_PDF Website ➡ https://upscpdf.com https://t.me/UPSC_PDF

Website ➡ https://upscpdf.com

Current Economics MCQs (Jan

2019 to Feb 20) + Eco Survey 2020

WriteToBeIAS.com Prelims 2020 Online Classes

by CA Rahul Kumar

Telegram https://t.me/writetobeias1 Youtube https://tinyurl.com/yx22y2yl Page 116

(b) Mergers and acquisition

(c) A TV channel launched by BBC

(d) None of the above

Ans:b

Exp: Competition Commission of India (CCI), in August announced a “greenchannel” to expedite

approval of mergers and acquisitions (M&As) that fall into certain categories. The Competition

Commission of India (CCI) received the first green channel combination filed undersub-section (2)

of Section 6 of the Competition Act, 2002 (Act) read with regulations 5 and 5A of the Competition

Commission of India (Procedure in regard to the transactions of business relating to combinations)

Regulations, 2011 (Combination Regulations), 3rd October, 2019.

https://pib.gov.in/PressReleseDetail.aspx?PRID=1586891

Q10.Consider following statements:

1. Zero Budget Natural Farming (ZBNF) is a set of farming methods, and a grass roots peasant

movement, which has spread to various states in India.

2.It was the southern Indian state of Kerala where ZBNF first evolved.

Which of the statements given above is/are correct?

(a)1 only

(b)2 only

(c)Both 1 and 2

(d)Neither 1 nor 2

Ans: a

Exp: Zero Budget Natural Farming (ZBNF)is a set of farming methods, and a grassroots peasant

movement, which has spread to various states in India. It was the southern Indian state of Karnataka

where it first evolved.

https://www.thehindu.com/opinion/lead/stirring-up-the-truth-about-zbnf/article29620843.ece

http://www.fao.org/agroecology/detail/en/c/443712/

Q11.Consider the following statements:

1. BS-VI mass emission standard for vehicles are mandatory throughout the country from April 1,

2022 to curb pollution across the country.

2. National Electronic Toll Collection (FASTag) programme can also be used for parking purposes.

Which of the above statements is/are correct?

(a) 1 only

(b) 2 only

(c) Both 1 and 2

(d) Neither 1 nor 2

Ans: b

Exp: BS-VI mass emission standard for vehicles are mandatory throughout the country from April 1,

2020 to curb pollution across the country. National Electronic Toll Collection (FASTag) programme

can also be used for parking purposes.

https://pib.gov.in/PressReleseDetail.aspx?PRID=1595553

Q12.Consider the following statements related toIREDA:

1. IREDA is India’s financial institution dedicated to clean energy expansion.

2. IREDA has financed the largest share of renewable energy projects in India.

3. It was founded under MNRE in 2009.

Which of the statements given above is/are correct ?

(a) 2 and 3 only

https://t.me/UPSC_PDF Website ➡ https://upscpdf.com https://t.me/UPSC_PDF

Website ➡ https://upscpdf.com

Current Economics MCQs (Jan

2019 to Feb 20) + Eco Survey 2020

WriteToBeIAS.com Prelims 2020 Online Classes

by CA Rahul Kumar

Telegram https://t.me/writetobeias1 Youtube https://tinyurl.com/yx22y2yl Page 117

(b) 2 only

(c) 1 and 2 only

(d) 1,2 and 3

Ans: c

Exp:IREDA is India’s financial institution dedicated to clean energy expansion. IREDA has financed

the largest share of renewable energy projects in India. It was founded under MNRE in 1987.

https://pib.gov.in/PressReleseDetail.aspx?PRID=1595888

Q13.Consider the following statements:

1. The Renewable Energy and Energy Efficiency Partnership(REEEP) develops innovative, efficient

financing mechanisms to strengthen markets for clean energy services in low- and middle-income

countries.

2. REEEP's work is made possible by generous donors and are funded primarily by sovereign

governments, multilateral and international organizations.

Which of the above statements is/are correct?

(a) 1 only

(b) 2 only

(c) Both 1 and 2

(d) Neither 1 nor 2

Ans:c

Exp: The Renewable Energy and Energy Efficiency Partnership(REEEP) develops innovative,

efficient financing mechanisms to strengthen markets for clean energy services in low- and middle-

income countries. REEEP's work is made possible by generous donors and are funded primarily by

sovereign governments, multilateral and international organizations.

https://pib.gov.in/PressReleseDetail.aspx?PRID=1598040

Q14.Which among the following plans to phase out fossil fuel financing?

(a)Nordic Investment Bank(NIB)

(b)Asian Development Bank(ADB)

(c)European Investment Bank(EIB)

(d)New Development Bank(NDB)

Ans:c

Exp: The European Investment Bank has agreed to phase out its multibillion-euro financing for

fossil fuels within the next two years to become the world’s first “climate bank”. The bank will end

its financing of oil, gas, and coal projects after 2021, a policy that will make the EU’s lending arm

the first multilateral lender to rule out financing for projects that contribute to the climate crisis.

Source:https://www.theguardian.com/environment/2019/nov/15/european-investment-bank-to-

phase-out-fossil-fuels-financing

Q15.Consider the following statements:

1.GCF helps developed and developing countries limit or reduce their green house gas (GHG)

emissions and adapt to climate change.

2.It was set up by the 194 countries who are parties to the United Nations Framework Convention on

Climate Change (UNFCCC) in 2009, as part of the Convention’s financial mechanism.

Which of the statements given above is/are correct?

(a)2 only

(b)1 only

(c)Both1 and 2

(d)Neither1 nor 2

https://t.me/UPSC_PDF Website ➡ https://upscpdf.com https://t.me/UPSC_PDF

Website ➡ https://upscpdf.com

Current Economics MCQs (Jan

2019 to Feb 20) + Eco Survey 2020

WriteToBeIAS.com Prelims 2020 Online Classes

by CA Rahul Kumar

Telegram https://t.me/writetobeias1 Youtube https://tinyurl.com/yx22y2yl Page 118

Ans:d

Exp: GCF helps developing countries limit or reduce their green house gas (GHG) emissions and

adapt to climate change. It was set up by the 194 countries who are parties to the United Nations

Framework Convention on Climate Change (UNFCCC) in 2010, as part of the Convention’s financial

mechanism.

Source:https://economictimes.indiatimes.com/news/international/world-news/green-climate-fund-

hosts-two-day-fundraising-conference-in-paris/articleshow/71764171.cms

https://www.greenclimate.fund/who-we-are/about-the-fund

Q16. Consider the following statements in respect of Trade Related Analysis of Fauna and Flora in

Commerce (TRAFFIC):

1. TRAFFIC is a bureau under United Nations Environment Programme(UNEP).

2. The emission of TRAFFIC is to ensure that trade in wild plants and animals is not a threat to the

conservation of nature.

Which of the above statements is/are correct?

(a) 1 only

(b) 2 only

(c) Both 1 and 2

(d) Neither 1 nor2

Ans:b

Exp: Wildlife trade is one of the world's most pressing conservation challenges, affecting hundreds

of millions of people and thousands of wildlife species. TRAFFIC is a leading non-governmental

organisation working globally on trade in wild

animalsandplantsinthecontextofbothbiodiversityconservationandsustainabledevelopment.

https://www.thehindu.com/sci-tech/energy-and-environment/indian-seeks-change-of-status-of-

several-species-like-star-tortoise-otters-in-cites/article28917079.ece

Q17.Consider the following statements:

1. Green climate fund is accountable to the World Bank.

2. The Fund’s investments can be in the form of grants, loans, equity or guarantees.

3. The Fund is headquartered in Songdo, in the Republic of Korea.

Select the correct answer using the codes given below:

(a) 1only

(b) 2 and 3 only

(c) 1 and 3 only

(d) 1, 2 and 3

Ans:b

Exp: GCF is accountable to the United Nations. It is guided by the principles and provisions of the

UN Framework Convention on Climate Change (UNFCCC).

The Green Climate Fund is the only stand-alone multilateral financing entity whose sole mandate is

to serve the Convention and that aims to deliver equal amounts of funding to mitigation and

adaptation.

The Fund’s Governing Instrumentsets out its mandate and working methods.

GCF is governedby a 24-member Board, comprised equally of developed and developing countries,

representing the United Nations Regional Groups. It makes decisions based only on the consensus

agreement of all Board members. Board documentsare available online.

The Fund is headquartered in Songdo, in the Republic of Korea.

https://pib.gov.in/PressReleseDetail.aspx?PRID=1582241

https://t.me/UPSC_PDF Website ➡ https://upscpdf.com https://t.me/UPSC_PDF

Website ➡ https://upscpdf.com

Current Economics MCQs (Jan

2019 to Feb 20) + Eco Survey 2020

WriteToBeIAS.com Prelims 2020 Online Classes

by CA Rahul Kumar

Telegram https://t.me/writetobeias1 Youtube https://tinyurl.com/yx22y2yl Page 119

Q18.Consider the following statements

1. The forest and tree cover 24.56% of the geographical area of the country.

2. India reduced its emission intensity by 21% from 2005 to 2014.

Select the correct code

a. 1 only

b. 2 only

c. Both 1 and 2

d. Neither 1 nor 2

Ans: c

Exp: . India is among a few countries in the world where forest and tree cover have increased

considerably. The forest and tree cover have reached 80.73 million hectare which is 24.56 per cent of

the geographical area of the country.

India’s sustained actions on addressing climate change have helped her to achieve great strides which

are reflected in reduction in the emission intensity of India’s GDP by 21 per cent during 2005-2014.

https://www.indiabudget.gov.in/economicsurvey/doc/vol2chapter/echap06_vol2.pdf

Q19. Consider the following statements

1. There has been an increase of forest and tree cover compared to the previous assessment.

2. In terms of canopy density classes, the area covered by Very Dense Forest is the highest.

Select the correct code

a. 1 only

b. 2 only

c. Both 1 and 2

d. Neither 1 nor 2

Ans: a

Exp: There has been an increase of 3,976 sq. km (0.56 per cent) of forest cover, 1,212 sq. km (1.29

per cent) of tree cover and 5,188 sq. km (0.65 per cent) of forest and tree cover put together, at the

national level as compared to the previous assessment 2017.

In terms of canopy density classes, area covered by Very Dense Forest (VDF) is 99,278 sq. km (3.02

per cent), Moderately Dense Forest (MDF) is 3,08,472 sq. km (9.39 per cent) and Open Forest (OF)

is 3,04,499 sq. km (9.26 per cent)

https://www.indiabudget.gov.in/economicsurvey/doc/vol2chapter/echap06_vol2.pdf

Q20.Global Forest Resource Assessment is released by

a. UNICEF

b. FAO

c. IUCN

d. WWF

Ans: b

Q21.Consider the following statements

1. Net change in carbon stock in India shows that net change is highest in litter carbon.

2. Net change in carbon stock in India shows that net change is least in soil organic carbon.

Select the correct code

a. 1 only

b. 2 only

c. Both 1 and 2

d. Neither 1 nor 2

Ans: d

https://t.me/UPSC_PDF Website ➡ https://upscpdf.com https://t.me/UPSC_PDF

Website ➡ https://upscpdf.com

Current Economics MCQs (Jan

2019 to Feb 20) + Eco Survey 2020

WriteToBeIAS.com Prelims 2020 Online Classes

by CA Rahul Kumar

Telegram https://t.me/writetobeias1 Youtube https://tinyurl.com/yx22y2yl Page 120

Exp: Net change in carbon stock in India shows that net change is highest in soil organic carbon

followed by Above Ground Biomass (AGB) and Dead wood. Litter carbon registered negative growth

rate as compared to 2017 assessment.

https://www.indiabudget.gov.in/economicsurvey/doc/vol2chapter/echap06_vol2.pdf

Q22.Consider the following statements

1. The Shannon-Wiener Index for Diversity is used for measuring species richness and

abundance.

2. The index also compares diversity of species among various habitats.

Select the correct code

a. 1 only

b. 2 only

c. Both 1 and 2

d. Neither 1 nor 2

Ans: c

Exp: India is one of the 17 mega diverse countries in the world. This is evident in the Shannon-Weiner

Index for Diversity which is used for measuring species richness and abundance. The index also

compares diversity of species among various habitats. India State of the Forest Report 2019 assessed

Shannon-Weiner Index for 16 biodiverse areas.

https://www.indiabudget.gov.in/economicsurvey/doc/vol2chapter/echap06_vol2.pdf

GOVERNMENT SCHEMES

Q1.Consider the following statements:

1. The ‘Food Safety Mitra (FSM)’ scheme will support medium and large scale food businesses to

comply with the food safety laws.

2. The ‘Eat Right Jhola’, are usable cloth bag shall replace plastic bags for grocery shopping in various

retail chains.

Which of the above statements is/are correct?

(a)1 only

(b)2 only

(c) Both 1 and 2

(d)Neither 1 nor 2

Ans: b

Exp: The ‘Food Safety Mitra (FSM)’ scheme will support small and medium scale food businesses

to comply with the food safety laws and facilitate licensing and registration, hygiene ratings and

training. The ‘Eat Right Jhola’, a reusable cloth bag shall replace plastic bags for grocery shopping

in various retail chains.

https://pib.gov.in/PressReleseDetail.aspx?PRID=1587688

Q2.Who among the following can join the National Pension System(NPS)?

(a)Resident Indian citizens only

(b)Persons of age from 21 to 55 only

(c)All State Government employees joining the services after the date of notification by the respective

State Governments

(d)All Central Government employees including those of Armed Forces joining the services on or

after 1St April, 2004

Ans: c

https://t.me/UPSC_PDF Website ➡ https://upscpdf.com https://t.me/UPSC_PDF

Website ➡ https://upscpdf.com

Current Economics MCQs (Jan

2019 to Feb 20) + Eco Survey 2020

WriteToBeIAS.com Prelims 2020 Online Classes

by CA Rahul Kumar

Telegram https://t.me/writetobeias1 Youtube https://tinyurl.com/yx22y2yl Page 121

Exp: PFRDA in its endeavour to promote and develop NPS has taken several initiatives towards

increasing the pension coverage in the country. Now, any Indian citizen, resident or non-resident and

OCIs are eligible to join NPS till the age of 65years.

https://pib.gov.in/PressReleseDetail.aspx?PRID=1587688

Q3.With reference to Pradhan Mantri Matru Vandana Yojana (PMMVY), consider the following

statements:

1.It is a flagship scheme of the Government for pregnant women and lactating mothers.

2.It is a direct benefit transfer scheme under which cash benefits are provided to pregnant women in

their bank account directly to meet enhanced nutritional needs and fully compensate for wage loss.

3.The eligible beneficiaries also receive cash incentive under Pradhan Mantri Surakshit Matritva

Abhiyan.

Which of the statements given above is/are correct?

(a)1 and 2 only

(b)1 only

(c)2 only

(d)1 and 3 only

Ans: b

Exp: It is a direct benefit transfer scheme under which cash benefits are provided to pregnant women

in their bank account directly to meet enhanced nutritional needs and partially compensate for wage

loss. The eligible beneficiaries also receive cash incentive under Janani Suraksha Yojana (JSY)

Q4.Consider the following statements:

1.UJALA is a flagship project of the government where it wants every BPL household in India to use

LED bulbs.

2.Energy Efficiency Services Limited (EESL), a joint venture of PSUs under the Union Ministry of

Power.

3.Street Light National Programme (SNLP) was launched in 2015.

Which of the statements given above is/are correct?

(a)2 only

(b)2 and 3 only

(c)3 only

(d)1 and 3 only

Ans:b

Exp: Launched on January 5th, 2015, the Government of India’s zero subsidy Unnat Jyoti by

Affordable LEDs for All (UJALA) and LED Street Lighting National Programme (SLNP), mark edits

fifth anniversary today. SLNP is the world’s largest street light replacement programme and UJALA

is the world’s largest domestic lighting project. Both have been spearheaded and implemented by

Energy Efficiency Services Limited (EESL), a joint venture of PSUs under the Ministry of Power,

Government of India. UJALA is a flagship project of the government where it wants every home in

India to use LED bulbs.

Source:https://pib.gov.in/newsite/PrintRelease.aspx?relid=197275

Q5. Consider the following statements related to Pradhan Mantri Van Dhan Yojana (PMVDY):

1. Pradhan Mantri Van Dhan Yojana (PMVDY) is a Market Linked Tribal Entrepreneurship

Development Program for forming clusters of tribal Self-Help Groups.

https://t.me/UPSC_PDF Website ➡ https://upscpdf.com https://t.me/UPSC_PDF

Website ➡ https://upscpdf.com

Current Economics MCQs (Jan

2019 to Feb 20) + Eco Survey 2020

WriteToBeIAS.com Prelims 2020 Online Classes

by CA Rahul Kumar

Telegram https://t.me/writetobeias1 Youtube https://tinyurl.com/yx22y2yl Page 122

2. TRIFED has also partnered with UNESCO for promoting Van Dhan program through the tribal

Culture, Craft, Cuisine and Commerce.

Which of the above statements is/are correct?

(a) 1 only

(b) 2 only

(c) Both 1 and 2

(d) Neither 1 nor 2

Ans: a

Exp: Pradhan Mantri Van Dhan Yojana (PMVDY) is a Market Linked Tribal Entrepreneurship

Development Program for forming clusters of tribal Self-Help Groups.TRIFED has also partnered

with UNICEF for promoting Van Dhan program through the tribal Culture, Craft, Cuisine and

Commerce.

https://pib.gov.in/PressReleseDetail.aspx?PRID=1595421

Q6. Consider the following statements

1. National Skill Development Fund was set up in 2009 by the Government of India for raising funds

both from Government and Non Government sectors for skill development.

2. The Fund meets its objectives through National Skill Development Corporation (NSDC) which is

an industry led ‘Not For Profit Company’ set up for building skill development capacity and forging

strong linkages with the market.

Which of the statements given above is/are correct?

Select the correct answer using the code given below:

(a) 1 only

(b) 2 only

(c) Both 1 and 2

(d) Neither 1 nor 2

Ans. c

Source: https://www.msde.gov.in/nationalskilldevelopmentfund.html

Q7.New Swarnima Scheme for Women is associated with

(a) Ministry of Labour and Employment

(b) Ministry of Social Justice and Empowerment

(c) Ministry of Skill Development and Entrepreneurship

(d) Ministry of Women and Child Development

Ans. b

Exp: National Backward Classes Finance and Development Corporation (NBCFDC), an

organization under the aegis of Ministry of Social Justice and Empowerment has this women

specific scheme for targeted women whose annual family income is less than Rs. 3.00 lakh per annum:

Source: https://pib.gov.in/newsite/PrintRelease.aspx?relid=191766

Q8.Consider the following statements

Under Pradhan Mantri KisanSamman Nidhi (PM-KISAN)

1. All Institutional land holders and vulnerable landholding farmer families, having cultivable land

upto 1.5 hectares, will be provided direct income support.

2. Common Service Centres (CSCs) have also been authorized to do registration of the farmers for

the Scheme upon payment of fees.

Which of the statements given above is/are not correct?

(a) 1 only

(b) 2 only

https://t.me/UPSC_PDF Website ➡ https://upscpdf.com https://t.me/UPSC_PDF

Website ➡ https://upscpdf.com

Current Economics MCQs (Jan

2019 to Feb 20) + Eco Survey 2020

WriteToBeIAS.com Prelims 2020 Online Classes

by CA Rahul Kumar

Telegram https://t.me/writetobeias1 Youtube https://tinyurl.com/yx22y2yl Page 123

(c) Both 1 and 2

(d) Neither 1 nor 2

Ans. a

Exp: Under Pradhan Mantri KisanSamman Nidhi (PM-KISAN) vulnerable landholding farmer

families, having cultivable land upto2 hectares, will be provided direct income support. All

Institutional Land holders are not eligible for the PM-Kisan Scheme.

Source:https://pib.gov.in/newsite/PrintRelease.aspx?relid=187932

https://www.pmkisan.gov.in/

Q9. Consider following statements about PM-Kisan Samman Nidhi Yojana?

1. The Farmer should be a citizen of India.

2. Those Small and marginal farmer families who have combined land holding or ownership of up to

2 hectares are eligible to apply.

3. The Persons who paid Income Tax in last assessment year are also eligible to apply

Select the correct answer using the code given below:

(a) 1 and 2 only

(b) 2 only

(c) 3 only

(d) 2 and 3 only

Ans: a

Exp:Eligibility for PM Kisan Samman Nidhi Yojana

The Farmer should be a citizen of India.

Those Small and marginal farmer families who have combined land holding or ownership of up to 2

hectares are eligible to apply.

All the Institutional Land holders are not eligible to apply for this scheme.

Central or State Government officers and employees who are serving or retired except Multi Tasking

Staff /Class IV/Group D employees are not eligible to apply.

All superannuated/retired pensioners whose monthly pension is above Rs.10,000/- except MTS/ Class

IV/Group D employees are not eligible to apply.

The Persons who paid Income Tax in last assessment year are not eligible to apply.

Professionals like Doctors, Engineers, Lawyers, Chartered Accountants, Architects, etc are higher

economic status persons so there are not eligible to apply for this scheme.

https://pib.gov.in/PressReleseDetail.aspx?PRID=1573091

http://pmkisansammanyojana.com/#Eligibility

Q10. Consider the following:

1. All landholder farmers’ families in the country are eligible for the PM-Kisan Scheme.

2. Under the PM-Kisan Scheme an income support of Rs.6000/- per year is provided to all farmer

families across the country in three equal instalments of Rs.2000/- each every four months.

Which of the statements given above is/are correct?

(a) 1 only

(b) 2 only

(c) Both 1 and 2

(d) Neither 1 nor 2

Ans: b

Exp: All landholder farmers’ families in the country are eligible for the PM-Kisan Scheme, except

those covered under the following exclusion criteria:

(I) All Institutional Land holders; and

(II) Farmer families in which one or more of its members belong to following categories :

https://t.me/UPSC_PDF Website ➡ https://upscpdf.com https://t.me/UPSC_PDF

Website ➡ https://upscpdf.com

Current Economics MCQs (Jan

2019 to Feb 20) + Eco Survey 2020

WriteToBeIAS.com Prelims 2020 Online Classes

by CA Rahul Kumar

Telegram https://t.me/writetobeias1 Youtube https://tinyurl.com/yx22y2yl Page 124

Former and present holders of constitutional posts

Former and present Ministers/ State Ministers and former/present Members of Lok Sabha/ Rajya

Sabha/ State Legislative Assemblies/ State Legislative Councils, former and present Mayors of

Municipal Corporations, former and present Chairpersons of District Panchayats.

All serving or retired officers and employees of Central/ State Government Ministries

/Offices/Departments and its field units Central or State PSEs and Attached offices /Autonomous

Institutions under Government as well as regular employees of the Local Bodies (Excluding Multi-

Tasking Staff / Class IV/Group D employees)

All superannuated/retired pensioners whose monthly pension is Rs.10,000/-or more (Excluding

Multi-Tasking Staff / Class IV/Group D employees)

All Persons who paid Income Tax in last assessment year.

vi) Professionals like Doctors, Engineers, Lawyers, Chartered Accountants, and Architects registered

with Professional bodies and carrying out profession by undertaking practices.

https://pib.gov.in/PressReleseDetail.aspx?PRID=1575610

Q11.Which of the following is/are the aims of “Digital India” Plan of the Government of India?

1.Formation of India’s own Internet companies like China did.

2.Establish a policy framework to encourage overseas multinational corporations that collect Big Data

to build their large data centres within our national geographical boundaries.

3.Connect many of our villages to the Internet and bring Wi-Fi to many of our schools, public places

and major tourist centres.

Select the correct answer using the code given below:

(a)1 and 2only

(b)3 only

(c)2 and 3 only

(d)1, 2 and 3

Ans:b

Exp: The Digital India programme is centred on three key vision areas:

Digital Infrastructure as a Core Utility to Every Citizen, Governance and Services on Demand, Digital

Empowerment of Citizens.

Source: yojana September

Q12.Consider the following statements

1. Better health outcomes

2. Achieving INDC targets

3.Women empowerment

4.Alternate livelihood opportunities

Which of the following statements are the positive outcomes of UJJWALA Yojana?

(a)1,2,3

(b)2, 3, 4

(c)1, 3, 4

(d)All

Ans:d

Exp:Pradhan Mantri Ujjwala Yojana: It aims to provide LPG (liquefied petroleum gas) connection

stopo or households. Under the scheme, an adult woman member of a below poverty line family

identified through the Socio-Economic Caste Census (SECC) is given a deposit-free LPG connection

with financial assistance of Rs 1,600 per connection by the Centre. The scheme is being implemented

by the Ministry of Petroleum and Natural Gas. Key objectives of the scheme are:

1.Empowering women and protecting their health.

https://t.me/UPSC_PDF Website ➡ https://upscpdf.com https://t.me/UPSC_PDF

Website ➡ https://upscpdf.com

Current Economics MCQs (Jan

2019 to Feb 20) + Eco Survey 2020

WriteToBeIAS.com Prelims 2020 Online Classes

by CA Rahul Kumar

Telegram https://t.me/writetobeias1 Youtube https://tinyurl.com/yx22y2yl Page 125

2.Reducing the serious health hazards associated with cooking based on fossil fuel.

3.Reducing the number of deaths in India due to un clean cooking fuel.

4.Preventing young children from significant number of acute respiratory illnesses caused due to

indoor air pollution by burning the fossil fuel.

It is found that the positive outcomes have a spiralling effect on women empowerment as time saved

to search and carry firewood from near by forests is saved and used to spend on other alternate

livelihood opportunities particularly in SHGs.

Source: Yojana September

Q13. Consider the following statements with respect to Atal Bhoojal Yojana

1. The emphasis of the scheme will be on recharge of ground water sources and revival of surface

water bodies by involving people at the local level.

2. The half of the fund will be supported by a World Bank loan and the rest will be funded by state

government.

Which of the statement(s) given above is/are correct?

(a) 1 only

(b) 2 only

(c) Both 1 and 2

(d) Neither 1 nor 2

Ans: a

Exp:

Atal Bhoojal Yojana is aimed at efficient management of available water resources.

The emphasis of the scheme will be on recharge of ground water sources and revival of surface

water bodies by involving people at the local level.

It will focus on demand side management (how to meet requirements by minimum use of water).

The half of the fund will be supported by a World Bank loan of 3000 crore and the rest will be

funded by central government.

It would initially be implemented with community participation in Gujarat, Maharashtra, Haryana,

Karnataka, Rajasthan, Uttar Pradesh and Madhya Pradesh.

It focuses primarily on involvement of communities and convergence with different water scheme.

https://www.thehindu.com/news/national/world-bank-nod-for-6000-cr-groundwater-recharge-

plan/article24097117.ece

Q14. Consider the following statements with respect to Kisan Urja Suraksha evam Utthaan

Mahabhiyan (KUSUM) Scheme

1. The Scheme aims to provide last mile electrical connectivity to all rural and urban households.

2. The scheme required the consumer to mandatorily have a bank account linked with Aadhaar

number for availing LPG Subsidy.

3. It is implemented by Ministry of New and Renewable Energy.

Which of the statement(s) given above is/are correct?

(a) 3 only

(b) 1 and 2 only

(c) 2 and 3 only

(d) None

Ans: a

Exp:

Kisan Urja Suraksha evam Utthaan Mahabhiyan (KUSUM) is a farmer-oriented solar power

scheme that will allow setting up grid-connected solar plants in rural areas and off-grid solar pumps.

It involves decentralized solar power production of up to 28,250 Megawatt (Mw) over five years.

https://t.me/UPSC_PDF Website ➡ https://upscpdf.com https://t.me/UPSC_PDF

Website ➡ https://upscpdf.com

Current Economics MCQs (Jan

2019 to Feb 20) + Eco Survey 2020

WriteToBeIAS.com Prelims 2020 Online Classes

by CA Rahul Kumar

Telegram https://t.me/writetobeias1 Youtube https://tinyurl.com/yx22y2yl Page 126

It aims at boosting farmers’ income by allowing them to sell additional power to the grid through

solar plants.

It is implemented by Ministry of New and Renewable Energy.

https://www.thehindu.com/business/cabinet-approves-phase-ii-of-rooftop-solar-

programme/article26314512.ece

Q15. Consider the following statements regarding Swachh Bharat Mission (Grameen) (SBM-G )

1. Construction of toilets in government schools will be done by the Ministry of Women and Child

Development

2. Construction of toilets in Aganwadis will be done by the Ministry

Which of the statements given above is/are correct?

(a) 1 only

(b) 2 only

(c) Both 1 and 2

(d) Neither 1 nor 2

Ans: d

Exp: Construction of toilets in government schools will be done by the Ministry of Women and Child

Development.Construction of toilets in Aganwadis will be done by the Ministry of Women and Child

Development

http://pib.nic.in/newsite/PrintRelease.aspx?relid=189803

Q16.With reference to the Pradhan Mantri Sahaj Bijli Har Ghar Yojana (SAUBHAGYA) Scheme in

India, which of the following statements is/are correct?

1. Electricity connections will be given to APL families for Rs 500, which will be payable in 10 equal

monthly instalments

2. It also includes the Repair and Maintenance (R&M) for 5 years.

Select the correct answer using the code given below:

(a) 1 only

(b) 2 only

(c) Both 1 and 2

(d) Neither 1nor 2

Ans: c

Exp: Pradhan Mantri Sahaj Bijli Har Ghar Yojana - Saubhagya is to provide energy access to all by

last mileconnectivity and electricity connections to all remaining un-electrified households in rural

as well asurban areas to achieve universal household electrification in the country.The electricity

connection to households include release of electricity connections by drawing a service cable from

the nearest pole to the household premise, installation of energy meter, wiring for a single light point

with LED bulb and a mobile charging point. In case the electricity pole is notavailable nearby from

household for drawing service cable, the erection of additional pole alongwith conductor and

associated accessories shall also be covered under the scheme.

https://www.india.gov.in/spotlight/pradhan-mantri-sahaj-bijli-har-ghar-yojana-saubhagya

http://yojana.gov.in/Recent_archive_English_2018.asp

Q17.Regarding the features of Deendayal Antyodaya Yojana (DAY) - urban, which of the following

statements is not correct?

(a) It is implemented by Ministry of Housing & Urban Poverty Alleviation

(b) Normally, Funding will be shared between the Centre and the States in the ratio of 75:25.

(c) Cost of construction of shelters for urban homeless is fully funded under the Scheme.

(d) None of the above

https://t.me/UPSC_PDF Website ➡ https://upscpdf.com https://t.me/UPSC_PDF

Website ➡ https://upscpdf.com

Current Economics MCQs (Jan

2019 to Feb 20) + Eco Survey 2020

WriteToBeIAS.com Prelims 2020 Online Classes

by CA Rahul Kumar

Telegram https://t.me/writetobeias1 Youtube https://tinyurl.com/yx22y2yl Page 127

Ans: d

Exp: Deen Dayal Antyodaya Yojana (DAY) with an aim to uplift the urban poor folks by

enhancingsustainable livelihood opportunities through skill development. Keeping in view the

objective ofMake in India, Skill Development is essential for socio economic betterment. Deen Dayal

AntyodayaYojana was launched under the Ministry of Housing and Urban Poverty Alleviation

(HUPA).Government of India has provisioned Rs.500 crore for the scheme. The scheme is integration

of the National Urban Livelihoods Mission (NULM) and National RuralLivelihoods Mission

(NRLM).

https://www.india.gov.in/spotlight/deen-dayal-antyodaya-yojana

http://yojana.gov.in/Recent_archive_English_2018.asp

Q18. Consider following statements related to Mid-Day Meal-

1. Cooking cost is linked to Inflation Index.

2. Mid-Day Meal Scheme (MDMS) covers all school children studying in Classes I-VIII of

Government, Government-Aided Schools.

Which of the statements given above is/are correct?

(a) 1 only

(b) 2 only

(c) Both 1 and 2

(d) Neither 1 nor 2

Ans: c

Exp: Cooking cost for MDM is linked to Inflation Index.Mid-Day Meal Scheme (MDMS) covers all

school children studying in Classes I-VIII of Government, Government-Aided Schools.

http://pib.nic.in/PressReleseDetail.aspx?PRID=1565284

Q19. Consider following statements:

1. Energy Efficiency Services Limited (EESL) is a joint venture of only two national Public-Sector

Undertakings.

2. It is implementing agency for Unnat Jyoti by Affordable LEDs for All (UJALA).

Which of the statements given above is/are correct?

(a) 1 only

(b) 2 only

(c) Both 1 and 2

(d) Neither 1 nor 2

Ans: b

Exp: Energy Efficiency Services Limited (EESL) is a joint venture of four national Public-Sector

Undertakings. It is implementing agency for Unnat Jyoti by Affordable LEDs for All (UJALA).

https://www.thehindu.com/business/Industry/eesl-interview/article26425163.ece

Q20. Pradhan Mantri Shram Yogi Maan-dhan (PM-SYM) Yojana is

(a) a pension scheme for workers in the unorganised sector

(b) a pension scheme for workers in the organised sector

(c) Both of the above

(d)None of the above

Ans: a

Exp: Pradhan Mantri Shram Yogi Maan-dhan (PM-SYM) Yojana, a pension scheme for workers in

the unorganised sector.

https://www.thehindu.com/news/cities/chennai/defence-minister-launches-pm-sym-pension-

scheme/article26441711.ece

https://t.me/UPSC_PDF Website ➡ https://upscpdf.com https://t.me/UPSC_PDF

Website ➡ https://upscpdf.com

Current Economics MCQs (Jan

2019 to Feb 20) + Eco Survey 2020

WriteToBeIAS.com Prelims 2020 Online Classes

by CA Rahul Kumar

Telegram https://t.me/writetobeias1 Youtube https://tinyurl.com/yx22y2yl Page 128

Q21.Consider the following statements related to Pradhan Mantri Ujjwala Yojana (PMUY):

1. LPG connections will be provided to BPL families with a support of Rs.2000 per connection.

2. As per a WHO report, smoke inhaled by women from unclean fuel is equivalent to burning 400

cigarettes in an hour.

Which of the statements given above is/are correct?

(a) 1 only

(b) 2 only

(c) Both 1 and 2

(d) Neither 1 nor 2

Ans: b

Exp: LPG connections will be provided to BPL families with a support of Rs.1600 per connection.

2. As per a WHO report, smoke inhaled by women from unclean fuel is equivalent to burning 400

cigarettes in an hour.

http://yojana.gov.in/19-3-19%20%20March%20Yojana%20Final.pdf

Q22. With reference to the ‘"Housing for All by 2022" Mission- URBAN, which of the

following statements is/are correct ?

1. It is implemented by the Ministry of Housing & Urban Poverty Alleviation

2. It is a Centrally Sponsored Scheme, except the credit linked subsidy component, which will be

implemented as a Central Sector Scheme

3. Ownership of houses will be in the name of woman and never jointly with husband

Select the correct answer using the code given below :

(a) 1 and 2 only

(b) 2 only

(c) 2 and 3 only

(d) 1, 2 and 3

Ans: a

Exp:

It is implemented by the Ministry of Housing & Urban Poverty Alleviation. It is a Centrally

Sponsored Scheme, except the credit linked subsidy component, which will beimplemented as a

Central Sector Scheme Ownership of houses will be in the name of woman OR jointly with husband.

http://yojana.gov.in/Recent_archive_English_2018.asp

Q23. Consider following statements:

1. Rashtriya Vayoshri Yojana (RVY) is scheme administered by The Ministry of Women and Child

Development.

2. Under the Scheme, physical aids and Assisted Living Devices are provided free of cost to the

beneficiary Women belonging to BPL category.

Which of the statements given above is/are correct?

(a) 1 only

(b) 2 only

(c) Both 1 and 2

(d) Neither 1 nor 2

Ans: d

Exp: The Ministry of Social Justice and Empowerment has introduced a scheme for providing

physical aids and Assisted Living Devices for Senior Citizens belonging to BPL category named

“Rashtriya Vayoshri Yojana (RVY)”. Under the Scheme, assisted living devices such as Walking

https://t.me/UPSC_PDF Website ➡ https://upscpdf.com https://t.me/UPSC_PDF

Website ➡ https://upscpdf.com

Current Economics MCQs (Jan

2019 to Feb 20) + Eco Survey 2020

WriteToBeIAS.com Prelims 2020 Online Classes

by CA Rahul Kumar

Telegram https://t.me/writetobeias1 Youtube https://tinyurl.com/yx22y2yl Page 129

Sticks, Elbow Crutches, Walkers/ Crutches, Tripods/ Quadpods, Hearing Aids, Wheelchairs,

Artificial Dentures and Spectacles are provided free of cost to the beneficiary senior citizens.

http://www.pib.nic.in/PressReleseDetail.aspx?PRID=1562683

Q24. Consider the following statements:

1. Uchhatar Avishkar Yojana (UAY) is 100% industry sponsored Scheme.

2. One of the objectives of UAY scheme is to promote innovation in IITs.

Which statement/statements given above is/are correct?

Select the correct answer using the codes given below.

(a) 1 only

(b) 2 only

(c) Both 1 and 2

(d) None

Ans: b

Exp: Uchhatar Avishkar Yojana (UAY) UAY promotes industry sponsored, outcome-oriented

research projects with an outlay of Rs. 475.00 crore for a period of two years beginning 2016-17. The

project cost is met to the extent of 50 per cent by MHRD and 25 per cent each by the Industry and

host Institute. The objectives of UAY scheme are to promote innovation in IITs, connect with

manufacturing industries, spur innovative mindset and promote collaboration and cooperation

between academia and industry.

http://yojana.gov.in/Yojana%20January2019.pdf

Q25. Which of the following is a ‘Scheme for Life Insurance’:-

(a) The Pradhan Mantri Jan Dhan Yojana (PMJDY)

(b) The Pradhan Mantri Jeevan Jyoti Bima Yojana (PMJJBY)

(c) The Pradhan Mantri Suraksha Bima Yojana (PMSBY)

(d) The Atal Pension Yojana (APY)

Ans: b

Exp: The Pradhan Mantri Jeevan Jyoti Bima Yojana (PMJJBY) is a one-year life insurance scheme,

renewable from year to year, offering coverage for death due to any reason and is available to people

in the age group of 18 to 50 years (life cover upto age 55) having a savings bank account who give

their consent to join and enable auto-debit Life cover of Rs. 2 lakhs is available for a one year period

stretching from June 1 to May 31 at a premium of Rs.330/- per annum per member and is renewable

every year which means premium of less than Re 1 per day.

http://yojana.gov.in/Yojana%20January2019.pdf

MISCELLANEOUS

Q1. With reference to IRSED, consider the following

1. The third meeting of the IRSED focussed on six core areas of cooperation, namely, Development

of Transport Infrastructure and Technologies; Development of Agriculture and Agro-Processing

sector; Small and Medium Business support; Digital Transformation and Frontier Technologies;

Cooperation in Trade, Banking, Finance, and Industry; and Tourism & Connectivity.

2. The IRSED was established following a bilateral MoU signed between NITI Aayog and the

Ministry of Economic Development of the Russian Federation during the 19th edition of the Annual

India-Russia Bilateral Summit.

3. The meeting was attended by finance ministers and senior leaders of both the countries to help

enhance strategic economic cooperation between our countries

Which of the statements given above is/are incorrect?

https://t.me/UPSC_PDF Website ➡ https://upscpdf.com https://t.me/UPSC_PDF

Website ➡ https://upscpdf.com

Current Economics MCQs (Jan

2019 to Feb 20) + Eco Survey 2020

WriteToBeIAS.com Prelims 2020 Online Classes

by CA Rahul Kumar

Telegram https://t.me/writetobeias1 Youtube https://tinyurl.com/yx22y2yl Page 130

Select the correct answer using the code given below:

(a) 1 and 3 only

(b) 1, 2 and 3

(c) 2 and 3 only

(d) 3 only

Ans.d

Exp:First India-Russia Strategic Economic Dialogue was held in St. Petersburg between November

25-26, 2018.Second India-Russia Strategic Economic Dialogue (IRSED) was recently held in New

Delhi.

The Second India-Russia Strategic Economic Dialogue (IRSED) was held today in New Delhi, under

the chairmanship of Dr. Rajiv Kumar, Vice-Chairman, National Institution for Transforming India

(NITI) Aayog and Mr. Timur Maksimov, Deputy Minister of the Economic Development of the

Russian Federation.

Participants included government officials, business leaders and experts, bringing in diverse

perspectives for ideation.

Source: https://pib.gov.in/newsite/PrintRelease.aspx?relid=191522

Q2. ‘Indian Performing Rights Society(IPRS)’recently seen in news is a/an:

(a) Representative body of artists and music owners

(b)Theatre network

(c)International child rights network

(d)Anti-drug trafficking organisation

Ans:a

Exp: The IPRS is a representative body of artists and music owners, including composers, lyricists,

authors and the publishers of music. The group has initiated the process of seeking royalties from

Indian telecom companies and platforms such as Apple and YouTube for the use of their works,

including songs and music made available for streaming and downloads.

Source:https://economictimes.indiatimes.com/industry/media/entertainment/indian-performing-

rights-society-wants-royalties-for-music-used-by-telcos/articleshow/62941437.cms

Q3.The author of book “Good economics for hard times” is

(a) Esther Duflo

(b) CRangarajan

(c)AmartyaSen

(d)Elinor Ostrom

Ans: a

Exp: Poor Economics: A Radical Rethinking of the Way to Fight Global Poverty (2011) is a non-

fiction book by Abhijit V. Banerjee and Esther Duflo, both professors of Economics at MIT.

https://www.thehindu.com/opinion/editorial/economics-of-poverty/article29683404.ece

Q4.Which one of the following is a partner with World Food Programme to develop the new global

hunger monitoring system called “Hunger Map LIVE”?

(a) Amazon

(b) Google

(c) Alibaba Cloud

(d) Walmart

Ans. c

https://t.me/UPSC_PDF Website ➡ https://upscpdf.com https://t.me/UPSC_PDF

Website ➡ https://upscpdf.com

Current Economics MCQs (Jan

2019 to Feb 20) + Eco Survey 2020

WriteToBeIAS.com Prelims 2020 Online Classes

by CA Rahul Kumar

Telegram https://t.me/writetobeias1 Youtube https://tinyurl.com/yx22y2yl Page 131

Exp: WFP has developed a new global hunger monitoring system called “Hunger Map LIVE” in

partnership with Alibaba Cloud, to help assess, monitor and visualize the severity and scope of hunger

in over 90 countries.

Source: http://mvam.org/2019/10/22/introducing-hunger-map-live/

Q5. The ‘Kimberley Process (KP)’ recently in the news, is related to

(a) Dispute resolution between parties

(b) Conflict diamonds

(c) Coal mining

(d) Gold standard

Ans: b

Exp: The Kimberley Process is a joint initiative involving Government, international diamond

industry and civil society to stem the flow of Conflict Diamonds. Conflict Diamonds means rough

diamonds used by rebel movements or their allies to finance conflict aimed at undermining legitimate

governments. It is also described in United Nations Security Council (UNSC) Resolutions.

https://pib.gov.in/PressReleseDetail.aspx?PRID=1574572

Q6. Ease of living index is released by

(a) Ministry of Housing and Urban Poverty Alleviation

(b) NITI Aayog

(c) NSSO

(d) Labour Bureau

Ans:a

Exp: Government’s aim to improve ease of living for the citizens. MoHUA released the first ever

‘Ease of Living Index’ in 2018 covering 111 Cities, and has recently launched assessment frameworks

for Ease of Living Index 2019. As per World Bank’s, Doing Business Report-2019, India’s rank in

Ease of Doing Business in Construction Permits has improved from 181 in 2018 to 52, i.e. a record

jump of 129 places. Online Building Permission System (OBPS) has been implemented in 1,705

cities including 439 AMRUT cities so far.

http://yojana.gov.in/FULLPDFYOJANAAUGUST2019.pdf

Q7.The 2019 Sveriges Riksbank Prize in Economic Sciences was conferred on

(a)Abhijit Banerjee

(b)Amartya Sen

(c)Jean Dreze

(d)None of the above

Ans: a

Exp: The Prime Minister has congratulated Abhijit Banerjee on being conferred the 2019 Sveriges

Riksbank Prize in Economic Sciences in Memory of Alfred Nobel.

https://pib.gov.in/PressReleseDetail.aspx?PRID=1587688

Q8.Consider the following statements:

1. Institute for Competitiveness , India is the Indian knot in the global network of the Institute for

Strategy and Competitiveness at London School of Economics.

2.Institute for Competitiveness, India conducts & supports indigenous research; offers academic &

executive courses; provides advisory services to the Corporate & the Government.

https://t.me/UPSC_PDF Website ➡ https://upscpdf.com https://t.me/UPSC_PDF

Website ➡ https://upscpdf.com

Current Economics MCQs (Jan

2019 to Feb 20) + Eco Survey 2020

WriteToBeIAS.com Prelims 2020 Online Classes

by CA Rahul Kumar

Telegram https://t.me/writetobeias1 Youtube https://tinyurl.com/yx22y2yl Page 132

Which of the above statements is/are correct?

(a)1 only

(b)2 only

(c)Both 1 and 2

(d)Neither 1 nor 2

Ans: b

Exp: Institute for Competitiveness, India is the Indian knot in the global network of the Institute for

Strategy and Competitiveness at Harvard Business School. Institute for Competitiveness, India

conducts & supports indigenous research; offers academic & executive courses; provides advisory

services to the Corporate & the Governments and organises events.

https://pib.gov.in/PressReleseDetail.aspx?PRID=1587688

Q9. Consider following statements w.r.t. NITI Aayog :-

1. The Vice President is the Ex-officio chairman.

2. Its Governing Council consists of Chief Ministers of all the States and Union territories with

Legislatures and lieutenant governors of Union Territories.

Which of the statements given above is/are correct?

Select the correct answer using the code given below:

(a) 1 only

(b) 2 only

(c) Both 1 and 2

(d) Neither 1 nor 2

Ans: b

Exp: The Prime Minister is the Ex-officio chairman. A Governing Council composed of Chief

Ministers of all the States and Union territories with Legislatures and lieutenant governorsof Union

Territories (except Delhi and Pondicherry)

https://www.thehindu.com/todays-paper/tp-business/ensure-borrowerdiscipline/article26727973.ece

Q10. Kafala system sometimes seen in the news recently was followed by which of the following

countries?

(a) Qatar

(b) Zambia

(c) Indonesia

(d) Maldives

Ans: a

Exp:

Qatar is set to abolish its controversial exit visa system for all foreign workers by the end of 2019

the UN’s International Labour Organization has recently said.

The Kafala (Sponsorship) System emerged in the 1950’s to regulate the relationship between

employers and migrant workers in many countries in West Asia.

It remains the routine practice in the Gulf Cooperation Council (GCC) countries of Bahrain,

Kuwait, Oman, Qatar, Saudi Arabia and the United Arab Emirates (UAE),

and also in the Arab states of Jordan and Lebanon.

Under the Kafala system a migrant worker’s immigration status is legally bound to an individual

employer or sponsor (kafeel) for their contract period.

The migrant worker cannot enter the country, transfer employment nor leave the country for any

reason without first obtaining explicit written permission from thekafeel.

https://www.thehindu.com/news/international/qatars-exit-visa-system-to-end-this-year-

un/article26958686.ece

https://t.me/UPSC_PDF Website ➡ https://upscpdf.com https://t.me/UPSC_PDF

Website ➡ https://upscpdf.com

Current Economics MCQs (Jan

2019 to Feb 20) + Eco Survey 2020

WriteToBeIAS.com Prelims 2020 Online Classes

by CA Rahul Kumar

Telegram https://t.me/writetobeias1 Youtube https://tinyurl.com/yx22y2yl Page 133

Q11. ‘The Theory of Moral Sentiments’ book is written by:-

(a) John Maynard Keynes

(b) Friedrich August von Hayek

(c) Karl Marx

(d) Adam Smith

Ans: d

Exp: Smith’s The Theory of Moral Sentiments and An Inquiry into the Nature and Causes of the

Wealth of Nations make it clear that he was not a laissez faire or free market capitalismapologist.

https://www.thehindu.com/todays-paper/tp-opinion/why-adam-smith-favoured-public-

education/article25954760.ece

Q12. The coin of Rs 350 denomination has been released by Government recently to mark the Birth

Anniversary of :-

(a) Guru Nanak Dev

(b) Guru Govind Singh

(c) Baba Bhim Rao Ambedkar

(d) Mahatma Buddha

Ans: b

Exp: a coin of Rs 350 denomination to mark the 350th Birth Anniversary of Shri Guru Gobind Singh

Ji.

http://pib.nic.in/PressReleseDetail.aspx?PRID=1559767

https://t.me/UPSC_PDF Website ➡ https://upscpdf.com https://t.me/UPSC_PDF

Website ➡ https://upscpdf.com